You are on page 1of 112

BALIUAG UNIVERSITY

CPA Review Program


Advanced Financial Accounting and Reporting
Final Preboard Examination

Instructions: Select the best answer for each of the following questions. Mark only one answer for each item on the
answer sheets provided. Strictly NO ERASURES ALLOWED.

1. Comparative trial balance of the home office of ACE Corporation and its two branches at December 31, 2016 were as
follows:

Home Office CHI Branch MIN Branch


Cash P 5,000 P15,000 P22,000
Accounts receivable 80,000 30,000 40,000
Inventories 150,000 60,000 48,000
CHI Branch 170,000
MIN Branch 165,000
Plant assets 730,000 250,000 200,000
Purchases 900,000
Shipments from home office 300,000 240,000
Expenses 300,000 75,000 50,000
Total P2,500,000 P730,000 P600,000

Accounts payable P100,000 P45,000 P30,000


Other liabilities 80,000 15,000 5,000
Loading in branch inventories 108,000
Share capital, P10 par 500,000
Retained earnings 262,000
Home office 170,000 165,000
Sales 1,000,000 500,000 400,000
Shipment branches 450,000
Totals P2,500,000 P730,000 P600,000

Additional information:
Home office and branch inventories at December 31, 2016 were:
Home office P120,000
CHI Branch (at billed prices) P 72,000
MIN Branch (at billed prices) 96,000
How much net income will ACE Corporation report for year-ended 2016?
A. P260,000
B. P122,000
C. P220,000
D. P595,000

2. Which of the following is not considered a legitimate expense of a partnership?


A. Interest paid to partners based on the amount of invested capital.
B. Depreciation on assets contributed to the partnership by partners.
C. Salaries for management hired to run the partnership.
D. Supplies used in the partners’ offices.

3. The estimated unit costs for Lodge Inc., when it is operating at a production and sales level of 12,000 units, are as
follows:

Estimated
Cost Item Unit Cost
Direct materials P 32
Direct labor 10
Variable factory overhead 15
Fixed factory overhead 6
Variable marketing 3
Fixed marketing 5
AFAR Page 1 of 12
Final Preboard Examination AFAR

Compute the total cost that would be incurred during a month with a production level of 11,500 units and a sales
level of 9,500 units.

A. P800,500 B. P813,000 C. P816,000 D. P852,000

4. If the partnership does not specify how income is to be allocated, profit and losses should be allocated
A. Equally.
B. In accordance with partners’ capital contribution.
C. In proportion to the weighted average of capital invested during the period.
D. Equitably so that partners are compensated for the time and effort expended on behalf of the partnership.

5. Riverdale Company incurred P80,000 direct labor cost in 2014 and had the following selected account balances at
the beginning and end of 2017: Finished goods January 1, P56,000; Work in process January 1, P24,000; Materials
January 1, P34,000; Finished goods December 31, P90,000; Work I process December 31, P28,000; Materials
December 31,0P48,000. The total cost of goods sold and actual factory overhead during the year are P280,000 and
P70,000 respectively.

Determine the total material purchases during the year.


A. P182,000 B. P162,000 C. P148,000 D. P144,000

6. When the retiring partner’s interest is greater than the settlement price, using bonus method, the difference is
A. Debited to the remaining partners’ capital accounts
B. Credited to the retiring partner’s capital account
C. Credited to the remaining partner’s capital accounts
D. Recognized in the profit or loss for the period.

7. The following data summarizes in part the results of operations for 2017 of Veronica Company.
Of the total cost of goods manufactured for 2017, 38% was for materials used, 30% for direct labor, and 32% for
manufacturing overhead.
During 2017, the company paid for 90% of the materials purchased, leaving P293,000 of unpaid invoices for
materials at year end. The company commenced 2017 operations with a materials inventory of P421,000. All
materials were purchased f.o.b. company’s plant.
The company disbursed P2,101,500 for direct labor during 2011. As of Dec. 31, 2017, the accrued liability for
direct amounted to P144,000, which was twice as much as last year’s accrual.
The inventory of finished goods on December 31, 2017, was 10% of the cost of the units fifnished during the
year, and goods in process on that date were one-half the finished goods inventory. This year’s finished goods
inventory was 150% of last year’s. there are no goods in process last year.
The manufacturing overhead, except for depreciation of factory buildings and equipment, is detailed below:
Indirect labor P 672,000
Heat, light, and power 226,200
Maintenance and repairs 488,300
Insurance – factory 18,100
Property taxes 64,400
Factory payroll taxes 97,000
Miscellaneous factory costs 276,400
P 1,842,400
How much is the cost of sales during 2017?
A. P 7,245,000 B. P 6,900,000 C. P 6,670,000 D. P5,757,500

8. An enterprise uses a branch accounting system in which it establishes separate formal accounting systems for its home
office operations and its branch office operations. Which of the following statements about this arrangement is false?
A. The home office account on the books of a branch office represents the equity interest of the home office in the net
assets of the branch.
B. The home office and branch office accounts are reciprocal accounts that must be eliminated in the preparation of the
enterprise’s financial statements that are presented in accordance with GAAP.
C. Unrealized profit from internal transfers between the home office and a branch must be eliminated in the
preparation of the enterprise’s financial statements that are presented in accordance with GAAP.
D. The branch office account on the books of the home office represents the equity interest of the branch office in the
net assets of the home office.

AFAR Page 2 of 12
Final Preboard Examination AFAR
9. During 2018, Jughead Company purchased materials costing P152,600. Materials requisitioned for jobs cost P98,000,
and indirect materials costing P42,000 were charged to Factory Overhead. Factory payrolls were P212,000 with
payroll taxes deducted of P60,000. Indirect labor of P71,000 included in the payrolls was charged to factory
Overhead. All other labor was direct labor charged to the jobs. Factory overhead was applied to the jobs at the rate
of P8 per machine hour. During the year, the company operated at 45,000 machine hours and incurred factory
overhead costs of P259,000 (in addition to the indirect materials and indirect labor previously stated). Depreciation
of P47,000 was included in the P259,000 of factory overhead costs.

Product costing P465,000 were completed during the year, and the cost of goods sold was P480,000. At the
beginning of the year, Jughead had the following balances:
Materials P27,000; Work in Process P48,000; Finished Goods P34,000
Determine the cost of work in process ending inventory for 2018.
A. P134,000 B. P182,000 C. P167,000 D. P194,000

10. Allocated expenses from the home office will be recorded by the home office as debit to
A. Investment in Branch C. Home Office
B. Expense account D. Cash

Items 11 and 12 are based on the following:


Benny, Jenny, and Kenny are partners in a trading business. They participate in the profits and losses equally. As of
December 31, 2016, the partners’ capital and drawing accounts are as follows:
Benny Jenny Kenny Total
Capital P20,000 P16,000 P600,000 P96,000
Drawing 12,000 8,000 4,000 24,000

The partners decided to liquidate the partnership. The operating profit for the year 2016 amounted to P14,400, which
was all exhausted including partnership assets. As of December 31, 2016, unpaid liabilities still amounted to P16,800.
Benny is personally insolvent, but both Jenny and Kenny have substantial private resources.
11. The total loss on realization was:
A. P72,000 B. P88,800 C. P96,000 D. P103,200
12. The amount received by Kenny in final cash distribution was:
A. P15,600 B. P16,800 C. P21,600 D. P32,400

13. The face amount of a loans payable is greater than the net realizable value of the attached inventory lien. The
inventory shall be classified under
A. Partially secured creditors C. Assets pledged with partially secured creditors
B. Fully secured creditors D. Assets pledged with fully secured creditors
14. It is equals to the total assets under receivership minus total liabilities to be liquidated when the latter is greater
than the assets.
A. Estate equity C. Capital deficiency
B. Estate deficit D. Receivership net loss

15. Partners D, I, L, and G, share profits 40%, 30%, 15%, and 15%, respectively. Their partnership agreement provides
that in the event of the death of a partner, the firm shall continue until the end of the fiscal period. Profits shall be
considered to have been earned proportionately during this period, and the deceased partners’ capital shall be
adjusted by the proper share of the profit or loss until the date of death. From that date until the date of settlement
with the estate there shall be added interest at 6% computed on the adjusted capital.

The remaining partners shall continue to share profits in the old ratio. Payment to the estate shall be made within
one year from the date of the partner’s death. Partner G died on November 16. On December 31, the end of the six-
month period, account balances on the partnership books before the income summary account is closed are as
follows:
Cash P 7,500 Notes payable P15,000
Accounts receivable 70,000 Accounts payable 70,500
Inventories 95,000 D, capital 42,000
Machinery and equipment, net 45,000 I, capital 37,500
Store furniture and fixtures, net 16,500 L, capital 24,000
G, capital 22,500
Income summary
_______ (7/1-12/31) 22,500
P 234,000 P 234,000

AFAR Page 3 of 12
Final Preboard Examination AFAR
The income summary account is closed on December 31. On this date, L decides to retire. D and I agree to pay the
balance in L’s capital account after distributions of profit, less 20%, and issue a partnership 60-day, 6% note to L in
settlement.
What amount is due to G’s estate on December 31?
A. P22,500 B. P25,875 C. P25,062.50 D. P25,218.99

16. Loss absorption potential of a partner computed during the preparation of cash priority program is equals to
A. {Partner’s capital account plus partner’s loan (receivable)} multiplied by the respective PL ratio.
B. {Partner’s capital account minus partner’s loan (receivable)} divided by the respective PL ratio.
C. {Partner’s capital account plus partner’s loan (payable)} multiplied by the respective PL ratio.
D. {Partner’s capital account minus partner’s loan (payable)} divided by the respective PL ratio.

17. On December 1, 2017, Coach Corp. entered into a 120-day forward contract to purchase 250,000US dollars for
speculative purposes, Coach Corp. fiscal year ends on December 31. The exchange rates are as follows:
Date Spot rate Forward rate (3/31/10)
December 1, 2017 P45.00 P45.50
December 31, 2017 46.00 46.50
January 30, 2018 45.60 45.30
March 31, 2018 45.10
How much is the forex gain or loss to be reported from this forward contract in 2018?
A. P250,000 B. P350,000 C. P300,000 D. P225,000

18. The third step in the revenue recognition model framework under IFRS 15 is
A. Determine the transaction price
B. Recognize revenue when (or as) the entity satisfies a performance obligation
C. Allocate the transaction price to the performance obligations in the contract
D. Identify the performance obligations in the contract

19. MELROSE Corporation issued a promissory note denominated in foreign currency for the purchase made from an
Italian supplier. The following were the related transactions: (in Italy Lire). On December 1, MELROSE Corporation
purchased merchandise from an Italian supplier for 60-day 18% promissory note for 108,000 Italy lire, at a selling
rate 1FC to P74.20. On December 31, the selling spot rate is 1FC to P74.85. On January 30, the selling spot rate is 1FC
to P75.75

On the settlement date, how much is the foreign exchange gain/loss?


A. P98,658 loss B. P172,422 gain C. P100,116 loss D.P172,422 loss

20. SAVEMORE Corp. entered into a forward contract to hedge a sale of inventory in October 26, 2017 to be collected
on January 24, 2017. 72,000 FC (foreign currency) in 90 days. The relevant exchange rate as follows:
Spot rate Forward rate (1/24/18)
October 26, 2017 P52.73 P52.77
December 31, 2017 52.82 52.89
January 24, 2018 52.94
What is the net forex gain (loss) from this transaction and hedge that will be reported on SAVEMORE’s 2017
statement of income?
A. P15,120 B. (P8,640) C. P6, 480 D. (P2,160)

21. The Construction-in-Process account accumulates the following when the percentage-of-completion method is used
A. Construction costs to date. C. Construction costs plus gross profit earned to date.
B. Construction costs to date less billings to date. D. Construction costs to date less payments received

22. SBX Restaurant sold a fast food restaurant franchise to NKE Corporation. The sale agreement signed on January 2,
2016 called for a P70,000 down payment plus two P15,000 annual payments representing the value of initial
franchise services rendered by SBX. The present value factor of two annual payments appropriately discounted at
10% is 1 7355. In addition, the agreement required the franchise to pay 5% of its gross revenues to the franchisor;
this was deemed sufficient to cover the cost and provide a reasonable profit margin on continuing franchise services
to be performed by SBX. The restaurant opened early in 2016 and its sales for the year amounted to P600,000.
SBX Restaurant’s 2016 total revenue from the franchise will be:
A. P 0 B. P 30,000 C. P56,033 D. P 126,000

23. Under installment sales method,

AFAR Page 4 of 12
Final Preboard Examination AFAR
A. Revenue, costs and gross profit are recognized proportionately to the cash that is received from the sale of the
product.
B. Gross profit is not recognized until the amount of cash received exceeds the cost of the item sold.
C. Gross profit is deferred proportionately to cash uncollected from sale of the product, but total revenue and costs are
recognized at point of sale.
D. Revenues and costs are recognized proportionately to the cash received from the sale of the product, but gross profit
is deferred until all cash is received.

24. On January 1, 2015, Mrs. Fields entered into a franchise agreement with KK to market their products. The
agreement provides for an initial fee of P15,500,000 payable as follows: P6,500,000 to be paid upon signing of the
contract and the balances in five equal annual payments every end of the year starting December 31, 2015. Mrs.
Fields signs a non-interest bearing notes for the balance. His credit rating indicates that he can borrow money at
15% interest for a loan of this type. The present value of an annuity of P1 at 15% for 5 periods is 3.352. The
agreement further provides that the franchise must pay a continuing franchise fee equal to 3% of the monthly gross
sales. On August 31, the franchiser completed the initial services required in the contract at a cost of P7,290,120 and
incurred indirect cost of P475,000. The franchisee commenced business operations on November 30, 2015. The
gross sales reported to the franchisor were P1,800,000 for December, 2015. The first installment payment was made
in due date.
Assume the collectability of the note is not reasonably assured the net income for the year ended, December 31,
2015 is
A. P4,121,288 B.P5,026,268 C. P5,243,480 D. P4,551,268

25. According to IFRS 15, where a contract has multiple performance obligations, an entity will allocate the transaction
price to the performance obligations in the contract by reference to their relative
A. Net realizable values C. Fair market values
B. Stand-alone selling prices D. Fair value less cost of disposal

Love Corporation has been using the cash method to account for income since its first year of operations in 2016. All
sales are made on credit with notes receivable given by the customers. The income statements for 2016 and 2017
included the following amounts:
2016 2017
Revenues – collection on principal P96,000 P150,000
Revenues – interest 10,800 16,500
Cost of goods purchased* 151,800 156,060
*Includes increase in inventory of goods on hand of P6,000 in 2016 and P24,000 in 2017
The balance due on the notes at the end of each year were as follows:

2016 2017
Notes receivable – 2016 P186,000 P108,000
Notes receivable – 2017 180,000
Unearned interest revenue – 2016 21,501 16,737
Unearned interest revenue – 2017 24,129

26. Under the installment method, how much is the realized gross profit in 2016?
A. P48,240 B. P96,000 C. P53,667 D. P10,10632
27. Under the installment method, how much is the realized gross profit in 2017?
A. P36,801 B. P33,186 C. P69,987 D. P64,845

28. An entity may use residual approach per IFRS 15 when allocating transaction price under which circumstance(s)?
i. The entity sells the same good or service to different customers (at or near the same time) for a broad range of
amounts
ii. The entity has not yet established a price for that good or service and the good or service has previously been sold
on a stand-alone basis.
A. Both i and ii C. ii only
B. Neither i nor ii D. i only

29. With regard to the definition of revenue given by IFRS15, which of the following statements is true?
A. Revenue includes cash received from share issues
B. Revenue arises from ordinary activities only
C. Revenue includes cash received from borrowings
D. Revenue may arise from either ordinary activities or extraordinary activities

AFAR Page 5 of 12
Final Preboard Examination AFAR

Items 30 and 31 are based on the following:


Belgium Corporation owns 80 percent of the stock of Hillinger Company. At the end of 2017, Belgium Corp. and
Hillinger Company reported the following partial operating results and inventory balances:
Total sales Belgium Corp. Hillinger Co.
Sales to Hillinger Company P 660,000 P 510,000
Sales to Belgium Corporation 140,000
Net income 20,000
Operating income (excluding income from
Hillinger Company) 70,000
Inventory on hand, December 31, 2017
purchased from Hillinger Company 48,000
purchased from Belgium Corporation 42,000

Belgium Corporation regularly prices its products at cost plus a 40 percent markup for profit. Hillinger Company prices its
sales at cost plus a 20 percent markup. The total sales reported by Belgium and Hillinger include both intercompany
sales and sales to nonaffiliates.

30. The consolidated cost of sales for 2017 must be:


A. P596,428 B. P616,428 C. P576,428 D. P536,428

31. The controlling interest net income for 2017 must be:
A. P67,600 B. P70,000 C. P90,000 D. P86,000

32. Defined by IFRS 11 as a joint arrangement whereby the parties that have joint control of the arrangement have rights to the
assets, and obligations for the liabilities, relating to the arrangement.
A. Joint operation C. Jointly controlled asset
B. Jointly controlled entity D. Joint venture
33. According to IFRS 11, a party that participates in, but does not have joint control of, a joint venture shall account for
the investment (assume nominal interest) using
A. IFRS 11 C. IFRS 9
B. IAS 28 D. IAS 7

34. Clydes Bakery owns 60 percent of the stock of Good Shepherd Products acquired several years ago at book values.
On January 1, 2017, inventory reported by Clydes Bakery included 20,000 bags of flour purchased from Good
Shepherd Products at P90 per bag. By December 31, 2017, all these beginning inventory purchased from Good
Shepherd Products had been baked into products and sold to customers by Clydes Bakery. There were no
transactions between Clydes Bakery and Good Shepherd Products during 2017.

Both Clydes Bakery and Good Shepherd Products price their sales at cost plus 50% mark-up for profit. Clydes Bakey
reported income from its baking operations of P3,000,000 and Good Shepherd Products reported net income of
P2,500,000 for 2017.
Compute the consolidated net income attributable to controlling interest.
A. P3,420,000 B. P4,860,000 C. P4,500,000 D. P5,580,000

35. Under IFRS for SMEs, transaction cost during the initial measurement of an investment in a jointly controlled entity
shall be added when using
A. Cost model only C. Equity model and fair value model
B. Fair value model only D. Cost model and equity model

36. Viewsonic Manufacturing purchased 80 percent of the stock of Ronnie Mines, Inc., in 2015. In preparing the
consolidated financial statements at the end of 2017, the controller of Viewsonic discovered that Viewsonic
manufacturing had purchased P750,000 of raw materials from Ronnie Mines during the year and that the parent
company had not paid for the last purchase of P120,000. All the inventory purchased was still on hand at year-end.
Ronnie Mine has spent P500,000 in producing the items sold to Viewsonic Manufacturing.
What effect, if any, will failure to eliminate or adjust for these items have on total current assets reported in the
consolidated balance sheet on December 31, 2017?
A. overstated by P250,000 C. overstated P370,000
B. overstated by P870,000 D. overstated P750,000

37. Asset acquisition as business combination will most likely result in


A. Acquirer recording assets and liabilities of acquiree at fair value
AFAR Page 6 of 12
Final Preboard Examination AFAR
B. Acquiree continuing its existence.
C. Acquirer recording the acquisition in an investment account
D. Acquirer purchasing shares of stocks of investee

Motul Co., a manufacturing company used process costing under the FIFO method for its products. The products
underwent two departments namely Assembly then Finishing. The following data were extracted from the system in the
Finishing department: Beginning inventory units 10,000 which were 70% incomplete. Ending inventory units 5,000 which
were 35% converted, transferred-out units from the Assembly department 80,000 and spoiled units of 2,800 of which
2,500 were considered normal. In the Finishing department, materials were added at the end of the process.

The following were the costs in the Finishing department: Beginning inventory costs of Transferred-In, Direct Materials,
Conversion respectively were P150,000, P80,000, P76,000. Current costs of Direct Materials and Conversion were
P742,500 and P650,000 respectively. Transferred-out cost from the Assembly department amounted to P1,162,500.

38. What are the equivalent units of production as to direct materials?


A. 85,000 B. 82,200 C. 80,300 D. 82,500
39. What is the cost of completed goods?
A. 2,630,400 B. 2,762,400 C. 2,672,400 D. 2,720,000

40. Under IFRS 3, which of the following falls under the third step in the acquisition method of business combination?
A. Determination of the acquisition date
B. Recognition and measurement of non-controlling interest
C. Recognition and measurement of goodwill or a gain from a bargain purchase.
D. Identification of the acquirer
41. For contingent consideration under IFRS 3, changes in fair value are recognized in profit or loss for contingent consideration
classified as
A. Equity and asset/liability C. Asset/liability only
B. Equity only D. None, changes in fair value is recognized in OCI

42. Miles Company established a branch in Ayala by sending merchandise costing P924,500 and effecting a fund transfer
of P400,000 cash on January 1, 2016. The branch purchased computer equipment costing P420,000 on April 1. As
per agreement, the home office will maintain all the property, plant and equipment records.

Ayala branch collected P56,000 worth of Ortigas branch’s receivable on August 4. Cash remittance to the home was
P250,000 on September 28.

On November 21, Ayala branch returned defective merchandise worth P125,000 to the home office.

At the end of the year, the company’s controller found out that the branch accountant had failed to record all the
transactions initiated by the home office from the second half of the year. Because of this, there is a significant
discrepancy between the balances of the reciprocal accounts.

For the purpose of reconciling the reciprocal accounts, the controller instructed the accounting staff of the home
office to send a copy of the Investment in Ayala general ledger to the branch.

Investment in Ayala
1/1 Merchandise to branch P924,500 4/2 Equipment acquisition P240,000
1/1 Fund transfer 400,000 9/30 Remittance 225,000
7/2 Merchandise to branch 135,000 11/22 Return of goods from branch 12,500
8/31 Fund transfer 95,000

What is the unadjusted balance of the Home Office Account?


A. 598,500 B. 585,500 C. 723,500 D. 335,500

43. What is the adjusted balance of the reciprocal accounts?


A. 844,500 B. 574,500 C. 901,000 D. 596,500

44. Which of the following terms best describes the financial statements of a parent in which the investments are
accounted for on the basis of the direct equity interest under IAS 27?
A. Combined financial statements C. Consolidated financial statements
B. Single financial statements D. Separate financial statements

AFAR Page 7 of 12
Final Preboard Examination AFAR
45. Under IFRS 10, when there is a changes in a parent's ownership interest in a subsidiary that do not result in the
parent losing control, any difference between the amount by which the non-controlling interests are adjusted is
recognized
A. Within equity C. in profit or loss
B. As a component of OCI D. Not recognized

46. SMDC Construction Corporation contracted with the province of Pampanga to construct a bridge at a contract price
of P16,000,000. SMDC Corporation expects to earn P1,520,000 on the contract. The percentage of completion
method is to be used and the completion stage is to be determined by estimates made by the engineer. The
following schedule summarizes the activities of the contract for years 2014-2016.
Estimate Engineer’s
Cost to Estimate of Billings Collection
Year Incurred Complete Completion on Contract on Billings
2014 P4,600,000 P9,640,000 31% P5,000,000 P4,500,000*
2015 4,500,000 5,000,000 58% 6,000,000 5,400,000*
2016 5,250,000 -0- 100% 5,000,000 6,100,000
*A 10% retainer accounts for the difference between billings and collections.

Under the percentage of completion method, using the engineer’s estimate as the measure of completion to be applied
to revenues and costs, how much is the gross profit earned each year?
2014 2015 2016 2014 2015 2016
A. P545,600; P498,400; P606,000 C. P1,760,000; P6,400,000; P1,650,000
B. P545,600; P1,044,000;P1,044,000 D. P1,760,000; P1,800,000; P1,650,000

47. In which of the following case is consolidation of financial statements in accordance with IFRS 10 required?
A. An investment entity with more than one subsidiaries.
B. The ultimate parent of investment entities, which is an investment entity itself.
C. Investment entity where its subsidiary provides services that relate to the investment entity’s investment activities.
D. None of the foregoing

48. THE DPWH received capital outlay allotment and Notice of Cash Allocation, net of tax remittance advice, form the
DBM in the amount of P10,000,000 and P8,000,000. Respectively, Upon approval of the project proposal for the
improvement of the administration building, purchase order for materials was issued in the amount of P4,000,000.
At the date of delivery, construction materials were inspected and paid in full. Materials of P3,500,000 were issued
and used in the construction. Payroll for labor cost in the amount of P1,500,000 less 10% with holding tax was
submitted to the administration, and cash advance was granted to special disbursing office for the eventual
payment of payroll. Withholding tax payable was remitted to the Bureau of Internal Revenue and upon approval of
the accomplishment report by the DPWH, the capital expenditures were taken in the books of DPWH.
Which of the following journal entries in the Regular Agency Books of DPWH is correct?
A. To record receipt of the allotment for capital outlay
Allotment receivable from DBM 10,000,000
Subsidiary income from national government 10,000,000

B. To record obligation of materials purchased


Construction in materials inventory 4,000,000
Accounts payable 4,000,000

C. To record the receipt of notice of cash allocation, net tax remittance advice
Cash – MDS, Regular 8,000,000
Subsidy income from national government 8,000,000

D. To record payment of construction materials, net of 10% withholding tax


Accounts payable 4,000,000
Due to BIR 400,000
Cash – Collecting officer 3,600,000

49. In accordance with IFRS 10, the parent and subsidiaries are required to have the same reporting dates, or
consolidation based on additional financial information prepared by subsidiary, unless impracticable. The difference
between the date of the subsidiary's financial statements and that of the consolidated financial statements when
permissible shall be no more than
A. Twelve months C. Six months
B. Three months D. Nine months
AFAR Page 8 of 12
Final Preboard Examination AFAR
50. The STONERICH Construction Company was the lowest bidder on an office building construction contract. The
contract bid was P70 million, with an estimated cost to complete the project of P60 million. The contract period was
34 months starting January 2013. The company uses the cost-to-cost method of estimating earnings. Because of
changes requested by the customer, the contract price was adjusted downward to P65 million on January 1, 2014.

A record of construction activities for the years 2013-2016 follows: (in millions)
2013 2014 2015 2016
Actual cost-current year P 25 P 33 P 4.1 -
Progress billings 21 31 13 -
Cash receipts 18 30 10 P7
Estimated costs to complete 35 4 - -

Compute the gross profit (loss) realized in 2014.


A. P (1.360) million B. P 2.806 million C. 3.317 million D.4.167 million

51. When the functional currency is the Philippine pesos, translation gain or loss computed under IAS 21 shall be
included in
A. Profit or loss C. Within equity (Retained earnings)
B. OCI D. Share premium

52. The partnership of Anton, Joseph and William has elected to cease all operations and liquidate business property. A
balance sheet drawn up at this time shows the following account balances:

Cash P48,000; Non-Cash assets P177,000; Liabilities P35,000; Anton, Capital (60%) P101,000; Joseph, Capital (20%)
28,000; William, Capital P61,000

The following transactions occur in liquidating this business:


 Safe capital balances are immediately distributed to the partners. Liquidation expenses of P9,000 are
estimated as a basis for this computation.
 Noncash assets with a book value of P80,000 are sold for P48,000.
 All liabilities are paid.
 Safe capital balances are again distributed.
 Remaining noncash assets are sold for P44,000.
 Liquidation expenses of P7,000 are paid.
 Remaining cash is distributed to the partners and the financial records of the business permanently closed.

How much did Anton recover from this business liquidation?


A. P45,800 B. P40,400 C. P50,000 D. P41,600

53. In accordance with IAS 21, for translation of financial statements using current rate method, which of the following
would most likely be translated using historical rate?
A. Sales C. Share capital
B. Nonmonetary assets D. Ending retained earnings

54. Which of the following statements is correct?


A. In May 2016, St. Jude purchased medical supplies from South Drugstore at a cost of P40,000. However, South
Drugstore notifies St. Jude that the invoice was being canceled and the medical supplies were being donated to
the hospital. St. Jude should record this donation of medical supplies as patient service revenue.
B. A storm destroyed the receiving area of the building of AB Spiritual Center, a not-for-profit religious
organization. A member of the fellowship renovated the are no change. In AB’s statement of activities the
construction and renovation of the said area should be reported as an increase in both expenses and liabilities.
C. On March 23, 2016, Ms. FG, an alumnus of OP School, a private, not-for-profit high school, contributed
P100,000, with the stipulation that the donation be used for faculty development seminar during 2017. During
2017, OP spent all of the donation in accordance with Ms. FG’s wishes. For the year ended December 31, 2017,
restricted net assets will decrease
D. Family Care a private not-for-profit voluntary health and welfare organization, received the following
contribution in 2016. P60,000 from donors who stipulated that the money not be spent until 2017 and P38,000
from donor who stipulated that the contributions be used for the acquisition of equipment, none of which was
acquired in 2016. Both contributions are classified as permanently restricted net assets for the year ending
December 31, 2016.

55. When an economy experiences hyperinflation, financial statements shall be restated using
AFAR Page 9 of 12
Final Preboard Examination AFAR
A. IAS 26 C. IAS 29
B. IAS 21 D. IAS 27

56. Rhapsody Corp. manufactures rafts for use in swimming pools. The standard cost for material and labor is P892 per
raft. This includes 8 kilograms of direct material at a standard cost of P50 per kilogram, and 6 hours of direct labor at
P82 per hour. The following data pertain to November,
 Work in process inventory on November 1: none
 Work in process inventory on November 30: 800 units (75 percent complete as to labor: material is
issued at the beginning of processing).
 Units completed: 5,600 units
 Purchase of materials: 50,000 kilograms for P2,492,500
 Total actual labor costs: P3,007,600
 Actual hours of labor: 36,500 hours
 Direct-material quantity variance: P15,000 unfavorable
The entry to record direct labor cost charged to production must be:
A. Work in process inventory 3,007,600
Payroll 3,007,600
B. Work in process inventory 2,755,200
Labor cost variance 252,400
Payroll 3,007,600
C. Work in process inventory 3,050,400
Labor efficiency variance 13,000
Labor rate variance 55,900
Payroll 3,007,600
D. Work in process inventory 3,050,400
Labor rate variance 14,600
Labor efficiency variance 57,400
Payroll 3,007,600

57. What are transferred-in costs in a process costing system?


A. Costs of product of a previous internal process that is subsequently used in a succeeding internal process.
B. Supervisory salaries that are transferred from an overhead cost center to a production cost center.
C. Ending work-in-process inventory of a previous process that will be used in a succeeding process.
D. Labor costs incurred from transferring employees from another department within the same plant instead of
hiring temporary workers from the outside.

58. Rap Company is insolvent and its statement of affairs shows the following information:

Estimated gains on realization of assets – P1,440,000


Estimated losses on realization of assets – P2,000,000
Additional assets – P1,280,000
Additional liabilities – P960,000
Capital stock – P2,000,000
Deficit – P1,200,000

The pro-rate payment on the peso to stockholders (estimated amount to be recovered by stockholders) is:
A. P0.30 B. P0.43 C. P0.57 D. P0.70

59. An error was made in the computation of the percentage of completion of the current year’s ending work-in-process
(EWIP) inventory. The error resulted in assigning a lower percentage of completion to each component of the
inventory than actually was the case. Consequently, the following were misstated:
1) The computation of costs per equivalent unit.
2) Cost assigned to cost of goods completed for the period.
3) The computation of total equivalent units.
What were the effects of the error?
____1____ ____2____ ____3____
A. Understate Overstate Overstate
B. Understate Understate Overstate
C. Overstate Understate Understate
D. Overstate Overstate Understate

AFAR Page 10 of 12
Final Preboard Examination AFAR
60. Three joint operators are involved in a joint operation that manufactures ships chandlery. At the beginning of the
year the joint operation held P50,000 in cash. During the year the joint operation incurred the following expenses:
Wages paid P20,000, Overheads accrued P10,000. Additionally, creditors amounting to P40,000 were paid and the
joint operators contributed P15,000 cash each to the joint operation. The balance of cash held by the joint operation
at the end of the year is:
A. P5,000 B. P25,000 C. P35,000 D. P75,000

61. Unit cost of the good units will be affected by the spoilage when
A. The spoilage is considered normal C. The spoilage is charged to specific job
B. The spoilage is considered abnormal D. The spoilage is charged to all production

62. On December 12, 2018, Beshywap Company entered into three forward exchange contract to purchase 100,000 FC
(foreign currency) in 90 days. The relevant exchange rates are as follows:

Spot Rate Forward Rate (for


March 12, 2019)
November 30, 2018 P0.87 P 0.89
December 12, 2018 0.88 0.90
December 31, 2018 0.92 0.93

The company entered into the first forward contract to hedge a purchase of inventory in November 2018, payable in
March 2019. At December 31, 2018, what amount of foreign currency transaction gain from this forward contract
should be included in net income?
A. P0 B. P3,000 C. P5,000 D. P10,000

63. A job order cost system uses a predetermined factory overhead date based on expected volume and expected fixed
cost. At the end of the year, under-applied overhead might be explained by which of the following situations?
_Actual Fixed Costs __Actual Volume_
a. Greater than expected Greater than expected
b. Less than expected Greater than expected
c. Greater than expected Less than expected
d. Less than expected Less than expected

64. Beshywap entered into as second forward contract to hedge a commitment to purchase equipment being
manufactured to Beshywap’s specifications. The expected delivery date is March 2019 at which time settlement is
due to the manufacturer. The hedge qualifies as a fair value hedge. At December 31, 2018, what amount of foreign
currency transaction gain from this forward contract should Beshywap include in net income?
A. P0 B. P3,000 C. P5,000 D. P10,000

65. As provided in the constitution, no money shall be paid out of the National Treasury, except in pursuance of
A. Appropriation C. Budget
B. Executive order D. General fund

66. Beshywap entered into the third forward contract for speculation. At December 31, 2018, what amount of foreign
currency transaction gain from the forward contract should be included in the net income?
A. P0 B. P3,000 C. P5,000 D. P10,000

67. Which government body is responsible for the adoption and promulgation of public sector accounting standards?
A. Commission on Audit C. Department of Budget and Management
B. Bureau of Treasury D. Department of Finance

68. A subsidiary of Salisbury, Inc. located in a foreign country whose functional currency is the foreign currency (or the
local currency). The subsidiary acquires inventory on credit on November 1, 2019, for P100,000 foreign currencies
(FC) that is sold on January 19, 2020 for 130,000 FC. The subsidiary pays for the inventory on January 31, 2020.
Currency exchange rates for 1 FC are as follows:
November 1, 2019 – P0.16
December 31, 2019 – 0.17
January 17, 2020 – 0.18
January 31, 2020 – 0.19
Average for 2020 – 0.20
What amount does Salisbury’s consolidated balance sheet report for this inventory at December 31, 2019?
A. P16,000 B. P17,000 C. P18,000 D. P19,000
AFAR Page 11 of 12
Final Preboard Examination AFAR
69. Unconditional promises to give that include promises of payments due in future periods (next year or later) are
reported as
A. deferred revenues until payment is received.
B. a memorandum, until the year of the promised payment.
C. restricted revenues.
D. unrestricted revenues.

70. Napro Charities, a not-for-profit agency, receives free electricity on a continuous basis from a local utility company.
The utility company’s contribution is made subject to cancellation by the donor. Napro Charities should account for
this contribution as a(n)
A. Unrestricted revenue only.
B. Restricted revenue only.
C. Restricted revenue and an expense.
D. Unrestricted revenue and an expense.

“The LORD detests lying lips, but he delights in people who are trustworthy.” Proverbs 12:22

“With integrity, you have nothing to fear, since you have nothing to hide. With integrity, you will do the right
thing, so you will have no guilt.” Zig Ziglar

Good luck and God Bless!

AFAR Page 12 of 12
BALIUAG UNIVERSITY
CPA Review Program
Advanced Financial Accounting and Reporting
Final Preboard Examination

Instructions: Select the best answer for each of the following questions. Mark only one answer for each item on the
answer sheets provided. Strictly NO ERASURES ALLOWED.

1. Comparative trial balance of the home office of ACE Corporation and its two branches at December 31, 2016 were as
follows:

Home Office CHI Branch MIN Branch


Cash P 5,000 P15,000 P22,000
Accounts receivable 80,000 30,000 40,000
Inventories 150,000 60,000 48,000
CHI Branch 170,000
MIN Branch 165,000
Plant assets 730,000 250,000 200,000
Purchases 900,000
Shipments from home office 300,000 240,000
Expenses 300,000 75,000 50,000
Total P2,500,000 P730,000 P600,000

Accounts payable P100,000 P45,000 P30,000


Other liabilities 80,000 15,000 5,000
Loading in branch inventories 108,000
Share capital, P10 par 500,000
Retained earnings 262,000
Home office 170,000 165,000
Sales 1,000,000 500,000 400,000
Shipment branches 450,000
Totals P2,500,000 P730,000 P600,000

Additional information:
Home office and branch inventories at December 31, 2016 were:
Home office P120,000
CHI Branch (at billed prices) P 72,000
MIN Branch (at billed prices) 96,000
How much net income will ACE Corporation report for year-ended 2016?
A. P260,000
B. P122,000
C. P220,000
D. P595,000

2. Which of the following is not considered a legitimate expense of a partnership?


A. Interest paid to partners based on the amount of invested capital.
B. Depreciation on assets contributed to the partnership by partners.
C. Salaries for management hired to run the partnership.
D. Supplies used in the partners’ offices.

3. The estimated unit costs for Lodge Inc., when it is operating at a production and sales level of 12,000 units, are as
follows:

Estimated
Cost Item Unit Cost
Direct materials P 32
Direct labor 10
Variable factory overhead 15
Fixed factory overhead 6
Variable marketing 3
Fixed marketing 5
AFAR Page 1 of 12
Final Preboard Examination AFAR

Compute the total cost that would be incurred during a month with a production level of 11,500 units and a sales level
of 9,500 units.

A. P800,500 B. P813,000 C. P816,000 D. P852,000

4. If the partnership does not specify how income is to be allocated, profit and losses should be allocated
A. Equally.
B. In accordance with partners’ capital contribution.
C. In proportion to the weighted average of capital invested during the period.
D. Equitably so that partners are compensated for the time and effort expended on behalf of the partnership.

5. Riverdale Company incurred P80,000 direct labor cost in 2014 and had the following selected account balances at
the beginning and end of 2017: Finished goods January 1, P56,000; Work in process January 1, P24,000; Materials
January 1, P34,000; Finished goods December 31, P90,000; Work I process December 31, P28,000; Materials
December 31,0P48,000. The total cost of goods sold and actual factory overhead during the year are P280,000 and
P70,000 respectively.

Determine the total material purchases during the year.


A. P182,000 B. P162,000 C. P148,000 D. P144,000

6. When the retiring partner’s interest is greater than the settlement price, using bonus method, the difference is
A. Debited to the remaining partners’ capital accounts
B. Credited to the retiring partner’s capital account
C. Credited to the remaining partner’s capital accounts
D. Recognized in the profit or loss for the period.

7. The following data summarizes in part the results of operations for 2017 of Veronica Company.
Of the total cost of goods manufactured for 2017, 38% was for materials used, 30% for direct labor, and 32% for
manufacturing overhead.
During 2017, the company paid for 90% of the materials purchased, leaving P293,000 of unpaid invoices for
materials at year end. The company commenced 2017 operations with a materials inventory of P421,000. All
materials were purchased f.o.b. company’s plant.
The company disbursed P2,101,500 for direct labor during 2011. As of Dec. 31, 2017, the accrued liability for
direct amounted to P144,000, which was twice as much as last year’s accrual.
The inventory of finished goods on December 31, 2017, was 10% of the cost of the units fifnished during the year,
and goods in process on that date were one-half the finished goods inventory. This year’s finished goods
inventory was 150% of last year’s. there are no goods in process last year.
The manufacturing overhead, except for depreciation of factory buildings and equipment, is detailed below:
Indirect labor P 672,000
Heat, light, and power 226,200
Maintenance and repairs 488,300
Insurance – factory 18,100
Property taxes 64,400
Factory payroll taxes 97,000
Miscellaneous factory costs 276,400
P 1,842,400
How much is the cost of sales during 2017?
A. P 7,245,000 B. P 6,900,000 C. P 6,670,000 D. P5,757,500

8. An enterprise uses a branch accounting system in which it establishes separate formal accounting systems for its home
office operations and its branch office operations. Which of the following statements about this arrangement is false?
A. The home office account on the books of a branch office represents the equity interest of the home office in the net
assets of the branch.
B. The home office and branch office accounts are reciprocal accounts that must be eliminated in the preparation of the
enterprise’s financial statements that are presented in accordance with GAAP.
C. Unrealized profit from internal transfers between the home office and a branch must be eliminated in the
preparation of the enterprise’s financial statements that are presented in accordance with GAAP.
D. The branch office account on the books of the home office represents the equity interest of the branch office in the
net assets of the home office.

AFAR Page 2 of 12
9. During 2018, Jughead Company purchased materials costing P152,600. Materials requisitioned for jobs cost P98,000,
and indirect materials costing P42,000 were charged to Factory Overhead. Factory payrolls were P212,000 with
payroll taxes deducted of P60,000. Indirect labor of P71,000 included in the payrolls was charged to factory
Overhead. All other labor was direct labor charged to the jobs. Factory overhead was applied to the jobs at the rate
of P8 per machine hour. During the year, the company operated at 45,000 machine hours and incurred factory
overhead costs of P259,000 (in addition to the indirect materials and indirect labor previously stated). Depreciation
of P47,000 was included in the P259,000 of factory overhead costs.

Product costing P465,000 were completed during the year, and the cost of goods sold was P480,000. At the
beginning of the year, Jughead had the following balances:
Materials P27,000; Work in Process P48,000; Finished Goods P34,000
Determine the cost of work in process ending inventory for 2018.
A. P134,000 B. P182,000 C. P167,000 D. P194,000

10. Allocated expenses from the home office will be recorded by the home office as debit to
A. Investment in Branch C. Home Office
B. Expense account D. Cash

Items 11 and 12 are based on the following:


Benny, Jenny, and Kenny are partners in a trading business. They participate in the profits and losses equally. As of
December 31, 2016, the partners’ capital and drawing accounts are as follows:
Benny Jenny Kenny Total
Capital P20,000 P16,000 P600,000 P96,000
Drawing 12,000 8,000 4,000 24,000

The partners decided to liquidate the partnership. The operating profit for the year 2016 amounted to P14,400, which
was all exhausted including partnership assets. As of December 31, 2016, unpaid liabilities still amounted to P16,800.
Benny is personally insolvent, but both Jenny and Kenny have substantial private resources.
11. The total loss on realization was:
A. P72,000 B. P88,800 C. P96,000 D. P103,200
12. The amount received by Kenny in final cash distribution was:
A. P15,600 B. P16,800 C. P21,600 D. P32,400

13. The face amount of a loans payable is greater than the net realizable value of the attached inventory lien. The
inventory shall be classified under
A. Partially secured creditors C. Assets pledged with partially secured creditors
B. Fully secured creditors D. Assets pledged with fully secured creditors
14. It is equals to the total assets under receivership minus total liabilities to be liquidated when the latter is greater
than the assets.
A. Estate equity C. Capital deficiency
B. Estate deficit D. Receivership net loss

15. Partners D, I, L, and G, share profits 40%, 30%, 15%, and 15%, respectively. Their partnership agreement provides
that in the event of the death of a partner, the firm shall continue until the end of the fiscal period. Profits shall be
considered to have been earned proportionately during this period, and the deceased partners’ capital shall be
adjusted by the proper share of the profit or loss until the date of death. From that date until the date of settlement
with the estate there shall be added interest at 6% computed on the adjusted capital.

The remaining partners shall continue to share profits in the old ratio. Payment to the estate shall be made within one
year from the date of the partner’s death. Partner G died on November 16. On December 31, the end of the six- month
period, account balances on the partnership books before the income summary account is closed are as follows:
Cash P 7,500 Notes payable P15,000
Accounts receivable 70,000 Accounts payable 70,500
Inventories 95,000 D, capital 42,000
Machinery and equipment, net 45,000 I, capital 37,500
Store furniture and fixtures, net 16,500 L, capital 24,000
G, capital 22,500
Income summary
(7/1-12/31) 22,500
P 234,000 P 234,000
The income summary account is closed on December 31. On this date, L decides to retire. D and I agree to pay the
balance in L’s capital account after distributions of profit, less 20%, and issue a partnership 60-day, 6% note to L in
settlement.
What amount is due to G’s estate on December 31?
A. P22,500 B. P25,875 C. P25,062.50 D. P25,218.99

16. Loss absorption potential of a partner computed during the preparation of cash priority program is equals to
A. {Partner’s capital account plus partner’s loan (receivable)} multiplied by the respective PL ratio.
B. {Partner’s capital account minus partner’s loan (receivable)} divided by the respective PL ratio.
C. {Partner’s capital account plus partner’s loan (payable)} multiplied by the respective PL ratio.
D. {Partner’s capital account minus partner’s loan (payable)} divided by the respective PL ratio.

17. On December 1, 2017, Coach Corp. entered into a 120-day forward contract to purchase 250,000US dollars for
speculative purposes, Coach Corp. fiscal year ends on December 31. The exchange rates are as follows:
Date Spot rate Forward rate (3/31/10)
December 1, 2017 P45.00 P45.50
December 31, 2017 46.00 46.50
January 30, 2018 45.60 45.30
March 31, 2018 45.10
How much is the forex gain or loss to be reported from this forward contract in 2018?
A. P250,000 B. P350,000 C. P300,000 D. P225,000

18. The third step in the revenue recognition model framework under IFRS 15 is
A. Determine the transaction price
B. Recognize revenue when (or as) the entity satisfies a performance obligation
C. Allocate the transaction price to the performance obligations in the contract
D. Identify the performance obligations in the contract

19. MELROSE Corporation issued a promissory note denominated in foreign currency for the purchase made from an
Italian supplier. The following were the related transactions: (in Italy Lire). On December 1, MELROSE Corporation
purchased merchandise from an Italian supplier for 60-day 18% promissory note for 108,000 Italy lire, at a selling
rate 1FC to P74.20. On December 31, the selling spot rate is 1FC to P74.85. On January 30, the selling spot rate is 1FC
to P75.75

On the settlement date, how much is the foreign exchange gain/loss?


A. P98,658 loss B. P172,422 gain C. P100,116 loss D.P172,422 loss

20. SAVEMORE Corp. entered into a forward contract to hedge a sale of inventory in October 26, 2017 to be collected
on January 24, 2017. 72,000 FC (foreign currency) in 90 days. The relevant exchange rate as follows:
Spot rate Forward rate (1/24/18)
October 26, 2017 P52.73 P52.77
December 31, 2017 52.82 52.89
January 24, 2018 52.94
What is the net forex gain (loss) from this transaction and hedge that will be reported on SAVEMORE’s 2017
statement of income?
A. P15,120 B. (P8,640) C. P6, 480 D. (P2,160)

21. The Construction-in-Process account accumulates the following when the percentage-of-completion method is used
A. Construction costs to date. C. Construction costs plus gross profit earned to date.
B. Construction costs to date less billings to date. D. Construction costs to date less payments received

22. SBX Restaurant sold a fast food restaurant franchise to NKE Corporation. The sale agreement signed on January 2,
2016 called for a P70,000 down payment plus two P15,000 annual payments representing the value of initial
franchise services rendered by SBX. The present value factor of two annual payments appropriately discounted at
10% is 1 7355. In addition, the agreement required the franchise to pay 5% of its gross revenues to the franchisor;
this was deemed sufficient to cover the cost and provide a reasonable profit margin on continuing franchise services
to be performed by SBX. The restaurant opened early in 2016 and its sales for the year amounted to P600,000.
SBX Restaurant’s 2016 total revenue from the franchise will be:
A. P 0 B. P 30,000 C. P56,033 D. P 126,000

23. Under installment sales method,


A. Revenue, costs and gross profit are recognized proportionately to the cash that is received from the sale of the
product.
B. Gross profit is not recognized until the amount of cash received exceeds the cost of the item sold.
C. Gross profit is deferred proportionately to cash uncollected from sale of the product, but total revenue and costs are
recognized at point of sale.
D. Revenues and costs are recognized proportionately to the cash received from the sale of the product, but gross profit
is deferred until all cash is received.

24. On January 1, 2015, Mrs. Fields entered into a franchise agreement with KK to market their products. The agreement
provides for an initial fee of P15,500,000 payable as follows: P6,500,000 to be paid upon signing of the contract and
the balances in five equal annual payments every end of the year starting December 31, 2015. Mrs. Fields signs a
non-interest bearing notes for the balance. His credit rating indicates that he can borrow money at 15% interest for a
loan of this type. The present value of an annuity of P1 at 15% for 5 periods is 3.352. The agreement further provides
that the franchise must pay a continuing franchise fee equal to 3% of the monthly gross sales. On August 31, the
franchiser completed the initial services required in the contract at a cost of P7,290,120 and incurred indirect cost of
P475,000. The franchisee commenced business operations on November 30, 2015. The gross sales reported to the
franchisor were P1,800,000 for December, 2015. The first installment payment was made in due date.
Assume the collectability of the note is not reasonably assured the net income for the year ended, December 31, 2015
is
A. P4,121,288 B.P5,026,268 C. P5,243,480 D. P4,551,268

25. According to IFRS 15, where a contract has multiple performance obligations, an entity will allocate the transaction
price to the performance obligations in the contract by reference to their relative
A. Net realizable values C. Fair market values
B. Stand-alone selling prices D. Fair value less cost of disposal

Love Corporation has been using the cash method to account for income since its first year of operations in 2016. All
sales are made on credit with notes receivable given by the customers. The income statements for 2016 and 2017
included the following amounts:
2016 2017
Revenues – collection on principal P96,000 P150,000
Revenues – interest 10,800 16,500
Cost of goods purchased* 151,800 156,060
*Includes increase in inventory of goods on hand of P6,000 in 2016 and P24,000 in 2017
The balance due on the notes at the end of each year were as follows:

2016 2017
Notes receivable – 2016 P186,000 P108,000
Notes receivable – 2017 180,000
Unearned interest revenue – 2016 21,501 16,737
Unearned interest revenue – 2017 24,129

26. Under the installment method, how much is the realized gross profit in 2016?
A. P48,240 B. P96,000 C. P53,667 D. P10,10632
27. Under the installment method, how much is the realized gross profit in 2017?
A. P36,801 B. P33,186 C. P69,987 D. P64,845

28. An entity may use residual approach per IFRS 15 when allocating transaction price under which circumstance(s)?
i. The entity sells the same good or service to different customers (at or near the same time) for a broad range of
amounts
ii. The entity has not yet established a price for that good or service and the good or service has previously been sold
on a stand-alone basis.
A. Both i and ii C. ii only
B. Neither i nor ii D. i only

29. With regard to the definition of revenue given by IFRS15, which of the following statements is true?
A. Revenue includes cash received from share issues
B. Revenue arises from ordinary activities only
C. Revenue includes cash received from borrowings
D. Revenue may arise from either ordinary activities or extraordinary activities
Items 30 and 31 are based on the following:
Belgium Corporation owns 80 percent of the stock of Hillinger Company. At the end of 2017, Belgium Corp. and
Hillinger Company reported the following partial operating results and inventory balances:
Total sales Belgium Corp. Hillinger Co.
Sales to Hillinger Company P 660,000 P 510,000
Sales to Belgium Corporation 140,000
Net income 20,000
Operating income (excluding income from
Hillinger Company) 70,000
Inventory on hand, December 31, 2017
purchased from Hillinger Company 48,000
purchased from Belgium Corporation 42,000

Belgium Corporation regularly prices its products at cost plus a 40 percent markup for profit. Hillinger Company prices its
sales at cost plus a 20 percent markup. The total sales reported by Belgium and Hillinger include both intercompany sales
and sales to nonaffiliates.

30. The consolidated cost of sales for 2017 must be:


A. P596,428 B. P616,428 C. P576,428 D. P536,428

31. The controlling interest net income for 2017 must be:
A. P67,600 B. P70,000 C. P90,000 D. P86,000

32. Defined by IFRS 11 as a joint arrangement whereby the parties that have joint control of the arrangement have rights to
the assets, and obligations for the liabilities, relating to the arrangement.
A. Joint operation C. Jointly controlled asset
B. Jointly controlled entity D. Joint venture
33. According to IFRS 11, a party that participates in, but does not have joint control of, a joint venture shall account for
the investment (assume nominal interest) using
A. IFRS 11 C. IFRS 9
B. IAS 28 D. IAS 7

34. Clydes Bakery owns 60 percent of the stock of Good Shepherd Products acquired several years ago at book values.
On January 1, 2017, inventory reported by Clydes Bakery included 20,000 bags of flour purchased from Good
Shepherd Products at P90 per bag. By December 31, 2017, all these beginning inventory purchased from Good
Shepherd Products had been baked into products and sold to customers by Clydes Bakery. There were no
transactions between Clydes Bakery and Good Shepherd Products during 2017.

Both Clydes Bakery and Good Shepherd Products price their sales at cost plus 50% mark-up for profit. Clydes Bakey
reported income from its baking operations of P3,000,000 and Good Shepherd Products reported net income of
P2,500,000 for 2017.
Compute the consolidated net income attributable to controlling interest.
A. P3,420,000 B. P4,860,000 C. P4,500,000 D. P5,580,000

35. Under IFRS for SMEs, transaction cost during the initial measurement of an investment in a jointly controlled entity
shall be added when using
A. Cost model only C. Equity model and fair value model
B. Fair value model only D. Cost model and equity model

36. Viewsonic Manufacturing purchased 80 percent of the stock of Ronnie Mines, Inc., in 2015. In preparing the
consolidated financial statements at the end of 2017, the controller of Viewsonic discovered that Viewsonic
manufacturing had purchased P750,000 of raw materials from Ronnie Mines during the year and that the parent
company had not paid for the last purchase of P120,000. All the inventory purchased was still on hand at year-end.
Ronnie Mine has spent P500,000 in producing the items sold to Viewsonic Manufacturing.
What effect, if any, will failure to eliminate or adjust for these items have on total current assets reported in the
consolidated balance sheet on December 31, 2017?
A. overstated by P250,000 C. overstated P370,000
B. overstated by P870,000 D. overstated P750,000

37. Asset acquisition as business combination will most likely result in


A. Acquirer recording assets and liabilities of acquiree at fair value
B. Acquiree continuing its existence.
C. Acquirer recording the acquisition in an investment account
D. Acquirer purchasing shares of stocks of investee

Motul Co., a manufacturing company used process costing under the FIFO method for its products. The products
underwent two departments namely Assembly then Finishing. The following data were extracted from the system in the
Finishing department: Beginning inventory units 10,000 which were 70% incomplete. Ending inventory units 5,000 which
were 35% converted, transferred-out units from the Assembly department 80,000 and spoiled units of 2,800 of which
2,500 were considered normal. In the Finishing department, materials were added at the end of the process.

The following were the costs in the Finishing department: Beginning inventory costs of Transferred-In, Direct Materials,
Conversion respectively were P150,000, P80,000, P76,000. Current costs of Direct Materials and Conversion were
P742,500 and P650,000 respectively. Transferred-out cost from the Assembly department amounted to P1,162,500.

38. What are the equivalent units of production as to direct materials?


A. 85,000 B. 82,200 C. 80,300 D. 82,500
39. What is the cost of completed goods?
A. 2,630,400 B. 2,762,400 C. 2,672,400 D. 2,720,000

40. Under IFRS 3, which of the following falls under the third step in the acquisition method of business combination?
A. Determination of the acquisition date
B. Recognition and measurement of non-controlling interest
C. Recognition and measurement of goodwill or a gain from a bargain purchase.
D. Identification of the acquirer
41. For contingent consideration under IFRS 3, changes in fair value are recognized in profit or loss for contingent
consideration classified as
A. Equity and asset/liability C. Asset/liability only
B. Equity only D. None, changes in fair value is recognized in OCI

42. Miles Company established a branch in Ayala by sending merchandise costing P924,500 and effecting a fund transfer
of P400,000 cash on January 1, 2016. The branch purchased computer equipment costing P420,000 on April 1. As
per agreement, the home office will maintain all the property, plant and equipment records.

Ayala branch collected P56,000 worth of Ortigas branch’s receivable on August 4. Cash remittance to the home was
P250,000 on September 28.

On November 21, Ayala branch returned defective merchandise worth P125,000 to the home office.

At the end of the year, the company’s controller found out that the branch accountant had failed to record all the
transactions initiated by the home office from the second half of the year. Because of this, there is a significant
discrepancy between the balances of the reciprocal accounts.

For the purpose of reconciling the reciprocal accounts, the controller instructed the accounting staff of the home office
to send a copy of the Investment in Ayala general ledger to the branch.

Investment in Ayala
1/1 Merchandise to branch P924,500 4/2 Equipment acquisition P240,000
1/1 Fund transfer 400,000 9/30 Remittance 225,000
7/2 Merchandise to branch 135,000 11/22 Return of goods from branch 12,500
8/31 Fund transfer 95,000

What is the unadjusted balance of the Home Office Account?


A. 598,500 B. 585,500 C. 723,500 D. 335,500

43. What is the adjusted balance of the reciprocal accounts?


A. 844,500 B. 574,500 C. 901,000 D. 596,500

44. Which of the following terms best describes the financial statements of a parent in which the investments are
accounted for on the basis of the direct equity interest under IAS 27?
A. Combined financial statements C. Consolidated financial statements
B. Single financial statements D. Separate financial statements
45. Under IFRS 10, when there is a changes in a parent's ownership interest in a subsidiary that do not result in the
parent losing control, any difference between the amount by which the non-controlling interests are adjusted is
recognized
A. Within equity C. in profit or loss
B. As a component of OCI D. Not recognized

46. SMDC Construction Corporation contracted with the province of Pampanga to construct a bridge at a contract price
of P16,000,000. SMDC Corporation expects to earn P1,520,000 on the contract. The percentage of completion
method is to be used and the completion stage is to be determined by estimates made by the engineer. The
following schedule summarizes the activities of the contract for years 2014-2016.
Estimate Engineer’s
Cost to Estimate of Billings Collection
Year Incurred Complete Completion on Contract on Billings
2014 P4,600,000 P9,640,000 31% P5,000,000 P4,500,000*
2015 4,500,000 5,000,000 58% 6,000,000 5,400,000*
2016 5,250,000 -0- 100% 5,000,000 6,100,000
*A 10% retainer accounts for the difference between billings and collections.

Under the percentage of completion method, using the engineer’s estimate as the measure of completion to be applied
to revenues and costs, how much is the gross profit earned each year?
2014 2015 2016 2014 2015 2016
A. P545,600; P498,400; P606,000 C. P1,760,000; P6,400,000; P1,650,000
B. P545,600; P1,044,000;P1,044,000 D. P1,760,000; P1,800,000; P1,650,000

47. In which of the following case is consolidation of financial statements in accordance with IFRS 10 required?
A. An investment entity with more than one subsidiaries.
B. The ultimate parent of investment entities, which is an investment entity itself.
C. Investment entity where its subsidiary provides services that relate to the investment entity’s investment activities.
D. None of the foregoing

48. THE DPWH received capital outlay allotment and Notice of Cash Allocation, net of tax remittance advice, form the
DBM in the amount of P10,000,000 and P8,000,000. Respectively, Upon approval of the project proposal for the
improvement of the administration building, purchase order for materials was issued in the amount of P4,000,000.
At the date of delivery, construction materials were inspected and paid in full. Materials of P3,500,000 were issued
and used in the construction. Payroll for labor cost in the amount of P1,500,000 less 10% with holding tax was
submitted to the administration, and cash advance was granted to special disbursing office for the eventual
payment of payroll. Withholding tax payable was remitted to the Bureau of Internal Revenue and upon approval of
the accomplishment report by the DPWH, the capital expenditures were taken in the books of DPWH.
Which of the following journal entries in the Regular Agency Books of DPWH is correct?
A. To record receipt of the allotment for capital outlay
Allotment receivable from DBM 10,000,000
Subsidiary income from national government 10,000,000

B. To record obligation of materials purchased


Construction in materials inventory 4,000,000
Accounts payable 4,000,000

C. To record the receipt of notice of cash allocation, net tax remittance advice
Cash – MDS, Regular 8,000,000
Subsidy income from national government 8,000,000

D. To record payment of construction materials, net of 10% withholding tax


Accounts payable 4,000,000
Due to BIR 400,000
Cash – Collecting officer 3,600,000

49. In accordance with IFRS 10, the parent and subsidiaries are required to have the same reporting dates, or
consolidation based on additional financial information prepared by subsidiary, unless impracticable. T he difference
between the date of the subsidiary's financial statements and that of the consolidated financial statements when
permissible shall be no more than
A. Twelve months C. Six months
B. Three months D. Nine months
50. The STONERICH Construction Company was the lowest bidder on an office building construction contract. The
contract bid was P70 million, with an estimated cost to complete the project of P60 million. The contract period was
34 months starting January 2013. The company uses the cost-to-cost method of estimating earnings. Because of
changes requested by the customer, the contract price was adjusted downward to P65 million on January 1, 2014.

A record of construction activities for the years 2013-2016 follows: (in millions)
2013 2014 2015 2016
Actual cost-current year P 25 P 33 P 4.1 -
Progress billings 21 31 13 -
Cash receipts 18 30 10 P7
Estimated costs to complete 35 4 - -

Compute the gross profit (loss) realized in 2014.


A. P (1.360) million B. P 2.806 million C. 3.317 million D.4.167 million

51. When the functional currency is the Philippine pesos, translation gain or loss computed under IAS 21 shall be
included in
A. Profit or loss C. Within equity (Retained earnings)
B. OCI D. Share premium

52. The partnership of Anton, Joseph and William has elected to cease all operations and liquidate business property. A
balance sheet drawn up at this time shows the following account balances:

Cash P48,000; Non-Cash assets P177,000; Liabilities P35,000; Anton, Capital (60%) P101,000; Joseph, Capital (20%)
28,000; William, Capital P61,000

The following transactions occur in liquidating this business:


 Safe capital balances are immediately distributed to the partners. Liquidation expenses of P9,000 are
estimated as a basis for this computation.
 Noncash assets with a book value of P80,000 are sold for P48,000.
 All liabilities are paid.
 Safe capital balances are again distributed.
 Remaining noncash assets are sold for P44,000.
 Liquidation expenses of P7,000 are paid.
 Remaining cash is distributed to the partners and the financial records of the business permanently closed.

How much did Anton recover from this business liquidation?


A. P45,800 B. P40,400 C. P50,000 D. P41,600

53. In accordance with IAS 21, for translation of financial statements using current rate method, which of the following
would most likely be translated using historical rate?
A. Sales C. Share capital
B. Nonmonetary assets D. Ending retained earnings

54. Which of the following statements is correct?


A. In May 2016, St. Jude purchased medical supplies from South Drugstore at a cost of P40,000. However, South
Drugstore notifies St. Jude that the invoice was being canceled and the medical supplies were being donated to
the hospital. St. Jude should record this donation of medical supplies as patient service revenue.
B. A storm destroyed the receiving area of the building of AB Spiritual Center, a not-for-profit religious
organization. A member of the fellowship renovated the are no change. In AB’s statement of activities the
construction and renovation of the said area should be reported as an increase in both expenses and liabilities.
C. On March 23, 2016, Ms. FG, an alumnus of OP School, a private, not-for-profit high school, contributed
P100,000, with the stipulation that the donation be used for faculty development seminar during 2017. During
2017, OP spent all of the donation in accordance with Ms. FG’s wishes. For the year ended December 31, 2017,
restricted net assets will decrease
D. Family Care a private not-for-profit voluntary health and welfare organization, received the following
contribution in 2016. P60,000 from donors who stipulated that the money not be spent until 2017 and P38,000
from donor who stipulated that the contributions be used for the acquisition of equipment, none of which was
acquired in 2016. Both contributions are classified as permanently restricted net assets for the year ending
December 31, 2016.

55. When an economy experiences hyperinflation, financial statements shall be restated using
A. IAS 26 C. IAS 29
B. IAS 21 D. IAS 27

56. Rhapsody Corp. manufactures rafts for use in swimming pools. The standard cost for material and labor is P892 per
raft. This includes 8 kilograms of direct material at a standard cost of P50 per kilogram, and 6 hours of direct labor at
P82 per hour. The following data pertain to November,
 Work in process inventory on November 1: none
 Work in process inventory on November 30: 800 units (75 percent complete as to labor: material is
issued at the beginning of processing).
 Units completed: 5,600 units
 Purchase of materials: 50,000 kilograms for P2,492,500
 Total actual labor costs: P3,007,600
 Actual hours of labor: 36,500 hours
 Direct-material quantity variance: P15,000 unfavorable
The entry to record direct labor cost charged to production must be:
A. Work in process inventory 3,007,600
Payroll 3,007,600
B. Work in process inventory 2,755,200
Labor cost variance 252,400
Payroll 3,007,600
C. Work in process inventory 3,050,400
Labor efficiency variance 13,000
Labor rate variance 55,900
Payroll 3,007,600
D. Work in process inventory 3,050,400
Labor rate variance 14,600
Labor efficiency variance 57,400
Payroll 3,007,600

57. What are transferred-in costs in a process costing system?


A. Costs of product of a previous internal process that is subsequently used in a succeeding internal process.
B. Supervisory salaries that are transferred from an overhead cost center to a production cost center.
C. Ending work-in-process inventory of a previous process that will be used in a succeeding process.
D. Labor costs incurred from transferring employees from another department within the same plant instead of
hiring temporary workers from the outside.

58. Rap Company is insolvent and its statement of affairs shows the following information:

Estimated gains on realization of assets – P1,440,000


Estimated losses on realization of assets – P2,000,000
Additional assets – P1,280,000
Additional liabilities – P960,000
Capital stock – P2,000,000 Deficit –
P1,200,000

The pro-rate payment on the peso to stockholders (estimated amount to be recovered by stockholders) is: A.
P0.30 B. P0.43 C. P0.57 D. P0.70

59. An error was made in the computation of the percentage of completion of the current year’s ending work-in-process
(EWIP) inventory. The error resulted in assigning a lower percentage of completion to each component of the
inventory than actually was the case. Consequently, the following were misstated:
1) The computation of costs per equivalent unit.
2) Cost assigned to cost of goods completed for the period.
3) The computation of total equivalent units.
What were the effects of the error?
1 2 3
A. Understate Overstate Overstate
B. Understate Understate Overstate
C. Overstate Understate Understate
D. Overstate Overstate Understate
60. Three joint operators are involved in a joint operation that manufactures ships chandlery. At the beginning of the
year the joint operation held P50,000 in cash. During the year the joint operation incurred the following expenses:
Wages paid P20,000, Overheads accrued P10,000. Additionally, creditors amounting to P40,000 were paid and the
joint operators contributed P15,000 cash each to the joint operation. The balance of cash held by the joint operation
at the end of the year is:
A. P5,000 B. P25,000 C. P35,000 D. P75,000

61. Unit cost of the good units will be affected by the spoilage when
A. The spoilage is considered normal C. The spoilage is charged to specific job
B. The spoilage is considered abnormal D. The spoilage is charged to all production

62. On December 12, 2018, Beshywap Company entered into three forward exchange contract to purchase 100,000 FC
(foreign currency) in 90 days. The relevant exchange rates are as follows:

Spot Rate Forward Rate (for


March 12, 2019)
November 30, 2018 P0.87 P 0.89
December 12, 2018 0.88 0.90
December 31, 2018 0.92 0.93

The company entered into the first forward contract to hedge a purchase of inventory in November 2018, payable in
March 2019. At December 31, 2018, what amount of foreign currency transaction gain from this forward contract should
be included in net income?
A. P0 B. P3,000 C. P5,000 D. P10,000

63. A job order cost system uses a predetermined factory overhead date based on expected volume and expected fixed
cost. At the end of the year, under-applied overhead might be explained by which of the following situations?
_Actual Fixed Costs Actual Volume_
a. Greater than expected Greater than expected
b. Less than expected Greater than expected
c. Greater than expected Less than expected
d. Less than expected Less than expected

64. Beshywap entered into as second forward contract to hedge a commitment to purchase equipment being
manufactured to Beshywap’s specifications. The expected delivery date is March 2019 at which time settlement is
due to the manufacturer. The hedge qualifies as a fair value hedge. At December 31, 2018, what amount of foreign
currency transaction gain from this forward contract should Beshywap include in net income?
A. P0 B. P3,000 C. P5,000 D. P10,000

65. As provided in the constitution, no money shall be paid out of the National Treasury, except in pursuance of
A. Appropriation C. Budget
B. Executive order D. General fund

66. Beshywap entered into the third forward contract for speculation. At December 31, 2018, what amount of foreign
currency transaction gain from the forward contract should be included in the net income?
A. P0 B. P3,000 C. P5,000 D. P10,000

67. Which government body is responsible for the adoption and promulgation of public sector accounting standards?
A. Commission on Audit C. Department of Budget and Management
B. Bureau of Treasury D. Department of Finance

68. A subsidiary of Salisbury, Inc. located in a foreign country whose functional currency is the foreign currency (or the
local currency). The subsidiary acquires inventory on credit on November 1, 2019, for P100,000 foreign currencies
(FC) that is sold on January 19, 2020 for 130,000 FC. The subsidiary pays for the inventory on January 31, 2020.
Currency exchange rates for 1 FC are as follows:
November 1, 2019 – P0.16
December 31, 2019 – 0.17
January 17, 2020 – 0.18
January 31, 2020 – 0.19
Average for 2020 – 0.20
What amount does Salisbury’s consolidated balance sheet report for this inventory at December 31, 2019? A.
P16,000 B. P17,000 C. P18,000 D. P19,000
69. Unconditional promises to give that include promises of payments due in future periods (next year or later) are
reported as
A. deferred revenues until payment is received.
B. a memorandum, until the year of the promised payment.
C. restricted revenues.
D. unrestricted revenues.

70. Napro Charities, a not-for-profit agency, receives free electricity on a continuous basis from a local utility company.
The utility company’s contribution is made subject to cancellation by the donor. Napro Charities should account for
this contribution as a(n)
A. Unrestricted revenue only.
B. Restricted revenue only.
C. Restricted revenue and an expense.
D. Unrestricted revenue and an expense.

“The LORD detests lying lips, but he delights in people who are trustworthy.” Proverbs 12:22

“With integrity, you have nothing to fear, since you have nothing to hide. With integrity, you will do the right
thing, so you will have no guilt.” Zig Ziglar

Good luck and God Bless!


CPA REVIEW SCHOOL OF THE PHILIPPINES
Manila

ADVANCE FINANCIAL ACCOUNTING AND REPORTING


FINAL PREBOARD EXAMINATIONS

Numbers 1 and 2

AB Corp. has two users, Department A and Department B. The following data apply for both
departments for the year:
Budgeted Overhead costs:
Fixed costs 3,700,000
Variable cost per direct labor hour 37.5
Budgeted direct labor hours:
Department A 80,000
Department B 120,000
Actual usage of direct labor hours:
Department A 60,000
Department B 80,000
AB Corp. applies overhead based on direct labor hours or uses a single overhead rate based on direct
labor hours.

1. What is the amount of overhead applied to Department A?


a. 3,360,000
b. 5,950,000
c. 5,025,000
d. 3,730,000

2. What is the amount of overhead applied to Department B?


a. 5,220,000
b. 4,480,000
c. 5,466,667
d. 6,700,000

Numbers 3 and 4
AB Corp. processes two products from a single raw material, Product A and Product B. The cost of
processing the two products up to the split-off point was P102,000. AB Corp. uses the NRV method of
allocating the joint cost. The following data were ascertained for the year:
Produced (gallons) Sales price Sales (gallons) Separable cost
Product A 12,000 P 7.00 8,500 P30,000
Product B 10,500 P18.00 9,000 P71,600

3. What is the joint cost allocated to Product A? (Round allocation ratio to the nearest percentage
ex. .71545 = 72%)
a. 25,500
b. 54,060
c. 32,640
d. 28,560

4. What is the joint cost allocated to Product B? (Round allocation ratio to the nearest percentage
ex. .71545 = 72%)
a. 69,360
b. 76,500
c. 47,940
d. 73,440
Page 2

Number 5
AB Corp. processes two products from a single raw material, Product A and Product B. The cost of
processing the two products up to the split-off point was P376,960.
The following data were extracted at the split-off point:
Produced (gallons) Sales price at split-off
Product A 42,500 P6.50
Product B 59,500 P5.50
The following data were extracted after further processing:
Produced (gallons) Final sales price Separable cost per gallon
Product A 42,000 P40.00 P8.00
Product B 58,200 P24.00 P8.00

What is the approximated/estimated net realizable value of Product B at the split-off point?
a. 1,360,000
b. 931,200
c. 1,344,000
d. 952,000

Numbers 6, 7 and 8
AB Corp. is experiencing financial difficulty and about to liquidate. The following data were available:
Book value of assets:
Cash 150,000
Inventories 200,000
Land 800,000
Stockholders’ equity:
Ordinary shares 500,000
Retained earnings (deficit) (325,000)
Book value of the liabilities of AB Corp. consists of accounts payable, salaries payable, loan payable,
and mortgage payable. The loan payable in the amount of P300,000 is secured by the inventories. The
mortgage payable in the amount of P450,000 is secured by the land with an estimated market value of
P400,000. Total liabilities with priority was P200,000 (including P50,000 estimated liquidation expenses
and salaries to employees). The holder of the mortgage payable received P440,000 at the end of
liquidation.

6. What is the estimated deficiency?


a. 15,000
b. 25,000
c. 10,000
d. 5,000

7. What is the amount received by the holder of the accounts payable?


a. 60,000
b. 75,000
c. 50,000
d. 65,000

8. What is the estimated market value of the inventories?


a. 350,000
b. 200,000
c. 450,000
d. 150,000
Page 3

Numbers 9, 10, 11, 12 and 13


Bacolod Manufacturing Corp. has the following cost of production data for the year:
Work-in-process beginning:
Job 03 Job 04
Direct materials 2,400 1,500
Direct labor 3,600 2,880
Applied overhead ? ?
Finished goods beginning:
Job 01 Job 02
Direct materials 18,000 1,500
Direct labor 24,000 2,880
Applied overhead ? ?
Total manufacturing cost added during the year:
Job 03 Job 04 Job 05 Job 06
Direct materials 10,920 13,200 36,000 4,800
Direct labor 14,400 16,800 42,000 7,200
Applied overhead ? ? ? ?
At the end of the year, Job 03, Job 04, Job 05, were completed. The predetermined overhead rate was
65% of direct labor cost. Actual overhead at the end of the year was P33,000.

9. What is the cost of goods manufactured at the end of the year?


a. 195,492
b. 180,900
c. 143,700
d. 180,912

10. What is the cost of goods manufactured for Job 03?


a. 105,300
b. 43,020
c. 47,172
d. 103,300

11. What is the cost of goods manufactured for Job 04?


a. 47,172
b. 40,920
c. 42,300
d. 43,020

12. What is the cost of goods manufactured for Job 05?


a. 8,340
b. 57,600
c. 34,680
d. 105,300

13. Assuming Job 06 was also completed, what is the cost of goods manufactured of Job 06?
a. 6,252
b. 20,580
c. 40,920
d. 16,680

14. What is the proper period for recording direct labor rate variance?
a. At the time of payment of salaries to factory workers
b. At the time of hiring of factory workers
c. At the time of rendition of services by factory workers
d. At the time of termination of employment of factory workers
Page 4
Number 15

On October 1, 2020, the Muntinlupa Main Office established a sales agency in Marikina.

 The main office sent samples of its merchandise amounting to P8,400 and a working fund
amounting to P72,000 to be maintained on the imprest basis.
 The samples sent were intended to last until June 1, 2021. During the first two months of
operations, the agency transmitted to the home office sale of goods amounting to P364,500, but the
home office were not able to fill-up 25% of the said transmitted sales orders.
 Collections from customers amounted to P73,941, net of 2% sales discount.
 Payments made by the agency during October and November were as follows: annual rent of
P57,600, transportation expense worth P5,600 and utilities amounting to P7,200.

 It also purchased an equipment worth P9,000 which will be depreciated at 20% per annum.
 The gross profit rate on sales agency order is 20% of gross sales.
What is the net income of the agency for the two months ended November 30, 2020?
a. 17,431
b. 28,366
c. 29,875
d. 26,866

Numbers 16, 17 and 18


The trial balance before adjustment for the Caloocan home office and Malabon branch show the
following items on December 31. Differences in the shipments account balances result from the home
office policy of billing the branch at a 20% mark-up based on cost.
Home Office books Branch books
Allowance for overvaluation of Branch merchandise P756,000
Shipments to branch ?
Purchases P 525,000
Shipments from Home Office 2,016,000
Merchandise Inventory, January 1 3,150,000
Sales 6,300,000
Expenses 378,000
The ending inventory of the branch amount to P2,100,000 composed of merchandise from the home
office amounting to P1,764,000. The remaining amount from outsiders.

16. By what amount did the Allowance for Overvaluation before adjustment increased during the
year?
a. 336,000
b. 756,000
c. 462,000
d. 420,000

17. In the books of the home office, what is the amount of adjustment to Income Summary –
Branch account at the end of the year?
a. 2,331,000
b. 510,996
c. 294,000
d. 462,000

18. Before closing, the Home Office Current account in the separate books of the branch is
understated by what amount?
a. 2,793,000
b. 462,000
c. 336,000
d. 2,331,000
Page 5

Numbers 19 and 20
Taguig Company has a branch in Makati and Pasig. The reciprocal accounts between the home office
and the branches were in agreement at the beginning of 2020. However at December 31, 2020, the
following reciprocal balances are found in the home office books: Investment in Makati, P186,500:
Investment in Pasig, P84,000.
Data for reconciliation of the reciprocal accounts are as follows:
a. On December 29, 2020, the home office has instructed Makati to transfer P74,000 cash to Pasig.
Makati recorded this transaction immediately. Upon receipt, Pasig has recorded this transfer at
P47,000. Taguig Company however has not yet recorded this interbranch transaction as of end of
the year.
b. Taguig has transferred goods costing P28,900 to Makati branch and paid P2,500 of shipping cost
on December 16, 2020. Makati shipped all of these goods to Pasig upon instruction of the home
office on December 30, 2020. Shipping cost is P3,600 freight collect. Had the goods were shipped
directly to the Pasig, P5,000 of freight cost should have been incurred. The interbranch shipment
was not recorded by the branches and the home office as well.
c. Makati has collected cash of P5,750 from Pasig’s customer. This transaction is not yet recorded
by Pasig and Taguig Company.
d. Taguig Company has already allocated P11,000 and P9,000 of administrative expenses to Makati
and Pasig respectively. The branches are not yet notified.
e. Makati remitted P14,300 cash to Taguig Company on December 12, 2020. The home office has
failed to record the said remittance.
f. Pasig returned goods costing P6,850 to Taguig Company. The goods were shipped on December
19 and received on December 24 but no entries have been made in the home office books.
Compute for the following:

19. What is the unadjusted balance of Home Office Current account in the books of Makati
branch?
a. 52,150
b. 87,200
c. 107,250
d. 92,950

20. What is the unadjusted balance of Home Office Current account in the books of Pasig branch?
a. 236,250
b. 122,000
c. 115,150
d. 84,850

21. How shall expenses of a nonprofit organization be presented in its statement of activities?
a. Addition to temporarily restricted net assets
b. Addition to permanently restricted net assets
c. Deduction from permanently restricted net assets
d. Deduction from unrestricted net assets

22. What is the cash account to be used by national government agencies for cash payments?
a. Cash modified disbursements system (MDS) regular
b. Cash treasury agency (TA) deposit regular
c. Cash in bank – Land Bank of the Philippines
d. Cash on hand

Page 6
Numbers 23, 24, 25 and 26
On January 1, 2018, MDC Inc. entered into a long-term construction contract for the construction of a
building at a contract price of P175M. Because of changes in the design of the project, the price increased
by P35M on January 1, 2020. The project was completed on December 31, 2021. The following
additional data are provided:
 Dependable estimates are available.
 Mobilization fee equivalent to 3% of original contract price must be made by the client
deductible on the first billing. Billings on the project were made 12%, 20%, 30% of the contract
price, respectively, for the first three years of the project.
 MDC made cash collection from the customer amounting to P17.5M and P52.5M on year 2019
and 2020.
 The cost incurred on year 2018 was P17.5M while the estimated cost to complete at the end of
2018 was P70M.
 The cumulative cost incurred in 2019 was P122.5M with a 50% degree of work completed as
of the end of 2019.
 The cost incurred on year 2020 was 35M with percentage of completion of 90% as of the end
of 2020.

23. What is the excess of construction in progress over progress billings (excess of
progress billings over construction in progress) on December 31, 2018?
a. 8.75M
b. 19.25M
c. 14M
d. 10.5M

24. What is the realized gross profit (loss) for the year ended December 31, 2019?
a. (87.5M)
b. (52.5M)
c. (70M)
d. (17.5M)

25. What is the amount of construction cost presented in the Statement of Comprehensive Income
in 2019?
a. 122.5M
b. 105M
c. 140M
d. 245M

26. What is the amount of account receivable to be reported by MDC on December 31, 2020?
a. 33.25M
b. 49M
c. 43.75M
d. 31.5M

27. Under PFRS 10, which financial statements must be presented by a parent corporation?
a. Separate financial statements
b. Combined financial statements
c. Consolidated financial statements
d. Condensed financial statements

28. Under PFRS 10, which of the following is not an essential element of control?
a. Power
b. Ability
c. Exposure or Right
d. Ownership of majority of voting stocks

Page 7
Numbers 29 and 30
On August 1, 2020, Tea-Account Inc. granted a franchise right to a franchisee for the operation of tea
shop using Tea-Account’s trade name for a period of 10 years starting August 1, 2020. The franchisee
is required to pay non-refundable initial franchise fee of P1,960,000 and continuing franchise fee of 4%
of franchisee’s annual sales. As part of its initial service, it is the obligation of the franchiser to train nine
staff and crew of the tea shop. In addition to that, Tea-Account has the obligation to deliver 1,200 units
of raw materials to the franchisee. The stand alone selling price of the right to use Tea-Account’s trade
name is P1,800,000. The stand alone selling price of the training of nine staff and crew is P225,000 while
the stand alone selling price of the 1,200 units of raw materials is P90,000.
On December 2, 2020, the franchisee started to operate. 800 units of raw materials were already delivered
and seven staff and crew were already trained as of December 31, 2020. The franchisee reported sales
revenue amounting to P280,000 on its first month of operation.

29. Under IFRS 15, what is the amount of Sales Revenue to be credited by Tea-Account Inc. on
December 31, 2020?
a. 55,603
b. 162,175
c. 83,404
d. 69,504

30. Under IFRS 15, what is the amount of Service Revenue to be credited by Tea-Account Inc. on
December 31, 2020?
a. 162,175
b. 217,778
c. 228,978
d. 287,282

31 Which of the following costs incurred by the acquirer in relation to business combination shall be
expensed as incurred?
a. Bond issue costs of financial liability at amortized cost
b. Direct costs of business combination
c. Stock issuance costs
d. Contingent consideration

Number 32
QRS Manufacturing Company ships merchandise costing P360,000 on consignment to XYZ Stores
under a perpetual inventory system with P37,500 of freight prepaid. XYZ pays P22,500 for local
advertising costs that are reimbursable from QRS. By the end of the period, XYZ has sold 2/5 of the
consigned merchandise for P210,000 cash. XYZ notifies QRS of the sales, retains 15% commission,
and remits the cash due to QRS.
Which of the following statements regarding the transactions above is TRUE?
a. XYZ will debit an expense account upon remittance to QRS.
b. QRS will debit a liability account upon receipt of the remittance from XYZ.
c. XYZ will prepare a memo entry for the P37,500 freight.
d. QRS will credit an asset account in the amount of P159,000 upon receiving the sales report.

33. Under PFRS 15, when shall an entity recognize its revenue from contract with customers?
a. When an entity enters into a contract with its customers
b. When an entity receives the consideration from its customers
c. When an entity satisfies its performance obligation to its customers
d. When an entity transfers the risk and rewards of ownership of the goods or services to its
customers

Page 8
34. When will the equivalent unit of production of direct materials under FIFO Process Costing be
always equal to Average Process Costing?
a. When all direct materials are added at the start of production process
b. When all direct materials are added at the end of production process
c. When all direct materials are added at the middle of production process
d. When direct materials are added uniformly throughout the production process

Number 35
An acquirer incurred the following costs in relation to the acquisition of a business:
 Stock issuance costs 100,000
 Direct costs of business combination 200,000
 Bond issue costs 300,000
How much of the abovementioned costs shall be capitalized as goodwill arising from business
combination?
a. 200,000
b. 300,000
c. 400,000
d. 0

Numbers 36 and 37
On January 1, 2021, Entity A and Entity B established a joint venture by investing P5,000,000 each for
equal capital interest in the arrangement. The joint venture reported the following data for the years
ended 2021, 2022 and 2023:
Year Net Income / (net Loss) Dividends Declared
2021 1,000,000 200,000
2022 (12,000,000) -
2023 6,000,000 3,000,000
36. What is the investment loss to be reported by Entity A in relation to the joint venture for the
year ended December 31, 2022?
a. (5,000,000)
b. (6,000,000)
c. (5,400,000)
d. (5,500,000)

37. What is the investment income to be reported by Entity B in relation to the joint venture for
the year ended December 31, 2023?
a. 3,000,000
b. 2,400,000
c. 1,500,000
d. 900,000

38. Which of the following claims shall be preferred in the liquidation of general partnership?
a. Just and equitable share of industrial partner in partnership profit
b. Proportionate share of capitalist partners in partnership profit
c. Advances to partnership by managing partner
d. Capital contribution of controlling partner

39. Which of the following claims during corporate liquidation shall be settled last?
a. Book value per share of ordinary shareholders
b. Liquidation value per share of preferred stockholders
c. Redemption value of redeemable preferred shareholders
d. Claims of unsecured creditors without priority

Page 9
Number 40
On January 1, 2021, Entity P acquired 80% of the common stocks of Entity S at a price of P2,400,000.
On the said date, the fair value of net assets of Entity S is estimated to be P3,200,000. The fair value of
the non-controlling interest is approximated at P650,000. It is the company policy to measure the non-
controlling interest at the minimum.
What is the gain on bargain purchase to be recognized by Entity P on January 1, 2021?
a. 160,000
b. 200,000
c. 150,000
d. 210,000

Numbers 41 and 42

The following assets are translated at the end of reporting period:

Assets Historical Rate Closing Rate


Cash 10,000,000 15,000,000
Inventory 20,000,000 25,000,000
Land 30,000,000 40,000,000

41. If the entity is translating from foreign currency to functional currency, what is the amount of
total assets to be presented by the entity at the end of reporting period?
a. 60,000,000
b. 80,000,000
c. 65,000,000
d. 75,000,000

42. Assuming the entity is translating from functional currency to its presentation currency, what
is the amount of total assets to be presented by the entity at the end of reporting period?
a. 60,000,000
b. 80,000,000
c. 65,000,000
d. 75,000,000

Numbers 43 and 44

On January 1, 2021, Entity P acquired 70% of the common stocks of Entity S at a price of P7,000,000
when the fair value of net assets of Entity S is approximated at P9,500,000. All the assets of entity S are
properly valued except for a building which is undervalued by P300,000 with a remaining useful life of
6 years. For the year ended December 31, 2021, Entity S reported net income of P250,000 and declared
P100,000 dividends to its common stockholders.

43. Under Equity Method, what is the book value of Investment in Entity S to be reported by Entity
P on its December 31, 2021 Statement of Financial Position?
a. 7,105,000
b. 7,000,000
c. 7,175,000
d. 7,070,000

44. If Entity P uses Cost Method, what is the income to be reported by Entity P in relation to its
investment in Entity S in its separate income statement for the year ended December 31, 2021?
a. 70,000
b. 140,000
c. 175,000
d. 75,000

Page 10

45. Which of the following costs shall be classified as conversion costs?


a. Direct labor costs and direct material costs
b. Direct labor costs and factory overhead costs
c. Direct material costs and factory overhead costs
d. Direct material costs, direct labor costs and factory overhead costs

Numbers 46 and 47
On July 1, 2021, Entity P acquired 60% of common stocks of Entity S. On the said date, all assets of
Entity S are properly valued except for an equipment with book value of P100,000 and fair value of
P150,000. On July 1, 2021, the equipment has remaining useful life of 5 years. On October 1, 2021,
Entity S sold the said undervalued equipment to Entity P at a price of P120,000. On December 31, 2021,
Entity P sold the said equipment to a third party at a price of P100,000.

46. What is the consolidated depreciation expense on the said equipment that will be reported by
Entity P for the year ended December 31, 2021?
a. 30,000
b. 15,000
c. 10,000
d. 13,500

47. What is the consolidated gain (loss) on sale of equipment to be reported by Entity for the year
ended December 31, 2021?
a. (35,000)
b. 10,000
c. (8,000)
d. 15,000

48. What is the proper classification of infrastructure asset if the concession operator has right or license
to charge users of the infrastructure asset over a specific period but it does not have guaranteed right
to receive cash from the grantor?
a. Property, plant and equipment
b. Investment property
c. Intangible asset
d. Financial asset

49. What is the accounting method to be used by an acquirer to treat each business combination?
a. Equity Method
b. Acquisition Method
c. Cost Method
d. Fair Value Method

Number 50
The following dividends were received by a parent entity from its investment in shares of stocks:
Cash dividend from Trading Securities 100,000
Property dividend from Associate at fair value 200,000
Property dividend from Joint Venture at fair value 300,000
Cash dividend from Subsidiary 400,000
What is the consolidated dividend income to be reported by parent entity in its consolidated
income statement?
a. 500,000
b. 400,000
c. 300,000
d. 100,000

Page 11

51. Which of the following transactions will increase the capital balance of a partner?
a. Share in other comprehensive loss of the partnership
b. Share in profit of the partnership
c. Drawing made during the period
d. Share in impairment loss arising from admission of a new partner

Numbers 52 and 53

On December 1, 2021, an entity acquired inventory on account at a cost of $1,000 payable on February
28, 2022. The functional currency of the entity is Philippine Peso. The following direct exchange rates
were given:

December 1, 2021 December 31, 2021 February 28, 2022


Buying Spot P50 P52 P47
Selling Spot P50 P59 P53

52. What is the foreign currency gain or (loss) to be reported by the entity for the year ended
December 31, 2022?
a. (5,000) loss
b. 6,000 gain
c. (4,000) loss
d. 2,000 gain

53. What is the book value of accounts payable to be reported by the entity on December 31, 2021?
a. 59,000
b. 55,000
c. 50,000
d. 52,000

54. When translating an entity’s financial statements from its functional currency to its presentation
currency, what is the exchange rate to be used to translate income or expense accounts?
a. Closing rate
b. Average rate
c. Transaction rate
d. Rate at the end of reporting period

55. In which method of accounting of investment in subsidiary in the separate financial statements of
parent corporation will the dividend income from subsidiary not appear?
a. Equity method
b. Cost method
c. FAFVPL
d. FAFVOCI

Number 56

Partner A and B have profits and loss agreement with the following provisions: Salaries of P30,000 and
P45,000 for A and B, respectively; a bonus to A of 10% of net income after salaries and bonus; and
interest of 10% on average capital balances of P20,000 and P35,000 for A and B, respectively. One-third
of any remaining profits are allocated to A and the balance to B.
If the partnership had net income of P102,500, how much should be allocated to partner A?
a. 41,000
b. 44,250
c. 41,167
d. 47,500

Page 12

57. Which of the following transactions will result to debit to investment in branch account?
a. Reporting net loss of the branch
b. Collection by the branch of home office’s receivable
c. Return by the branch to the home office of shipped inventory
d. Payment by the home office of its own liability

58. In which of the following contract shall the resulting revenue be recognized over a period of time?
a. Long-term construction contract
b. Consignment sale contract
c. Sale or return arrangement
d. Installment sale contract

Number 59
The UST Hospital had the following cash receipts and disbursements for the year ended December 31,
2020:
Collections of Receivables 500,000
Contribution restricted by the donor to the acquisition of medical supplies 70,000
Contribution for an establish of quasi endowment (Board-Designated) 100,000
Contribution restricted by donor to the acquisition of furniture and fixtures 90,000
Tuition from nursing school 200,000
Interest received from investment in regular endowments 35,000
Contribution for an establishment of term endowment 90,000
Payment of supporting expenses 150,000
Payment for the acquisition of furniture and fixtures related to the restricted
donation above. 60,000
Payment of program expenses 215,000
The interest received from regular endowment is restricted by the donor for acquisition of medical
equipment.
How much is the net cash provided by operating activities?
a. 335,000
b. 375,000
c. 410,000
d. 435,000

60. Which of the following statements is true?


a. Contributed services and facilities are recognized both as asset and contributions revenue, net of
expense.
b. Financial statements of not for profit organization, focuses on distinctions between current and
non-current fund.
c. Unconditional pledges are recognized as receivables and contributions revenue when collected.
d. The required financial statements for nonprofit organizations include: statement of
financial position, statement of cash flows, statement of activities and specifically for
Voluntary Health and Welfare Organizations – Statement of Functional Expenses.

61. Where shall foreign currency gain or loss arising from translating foreign currency denominated
elements of financial statements to entity’s functional currency be presented?
a. Income from Continuing Operation of Profit or Loss Section of Statement of
Comprehensive Income
b. Income from Continuing Operation of Profit or Loss Section of Statement of Comprehensive
Income
c. Other Comprehensive Income with Reclassification Adjustment Section of Statement of
Comprehensive Income
d. Other Comprehensive Income without Reclassification Adjustment Section of Statement of
Comprehensive Income

Page 13

Number 62
E, R and G operate a local accounting firm as partnership. After working together for several years, they
have decided to liquidate the partnership. The partners have presented the following balance sheet;

Cash P 200,000 Liabilities P400,000


Receivable from E 80,000 Loan Payable to R 100,000
Non-cash assets 1,620,000 E, Capital (10%) 900,000
R, Capital (50%) 300,000
G, Capital (40%) 200,000

The non-cash assets are sold for P800,000 with P210,000 of this amount being used to pay liquidation
expenses. All partners are personally insolvent.
How much of the cash must E received?
a. 261,667
b. 128,333
c. 390,000
d. 305,000

Number 63

FAR, MAS and TAX share profits and losses from their partnership in the ratio of 35%, 45% and 20%
respectively. Capital and loan balances related to each partner are as follows:

Loan to Partner from Partnership Loan to Partnership from partner Capital


FAR 100,000 500,000
MAS 70,000 280,000
TAX 200,000 250,000

In addition to loan to partner, assets of the partnership includes cash of P110,000, inventory of P360,000,
receivable of P260,000 and plant and equipment of P710,000. Partnership liabilities to non-partners
amount to P180,000.
If FAR receives already P450,000, how much TAX receives at this point?
a. 321,155
b. 364,286
c. 450,000
d. 375,000

Number 64

On May 1, 2020, the capital accounts of S, T and C are P1,260,000; P787,500 and P472,500 ,
respectively.
At this time, I is admitted to the firm, he purchased a 1/6 interest in the firm for P288,750. The old
partners equalized their capital investments. Afterwards, all the partners agree to divide profits and losses
equally. The new partnership closes its books June 30, 2020 reporting profit of P44,100 for two months.
Each partner made the following withdrawals: S and C P2,625 per month while T and I, P3,500 per
month. On June 30, 2020, I invest enough cash to increase his capital to a 1/3 interest in the partnership.
How much cash is to be invested by I?
a. 211,165.50
b. 70,000
c. 632,642.50
d. 633,762.50

Page 14
Number 65

Max decided to withdraw from his partnership with Fried and Chic. Before his withdrawal, Max’ capital
balance was P58,000, while Fried’s was P64,000 and Chic’s was P77,000. Also, the partnership’s total
assets amounted to P450,000, but the partners agreed that a fixed asset was under depreciated by
P15,000. Max, Fried and Chic share profits and losses in the ration of 2:4:4, respectively.
If Max was paid P53,200 upon his retirement, how much is the remaining partnership net assets
after Max’ withdrawal?
a. 182,800
b. 197,800
c. 130,800
d. 160,800

Number 66
Partners Irish, Ivan and Irvin share profits and losses in the ratio of 4:3:3. at the end of a very unprofitable
year, they decided to liquidate the firm. The balances of their accounts on this date are:
Cash 18,000
Other assets ?
Liabilities 90,000
Irish, cap 66,000
Ivan, cap 74,700
Irvin, cap 45,000
The liabilities included a loan of P30,000 from Irish. All the partners are personally solvent. The partners
plan to sell the assets on instalment.
If Ivan received P33,000 from the first distribution of cash, how much did Irish received at that
time?
a. 40,400
b. 24,750
c. 36,000
d. 33,000

Number 67
Solly and Dante are partners who share profits and losses in the ratio of 7:3, respectively. On February
1, 2020, their respective capital accounts were as follows: Solly P140,000 and Dante P120,000.
On that date they agree to admit Jam as partner with 1/3 interest in the capital and profit and losses, and
upon his investment of P100,000. The new partnership will begin with a total capital of P360,000.
Immediately after Jam’s admission, what are the capital balance of Solly, Dante and Jam?
a. 120,000; 120,000; 120,000
b. 126,000; 114,000; 120,000
c. 126,668; 113,322; 120,000
d. 140,000; 120,000; 100,000

68. What is the proper classification of Joint Arrangement when the parties exercising joint control
over arrangement has rights to the net assets of the said arrangement?
a. Joint operation
b. Jointly controlled asset
c. Joint venture
d. Jointly controlled operation

Page 15

Number 69
During 2020, Agency ABC transferred cash of P300,000 to Agency QRS for a land beautification
project. Subsequently, Agency ABC received a report from Agency QRS about the project. What is the
journal entry of Agency ABC to record the transfer of funds?
a. Debit Due from ABC P300,000 and Credit Cash, MDS, Regular P300,000.
b. Debit Cash-Treasury/Agency Deposit/Trust P300,000 and Credit Cash-Collecting Officers
P300,000.
c. Debit Cash Collecting Officers P300,000 and Credit Due to ABC P300,000
d. Debit Cash-Treasury/Agency Deposit/Trust P300,000 and Credit Cash-Collecting Officers
P300,000.

Number 70

An entity spent P5,600,000 (inclusive of VAT) in acquiring its new software package from outside party
at the beginning of the year. Such software shall be installed and used to speed up processing the entity’s
operations. The useful life of the software is determined to be 10 years and no residual value was assigned
to it.

What is the journal entry to record the purchase of a software package?

a. Computer Software 5,600,000


Cash-MDS, Regular 5,600,000

b. Computer Software 5,600,000


Cash Disbursing Officer 5,250,000
Due to BIR 350,000

c. Computer Software 5,600,000


Cash-Treasury/Agency Deposit 5,000,000
Due to BIR 600,000

d. Computer Software 5,600,000


Cash-MDS, regular 5,250,000
Due to BIR 350,000

-- E N D --
lOMoARcPSD|12648093

[AFAR] Re SA 41 - First Pre-Bo a rd Ex a ms (Questions,


Answers & Solutions)
Accountancy (Holy Angel University)

StuDocu is not sponsored or endorsed by any college or university


Downloaded by ruzz ruzz (ruzzst@jruzz.ml)
lOMoARcPSD|12648093

ReSA - THE REVIEW SCHOOL OF ACCOUNTANCY


CPA Review Batch 41  May 2021 CPA Licensure Examination 
First Pre-Board Examination
ADVANCED FINANCIAL ACCOUNTING & REPORTING Saturday, 20 February 2021 (6PM-9PM)

INSTRUCTIONS: Select the correct answer for each of the following questions. Mark only one answer for each
item by shading the box corresponding to the letter of your choice on the answer sheet provided. STRICTLY NO
ERASURES ALLOWED. Use pencil no. 2 only.
Set A
---------------------------------------------------------------------------------------------------------------------------
------

1. Tillman Textile Company has a single branch in Bulacan. On March 1, 2019, the home office
accounting records included an Allowance for Overvaluation of Inventories - Bulacan Branch
ledger account with a credit balance of P32,000. During March, merchandise costing P36,000
was shipped to the Bulacan Branch and billed at a price representing a 40% markup on the billed
price. On March 31, 2019, the branch prepared an income statement indicating a net loss of
P11,500 for March and ending inventories at billed prices of P25,000. What is the amount of
adjustment for Allowance for Overvaluation of Inventories to reflect the true branch net income?
A. P39,257 debit C. P39,333 debit
B. P46,000 credit D. P46,000 debit
2. The PQR Partnership is being dissolved. All liabilities have been paid and the remaining assets are
being realized gradually. The equity of the partnership is as follows:
Loans to
Partner’s (from) Profit and
Accounts Partnerships Loss Ratio
P P24,000 P 6,000 3
Q 36,000 - 3
R 60,000 (10,000) 4
The second cash payment to any Partner (s) under a program of priorities shall be made thus:
A. To R P2,000 C. To R P8,000
B. To Q P6,000 D. To Q P6,000 & R P8,000
Use the following information for 3 and 4:
Partners Dennis and Lilly have decided to liquidate their business. The following information is
available:
Cash . . . . . . . . . . . . . P 100,000 Accounts Payable . . P 100,000
Inventory . . . . . . . . . . 200,000 Dennis, Capital . . . . 120,000
Lilly, Capital . . . . . . . . __80,000
Total . . . . . . . . . . . . . . P 300,000 Total . . . . . . . . . . . . . . . P300,000

Dennis and Lilly share profits and losses in a 3:2 ratio. During the first month of liquidation, half the
inventory is sold for P60,000, and P60,000 of the accounts payable is paid. During the second month,
the rest of the inventory is sold for P45,000, and the remaining accounts payable are paid. Cash is
distributed at the end of each month, and the liquidation is completed at the end of the second
month.
3. Using a safe payments schedule, how much cash will be distributed to Dennis at the end of the first
month?
A. P 64,000 C. P 24,000
B. P 60,000 D. P 36,000
4. Assume instead that the remaining inventory was sold for P10,000 in the second month. What
payments will be made to Dennis and Lilly at the end of the second month?
Dennis Lilly Dennis Lilly
A. P 0 P 0 C. P 5,000 P 5,000
B. P 10,000 P 0 D. P 6,000 P 4,000
Items 5 and 6 are based on the following information:
On May 15, 2019, Atlas Sales Company received a shipment of merchandise with a selling price of
P15,000 from Philco Company. The consignment agreement provided for a sale of merchandise with
a credit with terms of 2/10 n/30. The commission of 15% was to be based on the accounts receivable
collected by the consignee. Cash discounts taken by customers, expenses applicable to goods on
consignment and any cash advanced to the consignor were deductible from the remittance by the
consignee.

Page 1 of 21 0915-2303213/0908-6567516  www.resacpareview.com

Downloaded by ruzz ruzz (ruzzst@jruzz.ml)


lOMoARcPSD|12648093

First Pre-board Examination -Batch 41


ADVANCED FINANCIAL ACCOUNTING & REPORTING
Atlas Sales Company advanced P6,000 to Philco Company upon receipt of the shipment. Expenses of
P800 was paid by Atlas. By June, 2019, 70% of the shipment had been sold, and 80% of the resulting
accounts receivable had been collected, all within the discount period. Remittance of the amount
due was made on June 30, 2019.
The consigned goods cost Philco Company P10,000 and freight charges of P120 had been paid to
ship it to Atlas Sales Company.
5. The cash remitted by Atlas Sales Company
A. P172 C. P2,230
B. P340 D. P2,340
6. The cost of inventory on consignment amounted to:
A. P1,500 C. P3,036
B. P3,000 D. P3,186
7. Under cash priority program, when all of the priorities are paid, any remaining cash distribution is

A. allocated to the partners based on their respective profit or loss ratios.


B. allocated to the partners based on the balances in their capital accounts after allocation of
losses.
C. allocated to the partners based on their pre-computed priorities.
D. allocated to the partners based on the relative values of their capital balances.
8. Which of the following is false in relation to consignment sales of a manufacturer of a household
goods to a retailer.
A. The manufacturer retains title to the products until they are scanned at the register.
B. The retailer does not have an obligation to pay the manufacturer until a sale occurs and any
unsold products may be returned to the manufacturer.
C. The manufacturer retains the right to call back or transfer unsold products to another retailer
until the sale to the consumer.
D. Once the retailer sells the products to the consumer, the manufacturer still has obligations for
the products, and the retailer still has return rights.

9. Eagle Company recently petitioned for bankruptcy and is now in the process of preparing a
statement of affairs. The following information has been assembled for this statement:

Assets Book Value Estimated Current Value


Cash . . . . . . . . . . . . . . . . . . . . P 70,000 P 70,000
Other current assets . . . . . . . 240,000 230,000
Building . . . . . . . . . . . . . . . . . . 600,000 700,000
Land . . . . . . . . . . . . . . . . . . . . 200,000 300,000
Liabilities
Liabilities with priority . . . . . . . . . . . . . . . . . . . . . . . . P 140,000
Mortgage payable (secured by building) . . . . . . 300,000
Note Payable (secured by land) . . . . . . . . . . . . . . 400,000
Unsecured liabilities . . . . . . . . . . . . . . . . . . . . . . . . . 600,000

What amount will be paid to the fully secured creditors and the creditors with priority?
Fully Secured Creditors Creditors with Priority
A. P300,000 P140,000
B. P300,000 P 92,000
C P600,000 P 92,000
.
D. P700,000 P140,000

10. Anselmo Company operates retail hobby shops from the main store and a branch store.
Merchandise is shipped from the main store and to the branch and billed to the branch at an
arbitrary 10% markup. Trial balances of the main store and branch as of December 31, 2018 are
as follows:
Main Store Branch
Debits:
Cash P 1,500 P 1,000
Accounts receivable – net 200 -
Inventory, December 31, 2017 3,500 2,500
Building – net 60,000 18,000

Page 2 of 21 0915-2303213/0908-6567516  www.resacpareview.com

Downloaded by ruzz ruzz (ruzzst@jruzz.ml)


lOMoARcPSD|12648093

First Pre-board Examination -Batch 41


ADVANCED FINANCIAL ACCOUNTING & REPORTING
Equipment – net 30,000 12,000
Branch store 32,300 -
Purchases 240,000 11,000
Shipments from home office - 99,000
Other expenses 15,000 7,000
Total debits P 382,500 P 150,500

Credits:
Accounts payable P 15,000 P 500
Unrealized inventory profit 9,200 -
Main Store - 30,000
Capital stock 50,000 -
Retained earnings 16,000 -
Sales 200,000 120,000
Shipments to branch 90,000 -
Profit from branch ____2,300 _________
Total credits P 382,500 P 150,500

Inventories on hand at December 31, 2018 at the main store and branch are P3,000 and P1,800,
respectively. The December 31, 2017 branch inventory includes merchandise purchased from
outsiders of P300, and the December 31, 2018 branch inventory includes P150 of merchandise
purchased from outsiders. The combined cost of goods sold amounted to:
A. P261,200 C. P243,150
B. P252,200 D. P252,150

11. The MSB Partnership has the following amounts:


● Sales, P84,000
● Cost of goods sold, P48,000
● Operating expenses, P12,000
● Salary allocations to partners, P15,600
● Interest paid to banks, P2,400
● Partners’ withdrawals, P9,600
Compute the partnership net income (loss):
A. P24,000 C. P 6,000
B. 21,600 D. ( 3,600)

Items 12 and 13 are based on the following information:


Daniela Co. (DC), a soft drink company, enters in to a licensing arrangement with Camila Worldwide
Inc. (CWI), an apparel company. The licensing arrangement permits CWI to use the DC trademarked
logo and tagline on a new line of CWI’s T-shirts, hats, shorts and other apparel for a three-year period.
As consideration, CWI pays DC a one-off fee of P55 million at the beginning of the license term and
an 11% sales-based royalty, calculated from the total quarterly sales of apparel items that include the
DC logo. The rights and terms granted by DC to CWI under the agreement are similar to those
granted by DC in licensing arrangements with other apparel companies. CWI will provide updated
sales data on a quarterly basis. DC has determined the license is a distinct performance obligation.
DC will undertake activities that will affect the intellectual property (IP) to which CWI has rights. The
rights granted by the license directly expose CWI to positive or negative effects of changes in the
activities on the IP. DC activities do not transfer a separate good or service to CWI as those activities
occur, even if CWI may benefit from the activities.
12. Applying PFRS 15, the upfront payment of P55M will be recognized as revenue

A. Point in time B. Over time C. Over the 3 year period D. B and C


13. Applying PFRS 15, the sales-based royalties (variable consideration)
A. are excluded from the transaction price until the underlying sales occur, at which point,
revenue from the sales-based royalties are recognized.
B. are assessed if highly probable to occur
C. are included from the transaction price until the underlying sales occur, at which point, revenue
from the sales-based royalties are recognized.
D. B and C

Page 3 of 21 0915-2303213/0908-6567516  www.resacpareview.com

Downloaded by ruzz ruzz (ruzzst@jruzz.ml)


lOMoARcPSD|12648093

First Pre-board Examination -Batch 41


ADVANCED FINANCIAL ACCOUNTING & REPORTING
14. As suggested by Article 1787 of the Philippine Civil Code and relevant PFRSs, the net contributions
(assets and related liabilities assumed by the partnership) of the partners to the partnership are
measured at

A. fair value C. discretionary amount determined by partners


B. cost D. any of these
15. The interest of the withdrawing, retiring, or deceased partner shall be adjusted for which of the
following?
I. his share of any profit or loss up to the date of his withdrawal, retirement or death, if he
withdraws, retires or dies during the year
II. his share of any revaluation gains or losses as at the date of his withdrawal, retirement, or
death

A. I only B. II only C. I or II D. I and II


16. Virtuoso has a sales agency in Cebu. Agency revenues and expenses are recorded in separate
agency accounts, with the operating results of both the agency and the home office generated
at each month-end. For the month of October 20x4, the home office paid P10,000 for advertising
costs on behalf of the agency and recorded this as follows:
A. Cash agency . . . . . . . . . . . . . . . . . . . . . . . . . . . 10,000
Cash . . . . . . . . . . . . . . . . . . . . . . . . . . . . . 10,000
B. Advertising expense . . . . . . . . . . . . . . . . . . . . 10,000
Cash . . . . . . . . . . . . . . . . . . . . . . . . . . . . .. 10,000
C. Accounts receivable – Cebu Agency . . . . . . 10,000
Cash . . . . . . . . . . . . . . . . . . . . . . . . . . . . . 10,000
D. Advertising expense – Cebu Agency . . . . . . . 10,000
Cash . . . . . . . . . . . . . . . . . . . . . . . . . . . . . . 10,000
Items 17 and 18 are based on the following information:
Account balances for the Ral, Tom, and Vic partnership on October 1, 2008 are as follows:
Cash P 21,000 Accounts payable P 80,000
Accounts receivable 63,000 Notes payable 50,000
Inventory 120,000 Ral, capital (30%) 43,600
Equipment 150,000 Tom, capital (50%) 150,000
Ral loan 15,000 Vic, capital (20%) 45,400
The partners have decided to liquidate the business. Activities for October and November are as
follows:
October
● Ral is short of funds and the partners agree to charge her loan to her capital account.
● P40,000 is collected on the accounts receivable; P4,000 is written off as uncollectible.
● Half the inventory is sold for P50,000.
● Equipment with a book value of P55,000 is sold for P60,000.
● The P50,000 bank note plus P600 accrued interest is paid in full.
● The accounts payable are paid.
● Liquidation expenses of P2,000 are paid.
● Except for a P5,000 contingency fund, all available cash is distributed to partners at the
end of October.
November
● The remaining equipment is sold for P38,000.
● Vic accepts inventory with a book value of P20,000 and a fair value of P10,000 as payment
for part of her capital balance. The rest of the inventory is written off.
● Accounts receivable of P10,000 are collected. The remaining receivables are written off.
● Liquidation expenses of P800 are paid.
● Remaining cash, including the contingency fund, is distributed to the partners.
17. How much would Tom receive for the month of October?
A. P16,700 C. P34,286
B. P33,400 D. P35,400
18. How much cash would Vic receive for the month of November?
A. P 6,886 C. P10,400
B. P 9,720 D. P35,400
Items 19 through 21 are based on the following information:
Selected balances from the Cebu Company’s Branch A and B are as follows:
Branch A Branch B
Inventory, Jan. 1, 2018 P 21,000 P 19,000

Page 4 of 21 0915-2303213/0908-6567516  www.resacpareview.com

Downloaded by ruzz ruzz (ruzzst@jruzz.ml)


lOMoARcPSD|12648093

First Pre-board Examination -Batch 41


ADVANCED FINANCIAL ACCOUNTING & REPORTING
Imprest Branch Fund 2,000 1,500
Inventory, Dec. 31, 2018 19,000 12,000
A/Receivable, Jan. 1, 2018 55,000 43,500
Merchandise from Home Office… 61,000 47,000
A/Receivable, Dec. 31, 2018 70,000 53,500
Sales 100,000 80,000
Cash Expenses… 21,000 14,300
All sales, collections, and expenses are handled at the branch. All cash received from sales and
collections are sent directly to the Home Office. Expenses are paid by the branch from the imprest
fund and immediately reimbursed by the Home Office and credited to the Home Office account. All
expenses paid by the branch are recorded in the books of the branch.
19. Compute the balance of the Home Office account in the books of Branch on January 1, 2018:
A B A B
A. P163,000 P67,000 C. P139,000 P111,000
B. P 64,000 P78,000 D. P 78,000 P 64,000
20. Compute the balance of the Home Office account on December 31, 2018.
A B A B
A. P110,000 P152,000 C. P64,000 P78,000
B. P 91,000 P 67,000 D. P78,000 P64,000
21. The entry in Branch B’s records in order to update the reciprocal Home Office Account on
December 31, 2018 assuming net income of the branch is being reported to the home office:
A. Dr. – Home Office Current / Cr. – Profit and Loss
B. Dr. – Profit and Loss / Cr. - Branch Current
C. Dr. – Branch Current / Cr. – Profit and Loss
D. Dr. - Profit and Loss / Cr. – Home Office
22. An entity shall determine whether a transaction or other event is a business combination by
applying the definition in PFRS 3, Business Combinations, which requires that the assets acquired
and liabilities assumed constitute a/an _________. If the assets acquired are not a/an _______, the
reporting entity shall account for the transaction or other event as a/an ______.

A. operating segment, operating segment, business combination


B. operating segment, business, asset acquisition
C. business, business, asset acquisition
D. business, operating segment, asset acquisition
23. Marga Holding Inc., a sub-holding of the Mondragon Group, makes an offer for all the equity
shares of Cassie Ltd. on July 1, 2018. The consideration for the offer is 50,000 shares in Marga
together with P10,000,000 cash. The offer is accepted on August 1, 2018. However, the offer is
conditional upon receiving the approval of the competition authority which is obtained on
September 30, 2018. In the past, the competition authority has never rejected the application for
any merger or combination. The shares are exchanged on August 10, 2018. What is the date of
acquisition?
A. July 1, 2018, the date of the offer
B. August 1, 2018, the date the offer has been accepted
C. August 10, 2018, the date the shares have been exchanged
D. September 30, 2018, the date of the approval by the competition authority
24. Under PFRS 15, sales-based royalty is recognized
A. as sales occurs
B. when the performance obligation is satisfied
C. at the later of: (1) the occurrence of sales or (2) when the performance obligation is satisfied
D. as the services are provided or the rights used

Items 25 to 27 are based on the following information:


OO and PP are partners sharing profits in this proportion – 60:40. A balance sheet prepared for the
partners on April 1, 20x4 shows the following:
Cash . . . . . . . . . . . . . . . . . . . . P48,000 Accounts payable . . . . . . . . . P 89,000
Accounts Receivable . . . . . . . 92,000 OO, capital . . . . . . . . . . . . . . 133,000
Inventories . . . . . . . . . . . . . . . . 165,000 PP, capital. . . . . . . . . . . . . . . 108,000
Equipment . . . . . . . . . . . . 70,000

Page 5 of 21 0915-2303213/0908-6567516  www.resacpareview.com

Downloaded by ruzz ruzz (ruzzst@jruzz.ml)


lOMoARcPSD|12648093

First Pre-board Examination -Batch 41


ADVANCED FINANCIAL ACCOUNTING & REPORTING
Less: Accumulated
Depreciation. . . . . . . . 45,000 25,000 _________
Total Assets . . . . . . . . . . . . . . P330,000 Total Liabilities & Capital . . . . P 330,000

On this date, the partners agree to admit RR as a partner. The terms of the agreement are
summarized below.

Assets and liabilities are to be restated as follows:


● An allowance for possible uncollectible of P4,500 is to be established.
● Inventories are to be restated at their present replacement value of P170,000.
● Accrued expenses of P4,000 are to be Recognized.

OO, PP and RR will divide profits in the ratio of 5:3:2. Capital balances of the partners after the
formation of the new partnership are to be in the aforementioned ratio, with OO and PP making cash
settlement between them outside of the partnership to adjust their capitals, and RR investing cash in
the partnership for his interest.
25. The cash to be invested by RR is:

A. P60,250 C. P50,000
B. P47,500 D. P59,375
26. The total capital of the partnership after the admission of RR is:
A. P296,875 C. P237,500
B. P301,250 D. P286,850
27. Cash settlement between OO and PP is:
A. OO will pay PP P17,537.50 C. OO will invest P17,537.50
B. PP will pay OO P17,537.50 D. PP will withdraw P17,537.50
28. PFRS 3 – Business Combinations does not apply to which of the following?
I. Formation of a joint arrangement.
II. Combination of entities or businesses under common control.
III. Acquisition of an asset or a group of assets that constitute a business.
IV. Acquisition by an investment entity of an investment in a subsidiary with that subsidiary not
providing services that relate to the investment entity's investment activities
V. Not-for-profit organizations.
A. I, II and IV only B. I, II, IV and V only C. I, II, III and V only D. I, II, III, IV and V

Items 29 and 30 are based on the following information:


29. Hotel Dian Inc. charges an initial franchise fee of P90,000 broken down as follows:
Rights to trade name, market area, and proprietary know-how P40,000
Training services 11,500
Equipment (cost of P10,800) 38,500
Total initial franchise fee P90,000
Upon signing of the agreement, a payment of P40,000 is due. Thereafter, two annual payments of
P30,000 are required. The credit rating of the franchisee is such that it would have to pay interest of
8% to borrow money. The franchise agreement is signed on August 1, 20x4, and the franchise
commences operation on November 1, 20x4. Assuming that no future services are required by the
franchisor once the franchise begins operations, the entry on November 1, 20x4 would include
A. a credit to Unearned Franchise Revenue for P40,000.
B. a debit to Service Revenue for P11,500.
C. a debit to Sales Revenue for P38,500.
D. a debit to Unearned Franchise Revenue for P40,000.

30. Assuming that the franchise agreement is signed on August 1, 20x5, and the franchise commences
operation on November 1, 20x5. Assume that the total training fees includes training services for
the period leading up to the franchise opening (P5,500 value) and for 3 months following opening.
The journal entry on August 1, 20x5 would include
A. a credit to Unearned Service Revenue for P11,500.
B. a credit to Unearned Service Revenue for P6,000.
C. a debit to Sales Revenue for P38,500.
D. a debit to Unearned Franchise Revenue for P40,000.

Page 6 of 21 0915-2303213/0908-6567516  www.resacpareview.com

Downloaded by ruzz ruzz (ruzzst@jruzz.ml)


lOMoARcPSD|12648093

First Pre-board Examination -Batch 41


ADVANCED FINANCIAL ACCOUNTING & REPORTING
Items 31 and 32 are based on the following information:
On August 5, 20x5, Famous Furniture shipped 20 dining sets on consignment to Furniture Outlet, Inc.
The cost of each dining set was P350. The cost of shipping the dining sets amounted to P1,800 and
was paid for by Famous Furniture. On December 30, 20x5, the consignee reported the sale of 15
dining sets at P850 each. The consignee remitted payment for the amount due after deducting a 6%
commission, advertising expense of P300, and installation and setup costs of P390.

31. The amount cash received by Famous Furniture is


A. P12,750 C. P11,685
B. P11,985 D. P11,295

32. The total profit on units sold for the consignor is


A. P11,295 C. P6,045
B. P 9,945 D. P4,695
33. If the partnership agreement does not specify how income is to be allocated, profits and loss
should be allocated

A. equally.
B. in accordance with their capital contribution.
C. in proportion to the weighted average of capital invested during the period.
D. equitably so that partners are compensated for the time and effort expended on behalf
of the partnership.
34. It is common for a construction entity to receive numerous variation orders from the customer
during the period of construction. These variation orders could arise due to changes in the design
of the asset being constructed and in the type of materials to be used for construction. Which of
the following is/are true about the requirements of PFRS 15 in relation to these changes?
I. These change orders or contract modification may need to be accounted for as a new
contract separate from the original contract if the modification adds distinct goods or services
priced at their stand-alone selling prices.
II. These change orders or variations in contract work are included in the contract revenue when
it is probable that the customer will approve the variation and the amount of revenue arising
from the variation can be reliably measured.

A. I only B. II only C. I or II D. I and II


35. It is possible that the construction entity will be entitled to an incentive bonus (for example, if the
quantity of the materials used in the project did not vary significantly from the budgeted quantity
in the bidding documents, if the entity will complete the project ahead of the target completion).
Which of the following is/are true about accounting for these variable considerations as provided
under PFRS 15?
I. If the consideration in the contract involves variable amount (incentives, claims, penalties,
etc.), the entity should estimate the amount of consideration to which it expects to be entitled.
II. An entity needs to estimate any variable consideration using either the expected value
method or the most likely amount method, and include it in the revenue to the extent that it is
highly probable that the revenue will not reverse.
III. Incentive payments are included in contract revenue when: (a) the contract is sufficiently
advanced that it is probable that the specified performance standards will be met or
exceeded; and (b) the amount of the incentive payment can be measured reliably.
IV. Claims are included in contract revenue only when: (a) negotiations have reached an
advanced stage such that it is probable that the customer will accept the claim; and (b) the
amount that it is probable will be accepted by the customer can be measured reliably.

A. I, II, III and IV B. I, II and IV only C. I and II only D. II only


36. Happy, Inc. opens a sales agency in Davao City, and a working fund for P20,000 is established on
the imprest basis. The first payment from the fund is P3,000 for rent. This transaction should be
recorded by the home office as follows:(9-F26)
A. No entry

B. Rent……………………………………………………………………………………3,000
Cash………………………………………………………………………… ....... 3,000
C. Davao Agency……………………………………………………………….......... 3,000
Cash…………………………………………………………………………….. 3,000
D. Davao Agency……………………………………………………………………… 3,000

Page 7 of 21 0915-2303213/0908-6567516  www.resacpareview.com

Downloaded by ruzz ruzz (ruzzst@jruzz.ml)


lOMoARcPSD|12648093

First Pre-board Examination -Batch 41


ADVANCED FINANCIAL ACCOUNTING & REPORTING
Working Fund..………………………………………………………………… 3,000
37. It is possible that: (1) a contractor may lower the pricing for a particular contract (which the client
was invited to bid) and recover foregone profits from other contracts with the same customer; or
(2) a contractor be awarded with multiple contracts with the same customer wherein the ultimate
goal of these contracts is to deliver a combined single output to the customer. PFRS 15 requires
entities to combine contracts entered into at or near the same time with the same customer if they
meet one or more of the following criteria, except:
A. the contracts are negotiated as a package with a single commercial objective
B. multiple contracts negotiated at the same time
C. the amount of consideration to be paid in one contract depends on the price or
performance of the other contract; or
D. the goods or services promised in the contracts (or some goods or services promised in each
of the contracts) are a single performance obligation

Items 38 through 40 are based on the following information:


On December 3, 2018, the Home Office of Kathy Office Supply Company recorded a shipment of
merchandise to its Davao branch as follows: (5-7F26)
Davao Branch……………………………………………………………… 39,000
Shipments to Branch………..………………………………………… 32,500
Unrealized Profit in Branch Inventory……….................................. 5,200
Cash (for freight charges)……………………………………............ 1,300

The Davao branch sells 40% of the merchandise to outside entities during the rest of December 2018.
The books of the home office and Kathy Office Supply are closed on December 31 of each year.
On January 5, 2019, the Davao branch transfers half of the original shipment to the Baguio branch,
and the Davao branch pays P650 as the shipment.

38. What amount should the 60% of the merchandise remaining unsold be included in (1) the
inventory of the Davao branch at December 31, 2018,
A. P20,280 C. P23,400
B. P22,620 D. P23,920
39. What amount should the 60% of the merchandise remaining unsold at December 31, 2018 be
included in the published balance sheet of Kathy Office Supply at December 31, 2018 shows
inventory at:
A. P19,500 C. P20,800
B. P20,280 D. P23,400

40. What is the entry on the home office books in respect to January 5, 2019 transfer, assuming that
the transfer cost of the merchandise to Baguio branch would have been P780.
A. Home Office………………………………………………………………………………………20,150
Cash…………………………………………………………………………………………… 780
Inventory……………………………………………………………………………………... 19,500
B. Shipments…………………………………………………………………………………………18,850
Freight-in………………………………………………………………………………………….. 780
Home Office Current………………………………………………………….................. 19,630
C. Branch Current – Baguio………………………………………………………………….......19,630
Excess Freight....………………………………………………………………………………… 520
Branch Current – Davao…......................……………………………………………… 20,150
D. Branch Current – Baguio…………………………………………………………………......19,630
Excess Freight…………………………………………………………………………………… 780
Branch Current – Davao..................…………………………………………………… 20,410
41. Which of the following statements is/are true in relation to reacquired intangible rights under PFRS
3?
I. Reacquired intangible rights are recognized as an asset and determine its fair value on the basis
of the remaining contractual term of the contract when market participants would consider
potential contractual renewals when measuring its fair value.
II. Reacquired rights are amortized over the remaining contractual period.

A. I only B. II only C. I and II D. None of the choices

Page 8 of 21 0915-2303213/0908-6567516  www.resacpareview.com

Downloaded by ruzz ruzz (ruzzst@jruzz.ml)


lOMoARcPSD|12648093

First Pre-board Examination -Batch 41


ADVANCED FINANCIAL ACCOUNTING & REPORTING
42. Based on PFRS 10, it pertains to an investee (e.g., CGU or an operating segment) within a legal
structure which can also be considered as business under PFRS 3.

A. relevant entity B. silo C. separate entity D. specified assets


43. Which of the following items are both exempted from the recognition and measurement
principles of PFRS 3?
I. Asset held for sale
II. Employee benefits
III. Income taxes
IV. Indemnification assets
V. Share-based payment

A. I, II, III, IV and V B. I, II and III only C. II, III and IV only D. II, III and V only

Items 44 and 45 are based on the following information:


CC admits DD for partnership interest in his business. The balance sheet accounts of CC on November
30,20x4 prior to the admission of DD are as follows:
Debits Credits
Cash . . . . . . . . . . . . . . . . . . . . . . . . . . . . . . . . . . . . . . . . P ?
Accounts receivable . . . . . . . . . . . . . . . . . . . . . . . . . . . 96,000
Merchandise inventory . . . . . . . . . . . . . . . . . . . . . . . . . . 144,000
Accounts payable . . . . . . . . . . . . . . . . . . . . . . . . . . . . . . P 49,600
CC, capital . . . . . . . . . . . . . . . . . . . . . . . . . . . . . . . . . . . ?

It is agreed that for purposes of establishing CC’s interest, the following adjustments should be made:
1. An allowance for doubtful accounts of 2% of accounts receivable is to be established.
2. The merchandise inventory is to be valued at P160,000.
3. Prepaid expenses of P5,200 and accrued expenses of P3,200 are to be recognized.
DD is to invest cash of P113,640 to give him a one-third (1/3) interest in the firm.
44. The balance of the capital of CC before the adjustments is:
A. P227,280 C. P211,200
B. P230,120 D. P250,500
45. The total assets of the partnership after the formation is:
A. P393,720 C. P291,320
B. P340,920 D. P309,520
46. On June 30, 20x4, the balance sheet for the partnership of Williams, Brown, and Lowe, together
with their respective profit and loss ratios, is summarized as follows:

Assets, at cost . . . . . . P300,000


Williams loan. . . . . . . . . . . . P 15,000
Williams capital (20%). . . . 70,000
Brown capital (20%). . . . 65,000
Lowe capital (60%). . . . . . . 150,000
Williams has decided to retire from the partnership, and by mutual agreement the assets are to be
adjusted to their fair value of P360,000 at June 30, 20x4. It is agreed that the partnership will pay
Williams P102,000 cash for his partnership interest exclusive of his loan, which is to be repaid in full.
Goodwill is to be recorded in this transaction, as implied (total) by the excess payment to Williams.
After Williams’s retirement, what are the capital account balances of Brown and Lowe,
respectively?
A. P65,000 and P150,000 C. P73,000 and P174,000
B. P97,000 and P246,000 D. P77,000 and P186,000

47. (000’s omitted) A construction contractor has a fixed price contract for P100,000 to construct a
building (the project). The contractor’s estimate of total contract costs is P60,000. It will take two
years to construct the building.

At the end of the first year of the project (31 December 2019):
● the contractor has incurred costs of P20,000 on the contract, including P2,000 on cement
that is held offsite
● an independent surveyor certified that 28 percent of the contract work is completed

Page 9 of 21 0915-2303213/0908-6567516  www.resacpareview.com

Downloaded by ruzz ruzz (ruzzst@jruzz.ml)


lOMoARcPSD|12648093

First Pre-board Examination -Batch 41


ADVANCED FINANCIAL ACCOUNTING & REPORTING
● the site was cleared (stipulated in the contract to constitute 10 percent of the total
project), the foundation laid (stipulated as 5 percent of the total project) and the walls of
the building erected (stipulated as 14 percent of the total project)

The contractor determines that the stage of completion of the construction contract is measured
most reliably by reference to the proportion that costs incurred for work performed to date bear to
the estimated total costs. If the contractor determines that the stage of completion of the
construction contract is measured most reliably by reference to independent surveys of worked
performed, the 31 December 2019 stage of completion of the contract is:
A. 33 1/3% C. 29.00%
B. 30% D. 28.00%

48. A local partnership was considering the possibility of liquidation since one of the partners (Ding)
was insolvent. Capital balances at that time were as follows. Profits and losses were divided on a
4:2:2:2 basis, respectively.
Ding, capital………………………………………………………….. P 60,000
Laurel, capital………………………………………………………… P 67,000
Ezzard, capital………………………………………………………… P 17,000
Tillman, capital……………………………………………………….. P 96,000

Ding's creditors filed a P25,000 claim against the partnership's assets. At that time, the partnership
held assets reported at P360,000 and liabilities of P120,000. If the assets could be sold for P228,000,
what is the minimum amount that Ding's creditors would have received?
A. P -0- C. P36,000
B. P 2,500 D. P38,250
49. A construction contract has a fixed price contract for P100,000 to construct a building of a design
that has never before been constructed and using materials that have never before been used in
the construction of building (the project).
The contractor began construction of the building in 2019 and expects that construction will take
at least five years. In 2019 the contractor incurred P5,000 contract costs on the project.
At the end of 2019 the contractor cannot estimate the outcome of the contract with sufficient
reliability to estimate the project’s percentage of completion (i.e., because of the uncertainties
arising from the new design and new materials the entity cannot estimate total expected contract
costs with sufficient reliability). It is highly likely that the contract price will be received from the
customer.
At the end of 2019 the contractor must recognize revenue of:
A. Nil or zero C. P100,000
B. P 5,000 D. Incomplete data
50. In all cases of dissolution, the partnership assets and liabilities at date of dissolution may need to
be revalued to their fair values. Any revaluation increase or decrease is

A. allocated only to the partners ceasing to be associated with the partnership.


B. allocated only to the partners existing after the dissolution.
C. allocated to all existing partners as at the date of dissolution.
D. no revaluation shall be made.

Items 51 to 54 are based on the following information:


On January 1, 2019, NT Company exchanges 15,000 shares of its common stock for all of the assets
and liabilities of OTG. Inc. Each of NT’s shares has a P4 par value and a P50 fair value. The fair value of
the stock exchanged in the acquisition was considered equal to OTG’s fair value. NT also paid P25,000
in stock registration and issuance costs in connection with the merger.
Several of OTG’s accounts have fair values that differ from their book values on this date:

Book Values Fair Values


Receivables . . . . . . . . . . . . . . . . . . . . . . . . . . . . . . . . . . P65,000 P63,000
Trademarks . . . . . . . . . . . . . . . . . . . . . . . . . . . . . . . . . . 95,000 225,000
Record music catalog . . . . . . . . . . . . . . . . . . . . . . . . . . 60,000 180,000
In-process research and development . . . . . . . . . . . . 0 200,000
Notes payable . . . . . . . . . . . . . . . . . . . . . . . . . . . . . . . . 50,000 45,000
Pre-combination January 1, 2019, book values for the two companies are as follows:
NT OTG
Cash . . . . . . . . . . . . . . . . . . . . . . . . . . . . . . . . . . . . . . . . . . . P 60,000 P 29,000

Page 10 of 21 0915-2303213/0908-6567516  www.resacpareview.com

Downloaded by ruzz ruzz (ruzzst@jruzz.ml)


lOMoARcPSD|12648093

First Pre-board Examination -Batch 41


ADVANCED FINANCIAL ACCOUNTING & REPORTING
Receivables . . . . . . . . . . . . . . . . . . . . . . . . . . . . . . . . . . . . 150,000 65,000
Trademarks . . . . . . . . . . . . . . . . . . . . . . . . . . . . . . . . . . . . 400,000 95,000
Record music catalog . . . . . . . . . . . . . . . . . . . . . . . . . . 840,000 60,000
Equipment (net) . . . . . . . . . . . . . . . . . . . . . . . . . . . . . . . . 320,000 105,000
Totals . . . . . . . . . . . . . . . . . . . . . . . . . . . . . . . . . . . . . . . . P1,770,000 P354,000
Accounts payable . . . . . . . . . . . . . . . . . . . . . . . . . . . . . . P 110,000 P 34,000
Notes payable . . . . . . . . . . . . . . . . . . . . . . . . . . . . . . . . . . . 370,000 50,000
Common stock . . . . . . . . . . . . . . . . . . . . . . . . . . . . . . . . 400,000 50,000
Additional paid-in capital . . . . . . . . . . . . . . . . . . . . . . . . 30,000 30,000
Retained earnings . . . . . . . . . . . . . . . . . . . . . . . . . . . . . . 860,000 190,000
Totals . . . . . . . . . . . . . . . . . . . . . . . . . . . . . . . . . . . . . . . . . . . P1,770,000 P354,000

Assume that this combination is a statutory merger so that OTG’s accounts will be transferred to the
records of NT. OTG will be dissolved and will no longer exist as a legal entity. Immediately the business
combination using the acquisition method, determine:

51. The total assets amounted to:


A. P2,124,000 C. P2,574,000
B. P2,547,000 D. P2,599,000
52. The common stock amounted to:
A. P 50,000 C. P450,000
B. P400,000 D. P460,000
53. The additional paid-in capital amounted to:
A. P 30,000 C. P695,000
B. P 60,000 D. P720,000
54. The retained earnings amounted to:
A. P190,000 C. P 860,000
B. P835,000 D. P1,050,000
Items 55 to 57 are based on the following information:
Carrying Value Fair Value
Cash . . . . . . . . . . . . . . . . . . . . . . P 20,000 P 20,000
Accounts Receivable . . . . . . . . 45,000 30,000
Inventory . . . . . . . . . . . . . . . . . . 60,000 35,000
Land . . . . . . . . . . . . . . . . . . . . . . 75,000 70,000
Building (net) . . . . . . . . . . . . . . . 180,000 100,000
Equipment (net) . . . . . . . . . . . . 170,000 80,000
Total . . . . . . . . . . . . . . . . . . . . . . P 550,000 P335,000

Orville Company recently petitioned for bankruptcy and is now in the process of preparing a
statement of affairs. The carrying values and estimated fair values of the assets of Orville Company
are as follows:
Debts of Orville are as follows:
Accounts payable . . . . . . . . . . . . . . . . . . . . . . . . . . . . . . . . . . P 60,000
Wages Payable(all have priority) . . . . . . . . . . . . . . . . . . . . . . 10,000
Taxes payable . . . . . . . . . . . . . . . . . . . . . . . . . . . . . . . . . . . . . 10,000
Notes payable (secured by receivable and inventory). . . 120,000
Interest on Notes Payable . . . . . . . . . . . . . . . . . . . . . . . . . . . . 6,000
Bonds Payable (secured by land and building) . . . . . . . . . 150,000
Interest on bonds Payable . . . . . . . . . . . . . . . . . . . . . . . . . . . 7,000
Total . . . . . . . . . . . . . . . . . . . . . . . . . . . . . . . . . . . . . . . . . . . . . . P 363,000

55. What is the total amount of unsecured claims?


A. P 93,000 C. P121,000
B. P113,000 D. P126,000

56. What estimated amount will be available for general unsecured creditors upon liquidation?
A. P28,000 C. P113,000
B. P93,000 D. P121,000

57. What is the estimated dividend percentage?


A. 23% C. 77%

Page 11 of 21 0915-2303213/0908-6567516  www.resacpareview.com

Downloaded by ruzz ruzz (ruzzst@jruzz.ml)


lOMoARcPSD|12648093

First Pre-board Examination -Batch 41


ADVANCED FINANCIAL ACCOUNTING & REPORTING
B. 93% D. 68%
58. Contractor ROMINA enters into a contract with the Owner to undertake the construction of a
state-of-the-art commercial building. The commercial building is highly customized to the owner’s
specifications and changes to these specifications by the owner are expected over the contract
term. Non-refundable, monthly progress payments are required in order to finance the contract.
The Owner can cancel the contract at any time (with a termination penalty) and any work in
progress shall be the property of the owner. As a result, another entity would not need to re-
perform the tasks performed to date. The physical possession and title of the building do not pass
until completion of the contract.

How should the contractor recognize revenue?


A. Point in time B. Over time C. POC D. Any of the choices
59. The branch account on the home office books of Block and Bell, Inc., and the home office
accounts on the branch books on January 31, 20x7, are as follows:

Beverly Hills Branch


20x7 Debit 20x7 Credit
Jan.1 Balance 50,615 Jan.20 Cash received from
branch 14,000
16 Merchandise 22,600 Remittance received
shipments from the branch
customer in
settlement of 65
branch account
31 Expenses chargeable
to branch 215
Home Office
20x7 Debit 20x7 Credit
Jan.10 Uncollectible Jan.1 Balance 28,415
account written-off 1,200
20 Cash remittance to 21 Correction for
home office 14,000 income understatement
for December 310
31 Cost of merchandise sold
21,400
31 Income for January 1,440
Shipments from Home Office
20x7 Debit 20x7 Credit
Jan.31 Cost of merchandise 21,400 Jan.1 Balance 22,200
sold
31 Shipments returned 16 Shipments from home
to home office 840 Office 21,200
The following additional data are available in reconciling the accounts:
a. A P1,400 shipment of goods charged by the home office to the Beverly Hills branch was
actually sent to the Brentwood branch.
b. The goods returned by the branch are in transit and do not appear on the home office
records.
c. The branch failed to recognize expenses incurred by the home office and chargeable against
income, P215, in calculating its income for January.
d. The allowance for doubtful accounts on branch receivables is maintained by the home office.
The correct balance of the reciprocal account amounted to:
A. P59,365 C. P57,460
B. P55,525 D. None of the above
60. Which of the following statements is/are true in relation to construction costs recognition under
PFRS 15?
I. PFRS 15 requires that costs relating to (partially) satisfied performance obligations should be
expensed as incurred, i.e., it indicates that contract costs are recognized as the work to which
they relate is performed.
II. PFRS 15 allows the allocation of costs in the same proportion as revenue (i.e., based on POC),
effectively normalizing profit margins.

Page 12 of 21 0915-2303213/0908-6567516  www.resacpareview.com

Downloaded by ruzz ruzz (ruzzst@jruzz.ml)


lOMoARcPSD|12648093

First Pre-board Examination -Batch 41


ADVANCED FINANCIAL ACCOUNTING & REPORTING
A. I only B. II only C. I or II D. I and II
61. The after-closing balances of Carler Corporation’s home office and its branch at January 1, 2018
were as follows:
Home Office Branch
Cash………………………………………………………………………. P 7,000 P 2,000
Accounts receivable-net………………………................................. 10,000 3,500
Inventory……………………………………………………………......... 15,000 5,500
Plant assets-net ……………………………………..........................45,000 0,000
Branch……………………………………………………………………. 28,000 -__
Total assets……………………………………...................................... P 105,000 P 31,000
Accounts payable………………………………………….................. P 4,500 P 2,500
Other liabilities………………………………………............................. 3,000 500
Unrealized profit-branch inventory................................................. 500 -
Home office……………………………………………………........... - 28,000
Capital stock…………………………………………………................. 80,000 -
Retained earnings………………………………………....................... 17,000 -
Total equities…………………………… ............................................. P 105,000 P 31,000
A summary of the operations of the home office and branch for 2008 follows:
1. Home office sales: P100,000, including P33,000 to the branch. A standard 10% markup on
cost applies to all sales to the branch. Branch sales to its customers totaled P50,000.
2. Purchases from outside entities: home office, P50,000; branch P7,000.
3. Collections from sales: home office P98,000 (including P30,000 from branch); branch
collections, P51,000.
4. Payments on account; home office, P51,500; branch P4,000.
5. Operating expenses paid: home office, P20,000; branch, P6,000
6. Depreciation on plant assets: home office, P4,000; branch P1,000.
7. Home office operating expenses allocated to the branch, P2,000.
8. At December 31, 2008, the home office inventory is P11,000 and the branch inventory is
P6,000, of which P1,050 was acquired from outside suppliers.
The combined net income amounted to:
A. P 0 C. P21,000
B. P4,550 D. P25,550
Use the following information for 62 to 65:
Bullen Inc. acquired assets and liabilities of Vicker Inc. on January 1, 20x4. The book value and fair
value of Vicker's accounts on that date (prior to creating the combination) follow, along with the
book value of Bullen's accounts:
Item Bullen - Vicker Vicker
Book Value Book Value Fair Value
Retained Earnings1/1/x4 . . . . . . . . . . . . . . . . . P 160,000 P 240,000
Cash and receivables . . . . . . . . . . . . . . . . . . 170,000 70,000 P 70,000
Inventory . . . . . . . . . . . . . . . . . . . . . . . . . . . . 230,000 170,000 210,000
Land . . . . . . . . . . . . . . . . . . . . . . . . . . . . . . . . 280,000 220,000 240,000
Buildings (net) . . . . . . . . . . . . . . . . . . . . . . . . . . 480,000 240,000 270,000
Equipment (net) . . . . . . . . . . . . . . . . . . . . . . . . 120,000 90,000 90,000
Liabilities . . . . . . . . . . . . . . . . . . . . . . . . . . . . . . 650,000 430,000 420,000
Common Stock . . . . . . . . . . . . . . . . . . . . . . . . 360,000 80,000
Additional paid-in capital . . . . . . . . . . . . . . . . 20,000 40,000
62. Assume that Bullen issued 12,000 shares of common stock with a P5 par value and a P47 fair value
to obtain all of Vicker's outstanding stock. In this transaction how much goodwill should be
recognized?
A. P144,000 D. P60,000
B. P104,000 E. P 0
C. P 64,000
63. Assume that Bullen issued 12,000 shares of common stock with a P5 par value and a P42 fair value
for all of the outstanding shares of Vicker. What will be the Additional Paid-In Capital and
Retained Earnings after the combination?
A. P20,000 and P160,000 D. P464,000 and P160,000
B. P20,000 and P260,000 E. P380,000 and P260,000
C. P380,000 and P160,000

Page 13 of 21 0915-2303213/0908-6567516  www.resacpareview.com

Downloaded by ruzz ruzz (ruzzst@jruzz.ml)


lOMoARcPSD|12648093

First Pre-board Examination -Batch 41


ADVANCED FINANCIAL ACCOUNTING & REPORTING
64. Assume that Bullen issued preferred stock with a par value of P240,000 and a fair value of
P500,000 for all of the net assets of Vicker in a business combination. What will be the balance in
the Inventory and Land accounts after the business combination?
A. P440,000, P496,000 D. P402,000, P520,000
B. P440,000, P520,000 E. P427,000, P510,000
C. P425,000, P505,000
65. Assume that Bullen paid a total of P480,000 in cash for all of the shares of Vicker. In addition,
Bullen paid P35,000 to a group of attorneys for their work in arranging the combination to be
accounted for as an acquisition. What will be the balance in goodwill?
A. P 0 C. P35,000
B. P20,000 D. P55,000

Items 66 to 70 are based on the following information:


In 2019, Chicago Construction began work on a three-year construction project to build a new
performing arts complex (the PAC). The PAC contract price is P150 million. Chicago recognizes
revenue on this contract over time according to percentage of completion. At the end of 2019, the
following financial statement information indicates the results to date for the PAC (missing items
denoted by letter):

Income Statement/Statement of Comprehensive Income:


Revenue P (w) million
Cost of construction __ 35 million
Gross profit P (x) million

Balance Sheet/Statement of Financial Position:


Accounts receivable from construction billings P 14 million
Construction in progress P 50 million
Less: Billings on construction __(y) million
Net billings in excess of construction in progress P (z) million

Statement of Cash Flow:


Cash collections P 46 million
Compute the following:
66. Total revenue recognized during 2019 (w):
A. P 35 million C. P100 million
B. P 50 million D. P150 million
67. Gross profit recognized during 2019 (x):
A. P 15 million C. P 46 million
B. P 35 million D. P 50 million
68. Billings on construction (y):
A. P 14 million C. P 50 million
B. P 46 million D. P 60 million
69. Net billings in excess of construction in progress (z):
A. P 10 million C. P 50 million
B. P 15 million D. P 60 million
70. Calculate the percentage of PAC that was completed during 2019:
A. Zero C. 65 %
B. 33 1/3% D. 76 / 3%

END of EXAMINATION -

*Faith may be defined briefly as an illogical belief in the occurrence of the impossible. *
*Faith is a higher faculty than reason. *

Page 14 of 21 0915-2303213/0908-6567516  www.resacpareview.com

Downloaded by ruzz ruzz (ruzzst@jruzz.ml)


lOMoARcPSD|12648093

First Pre-board Examination -Batch 41


ADVANCED FINANCIAL ACCOUNTING & REPORTING
Ta

1. D
100% 40% 60%
Billed Price Allowance Cost
Merchandise inventory, 1/1/05 32,000
Shipments *60,000 36,000 *24,000
Cost of goods available for sale 56,000
Less: MI, 3/31/05 (25,000 x 40%) 10,000
Overvaluation of CGS/RPBSales 46,000
*36,000 cost / 60% = 60,000 x 40% = 24,000. (Note: Markup is based on billed price)
2. D
INTERESTS PAYMENTS______
P Q R P Q R Total
Balances before realization
Loans………………….. P 6,000 P(10,000)
Capital………………... 24,000 P 36,000 60,000
Total interests………... P 30,000 P 36,000 P 50,000
Divided by: P&L ratio…… 3/10 3/10 4/10
Loss absorption abilities… P100,000 P120,000 P125,000
Priority I……………………… - - ( 5,000) P2,000 P2,000
P100,000 P120,000 P120,000
Priority II…………………… - (20,000) (20,000) P6,000 8,000 14,000
P100,000 P100,000 P100,000 P – P6,000 P10,000 P16,000
3. D
Dennis Lily Total
Capital before realization 120,000 80,000 200,000
Reduction in capital (3:2) ( 84,000) ( 56,000) (140,000)
Payment to partners 36,000 24,000 60,000*
*Payment to partners:
Cash, beginning………………………………………………………………………………P100,000
Proceeds……………………………………………………………………………………….. 60,000
Payment of liabilities – to be conservative – it should be in full……………………..( 100,000)
Payment to partners…………………………………………………………………………..P 60,000
4. D
Dennis Lily Total__
Capital before realization – refer to no. 32 84,000 56,000 140,000
Reduction in capital (3:2) (78,000) ( 52,000) (130,000)
Payment to partners 6,000 4,000 10,000*
*since cash was fully distributed last month, only the proceeds of P10,000 for the second
remains to be distributed.
5. A
Gross collection (P15,000 x 70% x 80%) P 8,400
Less: Cash discount taken by customer (P8,400 x 2%) __168
Net collection P 8,232
Less Charges:
Expenses P 800
Commission (P8,400 x 15%) _1,260 __2,060
Due to Consignor P 6,172
Less: Advances _6,000
Amount remitted P 172
6. C
Charges Related to
Total Consignment Inventory on
Charges Sales Consignment
(100%) (70%) (30%)
Consignor’s charges:
Cost P10,000 P 7,000 P 3,000
Freight 120 84 36
Consignee’s charges:
Expenses 800 800
Commission (15% x P10,500) 1,575 1,575
Cash discount (P10,500 x 80% x 2%) 168 168 ______
Total P12,663 P 9,627 P 3,036
Sales price (70% x P15,000) _10,500_
Profit on Consignment P 873

Page 15 of 21 0915-2303213/0908-6567516  www.resacpareview.com

Downloaded by ruzz ruzz (ruzzst@jruzz.ml)


lOMoARcPSD|12648093

First Pre-board Examination -Batch 41


ADVANCED FINANCIAL ACCOUNTING & REPORTING
7. A
8. D
9. A
Net Free Assets:
(P700,000 – P300,000) + P70,000 + P230,000 = P700,000 – P140,000 = P560,000
Total Unsecured Creditors without priority:
(P400,000 – P300,000) + P600,000 = P700,000
10. D
Combined Cost of Goods Sold:
Merchandise Inventory, 12/1/2017:
Home Office, cost………………………………………………… P 3,500
Branch: Outsiders, ……………………………..........................P 300
From Home Office (P2,500 – P300)/110%................ 2,000 2,300 P 5,800
Add Purchases (P240,000 + P11,000)………………………………. 251,000
COGAS…………………………………………………………………… . P 256,800
Less: Merchandise Inventory, 12/31/2018:
Home Office, cost………………………………………………… P 3,000
Branch: Outsiders………………………………………………….P 150
From Home Office (P1,800 – P150)/110%............... 1,500 1,650 4,650
Cost of Goods Sold…………………………………………………… P 252,150
11. B
Sales………………………………………………………………………………………. P84,000
Less: Cost of good sold……………………………………………………………. 48,000
Operating expenses…………………………………………………………. 12,000
Interest (expense) paid to bank…………………………………………… 2,400
Net Income……………………………………………………………………………… P21,600
Salary allocations to partner’s is considered as a distribution (or allocation) of net income
rather than as a determinant of net income. In other words, salaries to partners are not
treated as an expense in computing net income.
Partner’s withdrawal affects capital balance but not net income.
12. D
13. A
14. A
15. D
16. D
17. B (Correction: Inventory – P120,500)
OCTOBER RAL TOM VIC TOTAL
BALANCES BEFORE REALIZATION:
LOANS (TO) FROM (15,000) (15,000)
CAPITALS 43,600 150,000 45,400 239,000
TOTAL INTEREST FOR OCTOBER 28,600 150,000 45,400 224,000
REDUCTION IN INTEREST (57,180) (95,300) (38,120) (190,400)
BALANCES ( 28,580) 54,700 7,280 *33,400
POSSIBLE LOSS DUE TO INSOLVENCY (5:2) 28,580 (20,414) (8,166) - 0-
BALANCES 34,286 (886) 33,400
POSSIBLE LOSS DUE TO INSOLVENCY __( 886) __886 -0-
BALANCES _33,400 _33,400
*OCTOBER: PAYMENT TO PARTNERS, COMPUTED AS FOLLOWS:
CASH, BEGINNING……………………………………………………………………P 21,000
PROCEED S (P40,000 + P50,000 + P60,000).…………………………….. 150,000
PAYMENT OF BANK NOTE (P50,000 + P600)…………………………………( 50,600)
PAYMENT OF LIQUIDATION EXPENSES…………………………………………… ( 2,000)
PAYMENT OF ACCOUNTS PAYABLE…………………………………………………( 80,000)
CASH WITHHELD……………………………………………………………………… ( 5,000)
PAYMENT TO PARTNERS……………………………………………………………..P 33,400

18. A
NOVEMBER RAL TOM VIC TOTAL
TOTAL INTERESTS FOR OCTOBER 28,600 150,000 45,400 224,000
LESS: PAYMENTS _______ (33,400) _______ (33,400)
TOTAL INTEREST 28,600 116,600 45,400 190,600
REDUCTION IN INTEREST (38,520) (64,200) (25,680) (128,400)
BALANCES ( 9,920) 52,400 9,720 *52,200
POSSIBLE LOSS DUE TO INSOLVENCY (5:2) 9,920 (7,086) (2,834) - 0-
BALANCES 45,314 6,886 52,200

Page 16 of 21 0915-2303213/0908-6567516  www.resacpareview.com

Downloaded by ruzz ruzz (ruzzst@jruzz.ml)


lOMoARcPSD|12648093

First Pre-board Examination -Batch 41


ADVANCED FINANCIAL ACCOUNTING & REPORTING
*NOVEMBER: PAYMENT TO PARTNERS, COMPUTED AS FOLLOWS:
CASH, BEGINNING (WITHHELD OF LAST MONTH)…..……………………………P 5,000
PROCEEDS (P38,000 + P10,000)….………………………………………….. 48,000
PAYMENT OF LIQUIDATION EXPENSES…………………………………………… ( 800)
PAYMENT TO PARTNERS……………………………………………………………..P 52,200

19. D
Branch A Branch B
Assets:
Inventory, January 1 P 21,000 P 19,000
Imprest branch fund 2,000 1,500
Accounts receivable, January 1 55,000 43,500
Total Assets P 78,000 P 64,000
Less: Liabilities -0- -0-
Home Office Current Account P 78,000 P 64,000
20. B
Branch A Branch B
Assets:
Inventory, December 31 P 19,000 P 12,000
Imprest branch fund 2,000 1,500
Accounts receivable, December 31 70,000 53,500
Total Assets P 91,000 P 67,000
Less: Liabilities -0- -0-
Home Office Current Account P 91,000 P 67,000

21. D - incidentally, the entry in the books of the branch would be as follows:
Profit and loss summary ………………………………………………………… xxx
Home Office Current……………………………………………………. Xxx

22. C
23. C
24. C
25. D
Total capital of the new partnership (refer to No. 24) P 296,875
Multiply by RR’s interest 20%
Cash to be invested by RR P 59,375
26. A
OO PP Total
(60%) (40%)
Unadjusted capital balances P133,000 P108,000 P241,000
Adjustments:
Allowance for bad debts ( 2,700) ( 1,800) ( 4,500)
Inventories 3,000 2,000 5,000
Accrued expenses ( 2,400) ( 1,600) ( 4,000)
Adjusted capital balances P130,900 P106,600 P237,500

Total capital before the formation of the new partnership (see above) P 237,500
Divide by the total percentage share of OO and PP (50% + 30%) 80%
Total capital of the partnership after the admission of RR P 296,875

27. A
Agreed Capital Contributed Capital Settlement
OO P148,437.50 (50% x P296,875) P 130,900 P 17,537.50
PP 89,062.50 (30% x P296,875) 106,600 (17,537.50)
Therefore, OO will pay PP P17,537.50
28. A
29. D - the amount of P40,000 Is the nearest answer (refer to entry in No. 2)
November 1, 20x4: Date of Opening/Franchise Opens: - Rights to trade name....(to record
revenue from delivery of franchise rights – point in time/right of use)
Unearned Franchise Revenue ........................... . . . . . . . . . . 41,555
Franchise revenue . . . . . . . . . . . . . . . . . . . . . . . . . . . . . . . . . 41,555

Franchises often include a license (right of use-point in time), as well as goods and services
transferred at the start of the franchise as well as over the life (right of access-over time) of the
franchise.

Page 17 of 21 0915-2303213/0908-6567516  www.resacpareview.com

Downloaded by ruzz ruzz (ruzzst@jruzz.ml)


lOMoARcPSD|12648093

First Pre-board Examination -Batch 41


ADVANCED FINANCIAL ACCOUNTING & REPORTING
A license is said to transfer a right of use if the seller’s activities during the license period are not
expected to affect the intellectual property being licensed to the customer. In that case
revenue is recognized at the start of the license period, that is, when the right is transferred.
30. A – nearest amount for unearned service revenue.
August 1, 20x5: Date of Signing:
Cash. ....................................................... . . . . . . . . . . . . . . . . 40,000
Notes receivable (P30,000 x 2)................................................ 60,000
Unearned Interest Income/Discount on Notes Receivable 6,502
Unearned franchise revenue............................................... 41,555
Unearned service revenue – training services..................... 11,947
Unearned sales revenue – equipment................................ 39,996

Cash/down-payment..............................................................P 40,000
PV of Installment payment for two (2) periods:
P30,000 x 1.78326 (PV of an annuity of P1 for 2 periods) 53,498
Total............................................................................................P 93,498
Amount allocated to:
Rights to trade name: P93,498 x (40,000/90,000)............P 41,555
Training services: P93,498 x (11,500/90,000).................... 11,947
Equipment: P93,498 x (38,500/90,000)............................ 39,996
Total.....................................................................................P 93,498

Recognition of Franchise Rights Revenue Over Time


Depending on the economic substance of the rights, the franchisor may be providing access
to the right (over time) rather than transferring control of the franchise rights. In this case, the
franchise revenue is recognized over time, rather than at a point in time (August 1, 20x5),
therefore, the P11,500 is unearned service revenue (note: not as a unearned franchise
revenue in contrast to PAS 18)
31. D - (15 x P850)  (P12,750  .06)  P300  P390 = P11,295, or
Sales (P850 x 15)) P 12,750
Less Charges:
Commission (6% x P12,750) 765
Advertising 300
Delivery and installation 390
Remittance P 11,295
32. D
Charges Related to
Total Consignment Inventory on
Charges Sales Consignment
(20) (15) (5)
Consignor’s charges:
Cost, P350 per set P 7,000 P 5,250 P 1,750
Freight, P1,800 1,800 1,350 450
Consignee’s charges:
Commission (6% x P12,750) 765 765
Advertising 300 300
Delivery and installation 390 390 _______
Total P10,255 P 8,065 P 2,200
Sales price, P850 per set 12,750
Profit on Consignment P 4,695

33. B
34. A
35. C
36. A
37. B
38. C
Inventory of the Branch:
Shipments from home office at billed price….........................................P 37,700
x: Ending inventory %................................................................................. 60%
Ending inventory at billed price………………………………………………..P22,620
Add: Freight (P1,300 x 60%)……………………………………………………... 780
P23,400
Or, P39,000 x 60% = P23,400

Page 18 of 21 0915-2303213/0908-6567516  www.resacpareview.com

Downloaded by ruzz ruzz (ruzzst@jruzz.ml)


lOMoARcPSD|12648093

First Pre-board Examination -Batch 41


ADVANCED FINANCIAL ACCOUNTING & REPORTING
39. B
Inventory in the published balance sheet, at cost
Shipments at cost………………………………….........................................P 32,500
x: Ending inventory %.................................................................................. 60%
Ending inventory at billed price…………………………………………….....P19,500
Add: Freight (P1,300 x 60%)…………………………………………………...... 780
P 20,280
40. C
Home Office Books Davao Branch Baguio Branch
Davao Branch… 39,000 SFHO…………….37,700
STB, cost……. 32,500 Freight-in………. 1,300
Unrealized profit 5,200 HOC………….. 39,000
Cash (freight)…. 1,300
BC – Baguio……19,630 HOC……………….20,150 SFHO……… 18,850
Excess freight… 520 SFHO(50%)… 18,850 Freight-in.. 780
BC-Davao……. 20,150 Freight-in (50%) 650 HOC…… 19,630
Cash…………… 650

41. B
42. B
43. C
44. C
Total partnership capital (P113,640/1/3) P 340,920
Less DD’s capital 113,640
CC’s capital after adjustments P 227,280
Adjustments made:
Allowance for doubtful account (2% x P96,000) 1,920
Merchandise inventory ( 16,000)
Prepaid expenses ( 5,200)
Accrued expenses 3,200
CC’s capital before adjustments P 211,200
45. A
Assets invested by CC:
Cash:
Capital P211,200
Add Accounts payable 49,600
Total assets (excluding cash) P260,800
Less Noncash assets (96,000 + P144,000) 240,000 P20,800
Accounts receivable (96,000 – P1,920) 94,080
Merchandise inventory 160,000
Prepaid expenses 5,200 P 280,080
Cash invested by DD 113,640
Total assets of the partnership P 393,720
46. B
Amount paid P 102,000
Less: Book value of Williams
P70,000 + (P360,000 – P300,000) x 20% 82,000
Partial goodwill/revaluation adjustment P 20,000
Capitalized at P&L of Dixon 20%
Goodwill/revaluation P100,000
Brown: P65,000 + (P60,000 x 20%) + (P100,000 x 20%) P 97,000
Lowe: P150,000 + (P60,000 x 60%) + (P100,000 x 60%) P246,000
47. D - At 31 December 2019 the stage of completion of the contract is 28 percent determined
by the independent surveyor.
48. B
Ding Laurel Ezzard Tillman Total
Capital before realization 60,000 67,000 17,000 96,000 240,000
Loss on sale (4:2:2:2) (52,800) ( 26,400) (26,400) (26,400) (132,000)
7,200 40,600 ( 9,400) 69,600 108,000
Possible insolvency loss (4:2:2) ( 4,700) ( 2,350) ( 9,400) ( 2,350) -0-
Safe payments 2,500 38,250 0 67,250 108,000
49. . B – Cost recovery method (Zero-profit approach)/Point in Time should be applied since the
outcome of the construction contract cannot be reliably measurable. At the end of 2019
the contractor must recognized only to the extent of recoverable contract costs incurred
(i.e., P5,000 contract revenue and P5,000 construction costs/expenses).

Page 19 of 21 0915-2303213/0908-6567516  www.resacpareview.com

Downloaded by ruzz ruzz (ruzzst@jruzz.ml)


lOMoARcPSD|12648093

First Pre-board Examination -Batch 41


ADVANCED FINANCIAL ACCOUNTING & REPORTING
50. C
51. C
In accounting for the combination of NT and OTG, the fair value of the acquisition is
allocated to each identifiable asset and liability acquired with any remaining excess
attributed to goodwill.
Consideration transferred (shares issued) P750,000
Less: Fair value of net assets acquired:
Cash P29,000
Receivables 63,000
Trademarks 225,000
Record music catalog 180,000
In-process R&D 200,000
Equipment 105,000
Accounts payable (34,000)
Notes payable (45,000) 723,000
Goodwill P 27,000

Entry by NT to record combination with OTG:


Cash 29,000
Receivables 63,000
Trademarks 225,000
Record Music Catalog 180,000
Capitalized R&D 200,000
Equipment 105,000
Goodwill 27,000
Accounts Payable 34,000
Notes Payable 45,000
Common Stock (NewTune par value) 60,000
PIC - par 690,000
(To record merger with OTG at fair value)

PIC - par 25,000


Cash 25,000
(Stock issue costs incurred)

Post-Combination Balance Sheet:


Assets Liabilities and Owners’ Equity
Cash P 64,000 Accounts payable P 144,000
Receivables 213,000 Notes payable ___415,000
Trademarks 625,000 Total liabilities P 559,000
Record music catalog 1,020,000
Capitalized R&D 200,000 Common stock 460,000
Equipment 425,000 Paid-in capital - par 695,000
Goodwill 27,000 Retained earnings 860,000
Total P2,574,000 Total P2,574,000

52. D – refer to No. 51


53. C – refer to No. 51
54. C – refer to No. 51
55. C – P60,000 + [(P120,000 + P6,000) – (P30,000 + P35,000) = P121,000

56. B - P20,000 + P80,000 + [P170,000 – (P150,000 + P7,000)] = P113,000 – (P10,000 + P10,000)


= P93,000
Note: The lowest priority is given to claims by General Unsecured Creditors (i.e., without priority). These
creditors are paid only after secured creditors and unsecured creditors with priority are satisfied to the
extent of any legal limits. Often the general unsecured creditors receive less than the full amount of their
claim. The amounts to be paid to these creditors are usually stated as a percentage of total claim, such
as 77 cents per peso (refer to No. 19), or whatever the specific percentage is. The payment to general
unsecured creditors is often termed a “dividend”.

57. C – P93,000/P121,000 = 77% rounded. Refer to “Note” in No. 12


58. B

Page 20 of 21 0915-2303213/0908-6567516  www.resacpareview.com

Downloaded by ruzz ruzz (ruzzst@jruzz.ml)


lOMoARcPSD|12648093

First Pre-board Examination -Batch 41


ADVANCED FINANCIAL ACCOUNTING & REPORTING
59. C
HO Books - Branch Books -
Branch Home Office
Account Account
Balances before adjustments P 59,365 P 57,525
Adjustments:
Corrected branch income for January (P1,440 –
P215) 1,225
Understatement of branch paid by home office for
December 310
Expenses of branch paid by home office 215
Collection by home office of branch receivable ( 65)
Correction of branch income for January ( 215)
Merchandise transferred to Brentwood branch but
incorrectly charged by Beverly Hills branch ( 1,400)
Merchandise returns to home office in transit ( 840)
Uncollectible accounts of branch ( 1,200) _______
Corrected Balances P 57,460 P 57,460
60. A
61. D
Sales (P100,000 – P33,000 + P50,000)…………………………………………………….P 117,000
Less: Cost of goods sold:
Inventory, beg. [P15,000 + (P5,500/110%) or (P5,500 – P500)]……P20,000
Add: Purchases (P50,000 + P7,000)…………………………………… 57,000
COGAS……………………………………………………………………..P77,000
Less: Inventory, end [P11,000 + P1,050 + (P6,000- P1,050)/110%] 16,550 60,450
Gross profit………………………………………………………………………………….. P 56,550
Less: Expenses (P20,000 + P6,000 + P5,000)…………………………………………… 31,000
Combined Net income…………………………………………………………………... P 25,550

62. B – [(P47 x 12,000 shares) – (P70,000 + P210,000 + P240,000 + P270,000 + P90,000 – P420,000)
= P104,000
63. D
APIC: P20,000 + [(P42 – P5) x12,000 = P464,000
Retained earnings: P160,000, parent only
64. B
Inventory: PP230,000 + P210,000 = P440,000
Land: P280,000 + P240,000 = P520,000
65. B – [P480,000 – (P70,000 + P210,000 + P240,000 + P270,000 + P90,000 – P420,000)] = P20,000
66. B
Total revenue recognized during 2019 (w): P 50 million
CIP contains cost + gross profit = revenue, so W = P50
67. A
Gross profit recognized during 2019 (x): P50 - P35 = P15 P 15 million

68. D
Billings on construction (y): P14 + P46 = P60 P 60 million

69. A
Net billings in excess of construction in progress (z): Billings of P60 – CIP of P50 P10 million

70. B
Calculate the percentage of PAC that was completed during 2019: 33.33%
50/150 = 33.33%

Goodluck and GOD BLESS!!!


*When GOD measures a man, He puts the tape around the heart instead of the head. *
*Until you make peace with who you are, you’ll never be content with what you have. *
*Only passions, great passions, can elevate the soul to great things. *
*Most of the things worth doing in the world had been declared impossible before they were done. *
*The world belongs to the man who is wise enough to change his mind in the presence of facts. *
*There are only two things in the world to worry over; the things you can control,
and the things you can’t control. Fix the first forget the second. *

Page 21 of 21 0915-2303213/0908-6567516  www.resacpareview.com

Downloaded by ruzz ruzz (ruzzst@jruzz.ml)


lOMoARcPSD|12648093

AFAR answer key reviewer

BSA (Batangas State University)

StuDocu is not sponsored or endorsed by any college or university


Downloaded by ruzz ruzz (ruzzst@jruzz.ml)
lOMoARcPSD|12648093

ADVANCED FINANCIAL ACCOUNTING AND REPORTING

INSTRUCTIONS: Select the best answer for each of the following questions. ALL questions are compulsory and
MUST be attempted. Mark only one answer for each item on the answer sheet provided. Strictly NO ERASURES
ALLOWED. Erasures will render your examination answer sheet INVALID. Use PENCIL NO. 2 only. GOODLUCK!

1. The partnership agreement between Ken and Avery a. I only c. I or II


stipulates that Ken is to receive a 20% bonus on b. II only d. I and II
profits, with residual profit and loss to be apportioned
in the ratio 3:2, respectively. Which partner has a
greater advantage when the partnership has a profit 7. State the proper order of partnership liquidation.
and hen it incurred a loss? I. Outside creditors
a. Profit – Ken; Loss – Avery II. Owners’ interests
b. Profit – Avery; Loss – Avery III. Inside creditors
c. Profit – Ken; Loss – Ken a. I, II, III c. II, I, III
d. Profit – Avery; Loss – Ken b. III, I, II d. I, III, II

2. The estimated amount available for free assets in a 8. Under cash priority program, when all of the priorities
Statement of Affairs for a business enterprise are paid, any remaining cash distribution is
undergoing bankruptcy liquidation is equal to the a. allocated to the partners based on their respective
assets profit or loss ratios.
a. Current fair value less carrying amounts b. allocated to the partners based on the balances in
b. Carrying amounts less current fair values their capital accounts after allocation of losses.
c. Carrying amounts plus gain or less loss on c. allocated to the partners based on their pre-
realization computed priorities.
d. Carrying amounts plus loss or less gain on d. allocated to the partners based on the relative
realization values of their capital balances.

3. As suggested by Article 1787 of the Philippine Civil 9. Under the cost recovery method,
Code and relevant PFRSs, the net contributions (assets a. the initial collections on the sale are treated as
and related liabilities assumed by the partnership) of recovery of the inventory sold. Thus, no gross
the partners to the partnership are measured at profit or interest income is recognized until total
a. fair value collections from the sale equals the cost of
b. cost inventory sold.
c. discretionary amount determined by partners b. the initial collections on the sale are treated as
d. any of these recovery of the inventory sold. Thus, no gross
profit is recognized until total collections from the
sale equals the cost of inventory sold.
4. On November 1, 2014, Klaus Co. obtained franchise c. A or B
rights from “The Originals” Co. The initial franchise fee d. None of the above.
included consideration for inventory and equipment to
be delivered to Klaus. All of the necessary preparations 10. Under PAS 11 – Construction Contracts, the primary
were completed, and Klaus Co. started operations, on issue in accounting for construction contracts is
January 31, 2015. The inventory and equipment were a. the allocation of contract revenue to the
delivered to Klaus on December 1, 2014. How would accounting periods in which construction work is
“The originals” Co. recognize revenue for the supply of performed.
inventory and equipment? b. the allocation of contract costs to the accounting
a. recognize in full on November 1, 2014 periods in which construction work is performed.
b. recognize in full on December 1, 2014 c. the determination of percentage of completion.
c. recognize in full on January 31, 2015 d. A and B.
d. deferred and amortize over the franchise term e. All of the choices.
starting January 31, 2015
11. PAS 11 – Construction Contracts provides that any
5. An advance cash distribution plan is prepared expected loss on the construction contract is
a. Each time cash is distributed to partners in an a. recognized as an expense immediately.
installment liquidation b. recognized as an expense immediately as an
b. Each time a partnership asset is sold in an adjustment to the revenue already recognized.
installment liquidation c. recognized as an expense immediately adjunct to
c. To determine the order and amount of cash each the costs of construction already recognized.
partner will receive as it becomes available for d. deferred and amortized over the remaining
distribution construction period.
d. None of these
12. Which of the following appears on the statement of
6. The interest of the withdrawing, retiring, or deceased financial position of a contractor who is applying PAS
partner shall be adjusted for which of the following? 11 – Construction Contracts?
I. His share of any profit or loss up to the date of his a. Construction in progress as current asset.
withdrawal, retirement or death, if he withdraws, b. Progress billings as current liability.
retires or dies during the year c. Amount of due from (due to) customers for
II. His share of any revaluation gains or losses as at contract work.
the date of his withdrawal, retirement, or death d. Any of the choices.

Page 1 of 8 www.prtc.com.ph AFAR

Downloaded by ruzz ruzz (ruzzst@jruzz.ml)


lOMoARcPSD|12648093

TeamPRTC

13. The realization of income on installment sales II. initial services required by the franchise agreement
transactions involves are substantially performed.
a. Recognition of the difference between the cash III. no other material conditions or obligations exist.
collected on installment sales and the cash
expenses incurred a. I, II and III c. I and II only
b. Deferring the net income related to installment b. II and III only d. I and III only
sales and recognizing the income as cash is
collected 20. If franchise rights are repossessed and the franchisor
c. Deferring gross profit while recognizing operating refunds the consideration received,
or financial expenses in the period incurred I. the original franchise sale is canceled. “Gain or
d. Deferring gross profit and all additional expenses loss” from cancellation may arise after
related to installment sales until cash is collected derecognition of account balances associated with
the franchise cancelled.
14. In selecting an accounting method for a newly II. the transaction shall not be regarded as a sale
contracted long-term construction project, the principal cancellation. However, impairment loss may arise
factor to be considered should be from forfeiture of collectibles.
a. The terms of payment a. I only c. I or II
b. The nature of the contractor’s technical facilities b. II only d. I and II
used in construction
c. The method commonly used by the contractor for 21. Which of the following is an inventory account of a
other long-term construction contracts manufacturer but not of a merchandiser?
d. The degree to which a reliable estimate of the a. Cost of goods manufactured
costs to complete and extent of progress toward b. Merchandise Inventory
completion is practicable c. Work in process inventory
d. Direct labor
15. SMDC Construction Company’s project extend over a. Cost of goods sold
several years and collection of receivables is b. Manufacturing overhead applied
reasonably certain. Each project has a contract that c. Direct materials used
specifies a price and the rights and obligations of all d. Finished goods inventory
parties. Both the contractor and the customer are
expected to fulfill their contractual
a. Materials inventory
obligations on each project. Reliable
b. Direct labor
estimates can be made of the extent of
c. Manufacturing overhead
progress and costs to complete each
d. Selling expenses
project. The method that SMDC must
22. Cost of goods manufactured is used to compute
use to account for construction revenue is
a. Installment sales method
b. Percentage- of- completion method 23. Which of the following is a period cost?
c. Completed –contract method
d. Cost recovery method 24. Job order costing would be an appropriate system to
account for the manufacture of
16. One of the more popular input measures used to a. Aircraft
25. b. Matches A
determine the progress toward completion in the
percentage- of-completion method is c. Zippers
a. Revenue-percentage basis d. Cardboard boxes
b. Cost-percentage basis written order sent to inform the purchasing
c. Progress completion basis department of a need for materials is called a
d. Cost –to- cost basis a. Purchase order
26. b. Purchase requisition U
17. The theoretical support for using the percentage- of- c. Receiving report n
completion method of accounting for long-term d. Materials requisition form d
construction projects is that it e
a. Is more conservative than the cost recovery r a periodic inventory system, the purchase of
method materials is recorded in an account entitled
b. Reports a lower net income figure than the cost a. Cost of Goods Sold
recovery method 27. b. Purchases
c. More closely conforms to the cost principle c. Materials inventory
d. Produces a realistic matching of expenses with d. Work in Process Inventory
revenues The total of the materials subsidiary ledger inventory cards
must be equal to the amount in the following account
18. It is the one-off payment made by the franchisee to
a. Cost of Goods Sold
the franchisor to obtain the franchise right. 28. b. Purchases
a. Initial franchise fee
c. Materials Inventory
b. Continuing franchise fee
d. Work in Process Inventory
c. Fixer’s fee
Which of the following is usually prepared daily by
d. Any of the choices
employees for each job worked on?
19. There is substantial performance when: a. Job time tickets
I. the franchisor has no remaining obligation or intent b. Time card
to refund any cash received or forgive any unpaid c. Punch card
notes or receivables. d. Cost control card

Page 2 of 8 www.prtc.com.ph AFAR

Downloaded by ruzz ruzz (ruzzst@jruzz.ml)


lOMoARcPSD|12648093

TeamPRTC

29. Under a perpetual inventory system, the purchase of a. CLAIRE, P1,045,080; DAISY, P376,200; & ELSIE,
materials is recorded in an account entitled P221,400
a. Cost of Goods Sold b. CLAIRE, P1,161,200; DAISY, P418,000; & ELSIE,
30. b. Purchases P246,000
c. Materials inventory c. CLAIRE, P1,987,500; DAISY, P189,000; & ELSIE,
d. Work in process inventory P217,500
Factory worker fringe benefit costs are usually charged to d. CLAIRE, P1,095,120; DAISY, P547,560; & ELSIE,
P182,520
a. Work in process Inventory
b. Direct labor
c. Administrative expenses On January 1, 2015, FRIDA and GLACE formed a
d. Factory overhead partnership by contributing cash of P405,000 and
The following condensed balance sheet is prepared for
P270,000, respectively. On February 1 2015, Partner
QUIEL and ROGER, who share profits and losses in the FRIDA contributed an additional P135,000 cash to the
ratio of 60:40, respectively:
partnership and on August 1, 2014 Partner FRIDA made a
Other assets P 405,000 Accounts P108,000 permanent withdrawal of P67,500. On May 1, 2015,
payable
Partner GLACE contributed machinery with a fair market
Quiel, loan 18,000 Quiel, capital 175,500 value of P90,000 and a net book value of P75,000 when
Roger, capital 139,500
contributed. On November 1, 2015 Partner GLACE
Total P 423,000 Total P 423,000 contributed an additional P45,000 cash to the partnership.
Both partners withdrew one-fourth of their salary
31. The partners have decided to liquidate the partnership. allowances in 2015.
If the other assets are sold for P346,500, what amount
of the available cash should be distributed to QUIEL? The partnership reported a net income of P257,400 in
a. P136,000 c. P122,400 2014 and the profit and loss agreement are as follows:
b. P156,000 d. P195,000 a. Interest at 6% is allowed on average capital
balances;
b. Salaries of P2,700 per month to each partner;
On January 1, 2014, the partners SELYA, TESSA, and c. Bonus to FRIDA of 10% of net income after interest,
URSULA, who share profits and losses in the ratio of 5:3:2, salaries, and bonus; and
respectively, decided to liquidate their partnership. On this d. Balance to be divided in the ratio of 6:4 to FRIDA
date the partnership condensed balance sheet was as and GLACE, respectively.
follows:
34. Determine how the net income will be allocated to the
Cash P 45,000 Liabilities P 54,000 partners:
Other assets 225,000 Selya, capital 72,000 a. FRIDA, P160,000 and GLACE, P126,000
Tess, capital 81,000 b. FRIDA, P 180,000 and GLACE, P106,000
Ursula, capital 63,000 c. FRIDA, P170,000 and GLACE, P116,000
Total P 270,000 Total P270,000 d. FRIDA, P153,000 and GLACE, P104,400

On January 15, 2014, the first cash sale of other assets HAIDEE and ISABEL are partners sharing profits and losses
with a carrying amount of P135,000 realized P108,000. in the ratio of 60% and 40%, respectively. The partnership
Safe installment payments were made on the same date. balance sheet at August 30, 2014 follows:
32. How much cash should be distributed to each partner?
SELYA TESSA URSULA Cash P 12,150 Accounts payable P 13,500
a. P15,000 P51,000 P44,000 Other assets 119,700 Haidee, Loan 5,850
b. P40,000 P45,000 P35,000 Isabel, Loan 9,000 Haidee, capital 81,000
c. P55,000 P33,000 P22,000 Isabel, capital 40,500
d. P13,500 P45,900 P39,600 Total P 140,850 Total P140,850

CLAIRE, DAISY, and ELSIE formed the CDE Partnership on At this date, JOSIE was admitted as a partner for a
August 1, 2015, with the following assets, measured at fair consideration of P43,875 cash for a 40% interest in capital
market values, contributed by each partner: and in profits.
CLAIRE DAISY ELSIE 35. Assume JOSIE is admitted by purchase of 40% each
of the original partners’ interest, determine how the
Cash P 324,000 P108,000 P129,600
P43,875 will be apportioned to HAIDEE and ISABEL
Accounts
a. HAIDEE, P32,850 and ISABEL, P15,900
receivable 73,080 - 91,800
b. HAIDEE, P32,450 and ISABEL, P16,300
Plant, Property, &
c. HAIDEE, P29,565 and ISABEL, P14,310
Equipment (PPE) 1,620,000 340,200 -
d. HAIDEE, P32,950 and ISABEL, P15,800
A part of CLAIRE’s cash contribution, P216,000, comes
from personal borrowings. Also, the PPE of CLAIRE and
DAISY are mortgaged with the bank for P972,000 and
P72,000, respectively. The partnership is to assume
responsibility for these PPE mortgages. The partners have
agreed to share profits and losses on a 5:2:3 ratio, to
CLAIRE, DAISY, and ELSIE, respectively.
33. What is the capital balance for each partner at the
opening of business on August 1, 2015?

Page 3 of 8 www.prtc.com.ph AFAR

Downloaded by ruzz ruzz (ruzzst@jruzz.ml)


lOMoARcPSD|12648093

TeamPRTC

PRINCESS COMPANY filed a voluntary bankruptcy P1,176,000. Operating expenses (includes losses on
petition on August 15, 2013 and the statement of repossession) total to 75% of the realized gross profit.
affairs reflect the following amounts: 39. What is the net income for the year ended December
BOOK ESTIMATED 31, 2015?
CARRYING CURRENT a. P329,142 c. P 543,984
VALUE VALUE b. P546,000 d. P 279,918

Pledged with fully secured P 150,000


creditors P 185,000 BEIGE STALKS CORPORATION, which began operations on
Pledged with partially January 1, 2014, appropriately uses the installment
90,000
secured creditors 60,000 method of accounting for revenues. The following
210,000
Free Assets 160,000 information is available for the years ended December 31,
P 450,000
P 405,000 2014 and 2015:
Liabilities
2014 2015
P 35,000
Liabilities with priority Cost of installment sales P 960,000 P1,920,000
130,000
Fully secured creditors GP realized on sales made in
100,000
Partially secured creditors 2014 144,000 86,400
270,000
Unsecured creditors 2015 - 192,000
P 535,000
GPR based on cost 30% 40%
40. What is the ending balance of installments receivable
36. How much cash will be available to pay the unsecured
at December 31, 2015?
non-priority claims?
a. P2,265,600 c. P1,632,000
a. P240,000 c. P160,000
b. P1,704,000 d. P1,176,000
b. P180,000 d. P125,000

On January 1, 2014, MAXX SERVICES, INC. signed an


The following data were taken from the statement of
agreement authorizing LALLA COMPANY to operate as a
affairs of MARACLARA CORPORATION:
franchisee over a 20-year period for an initial franchise fee
Assets pledged for fully secured liabilities
of P137,500 received when the agreement was signed.
(current fair value, P75,000) P 90,000
LALLA commenced operations on July 1, 2014, at which
Assets pledged for partially secured
date all of the initial services required of MAXX SERVICES
liabilities (current fair value P52,000) 74,000
had been performed. The agreement also provides that
Free assets (current fair value , P40,000) 70,000
LALLA must pay annually to MAXX a continuing franchise
Unsecured liabilities with priority 7,000
fee equal to 5% of the revenue from the franchise. LALLA
Fully secured liabilities 30,000
COMPANY’s franchise revenue for 2014 was P1,100,000.
Partially secured liabilities 60,000
41. For th e year ended December 31, 2014, how much
Unsecured liabilities without priority 112,000
should MAXX SERVICES record as revenue from
franchise fees with respect to the LALLA account?
37. The amount that will be paid to creditors with priority
a. P192,500 c. P123,750
is:
b. P137,500 d. P 60,500
a. P7,000 c. P7,500
b. P6,000 d. P6,200
GREAT DANE, INC., franchisor, entered into a franchise
agreement with PITBULL COMPANY, franchisee, on July 1,
SILVER PLATTER COMPANY which began operations on
2015. The total franchise fees agreed upon is P550,000, of
January 2, 2014, appropriately uses the installment
which P50,000 is payable upon signing and the balance is
method of revenue recognition. The following information
to be covered by a note payable in four equal annual
pertains to the company’s operations for 2014 and 2015
installments. The direct franchise cost incurred was
2014 2015
P325,000. Indirect franchise expenses of P31,250 was also
Sales P 307,200 P460,800
paid. The relevant interest rate is 12% and the note is
Collections from
reasonably assured of collection. The agreement also
2014 sales 102,400 51,200
provides for the payment of continuing franchise fees at
2015 sales 0 153,600
4% of the franchisee’s gross sales. The franchise outlet
Accounts written off from
commences its operations on December 1, 2015 and had a
2014 sales 25,600 76,800
gross sales of P250,000 for the month.
2015 sales 0 153,600
42. Assuming the notes are interest-bearing, how much
Gross profit rates 40% 30%
net income will be reported for 2015?
a. P198,750 c. P 233,750
38. What amount should SILVER PLATTER COMPANY report
b. P 77,550 d. P 73,750
as deferred gross profit in its December 31, 2015
balance sheet?
a. P 76,800 c. P 114,688 On January 2, 2015, QUICKBUILD ERECTORS
b. P102,400 d. P 66,560 entered into contract to construct two projects. The
following data relate to the construction activities.
Project A Project B
BROWN DERBY COMPANY began operations on June 1,
Contract price P945,000 P675,000
2015. The following information are extracted from its
Cost incurred during 2015 540,000 630,000
records at year-end. Cost of installment sales, P2,991,424;
Estimated costs to complete 270,000 157,500
Cost of Regular Sales, P1,680,000. Mark-up on installment
Billings to customer 337,500 607,500
sales is 40% of cost while regular sales is 33-1/3% based
on sales. At the end of 2015, the balance of Installment
accounts receivable is P2,520,000; Accounts receivable is

Page 4 of 8 www.prtc.com.ph AFAR

Downloaded by ruzz ruzz (ruzzst@jruzz.ml)


lOMoARcPSD|12648093

TeamPRTC

43. What amount of gross profit should Assuming that the branch ending inventory acquired from
QUICKBUILD ERECTORS report in its 2015 other vendors (OV) is P73,125
income statement under the following
46. What is the net income (loss) of the branch insofar as
methods? the home office is concerned?
Percentage of Zero Profit a. P534,000 c. P315,000
Completion Method Method b. P681,750 d. (P147,750)
a. P 0 P (90,000)
b. P (112,500) P (22,500)
c. P ( 22,500) P 0 Teardrops Commercial Corp. maintains a branch in Iloilo
d. P ( 22,500) P(112,500) City. Selected balances taken from the books of Teardrops
and its Bacolod City branch as of December 31, 2015 are
as follows:
44. BEST - EVER CONSTRUCTION, INC . recognizes Home Office Branch
construction revenue and costs using the percentage of Office
completion method. During 2014, a single long-term Merchandise Inventory,
project was begun which continued through 2015. Jan 1 P 12,000 P 8,000
Information on the project follows: Purchases 150,000 30,000
2014 2015 Shipments from Home
Accounts receivable P350,000 P1,050,000 Office 93,750
Incurred costs during year 367,500 672,000 Shipments to Branch 75,000
Construction in progress 427,000 1,274,000 Branch Inventory
Billings on contract 350,000 1,470,000 Allowance 19,750
The construction accounts are at amounts t0-date. Sales 115,000 176,500
Merchandise Inventory,
What is the gross profit recognized from this long-term Dec 31 14,000 10,350
contract?
2014 2015 P4,350 of the branch's ending inventory came from
a. P 77,000 P 798,000 purchases from suppliers other than the home office.
b. 77,000 350,000
c. 59,500 448,000 47. As far as the home office is concerned, the cost of
d. 59,500 175,000 sales of the branch was:
a. P 97,120 c. P121,400
b. P102,850 d. P131,850
CIGNAL ERECTORS began operations on January 2,
2015. During the year, the company entered into a
contract with TEAM Company to construct a During the year 2015 the Bacolod Corporation bills its Iloilo
manufacturing facility. At that time CIGNAL estimated branch at 140% of cost. Goods billed at P346,500 were
that it would take five years to complete the facility at shipped to the branch. The account Allowance for
a cost of P3,937,500. The total contract price for the overvaluation has a balance of P122,400 before
construction of the facility is P5,468,750. During the adjustment. The beginning inventory of the branch from
year, the company incurred P962,500 in construction the home office at cost is P93,600; the beginning inventory
costs related to the construction project. The of the branch from outsiders is P15,200, purchases from
estimated cost to complete the contract is P2,730.000 outsiders is P130,500.
TEAM was billed and paid 30% of the contract price 48. Cost of goods available for sale of the Iloilo Branch in
subject to a 10% retention. 2015 is
a. P486,800 c. P609,200
45. Using the percentage of completion method, how much b. P623,240 d. P463,500
is the excess of Construction in Progress over Contract
Billings or Contract Billings over Construction in
Progress? GHI Company bills its Bulacan Branch for merchandise
a. P273,437 (current liability) shipments at 125% of cost. As of cut-off date, December
b. P273,437 (current asset) 31, 2015, the following data were available:
c. P437,500 (current asset) Mdse. Fr Home Mdse.
d. P437,500 (current liability) Office(at billed Purchased
prices) from Outsiders
Total
Presented below are items taken from the unadjusted trial Merchandise,
balances of NCR Company and its Manila Branch on December 1 P300,000 P120,000 P420,000
December 31, 2015: Additions to
Home Office Branch stock during
Books Books December 450,000 360,000 810,000
Shipment to branch P2,250,000 Merchandise,
AFOVOBI 749,250 December 31 420,000 150,000 570,000
Shipment from HO P2,925,000
Purchases (from OV) 1,084,500 The branch returned P15,000 worth of merchandise to the
MI, January 1 921,375 Home Office acquired at billed price.
MI, December 31 365,625 49. The amount of the allowance for overvaluation account
Sales 4,800,000 that was realized as income in view of branch sales for
Expenses 382,500 the month of December was:
a. P63,000 c. P87,500
b. b. P66,000 d. P84,000

Page 5 of 8 www.prtc.com.ph AFAR

Downloaded by ruzz ruzz (ruzzst@jruzz.ml)


lOMoARcPSD|12648093

TeamPRTC

Current assets P280,000 P 320,000


Plant assets (net) 680,000 1,280,000
Liabilities 320,000
The Batangas Corporation operates a branch in Lipa City. Common stock 64,000
The Home Office ships merchandise to the branch at 125% Additional paid-in capital 256,000
of its cost. Selected information from the December 31, Retained earnings 320,000
2015 trial balance are as follows:
Home Office Branch Office 53. To have a goodwill of P 120,000, the number of shares
Books Books to be issued by Carl Company should be
Sales P600,000 P300,000 a. 30,000 shares c. 29,000 shares
Shipments to branch 200,000 b. 30,400 shares d. 35,000 shares
Purchases 350,000
Shipments from Home
Office 250,000 On August 1, 2014, Blite Company paid P850,000 for all
Inventory, January 1 100.000 40,000 the net assets of Ong Enterprises in a transaction properly
Allowance for Overvaluation recorded as a purchase. The recorded assets and liabilities
of branch Inventory 58,000 of Ong Enterprises on August 1, 2014, follow:
Expenses 120,000 50,000 Cash P 80,000
Inventory 240,000
Inventory at December 31, 2015: Property and equipment, net 480,000
Home Office P30,000 Liabilities (180,000)
Branch Office 60,000
On August 1, 2014 it was determined that the inventory of
50. The combined net income of the home office and the Ong had a fair market value of P190,000, and the property
branch after adjustment is: and equipment (net) had a fair market value of P560,000.
a. P326,000 c. P500,000 54. What is the amount of goodwill resulting from the
b. P496,000 d. P280,000 business combination?
a. P 0 c. P200,000
b. P 20,000 d. P230,000
Quad Corporation purchases all of the net assets of
Chrome, Inc., for P320,000. Immediately prior to the
combination, Chrome’s net assets were carried on the 55. Stain Corporation is an 80%-owned subsidiary of Paint
books at P180,000, and Chrome had retained earnings of Corporation. During 2014 Stain sold merchandise that
P24,000. The fair value of Chrome’s net assets at the date cost P96,000 to Paint for P128,000. Paint's ending
of combination is P248,000. Quad Corporation had inventory at December 31, 2014 contained unrealized
retained earnings of P40,000 and no goodwill immediately profit of P6,400 from the intercompany sales. During
prior to the combination 2015 Stain sold merchandise that cost P112,000 to
51. Immediately after the combination, the combined Paint for P152,000. One-half of this remained unsold
company reports goodwill and retained earnings of: by Paint at December 31, 2015 For 2015 Paint's
Goodwill Retained Earnings separate income was P200,000 and Stain's reported
a. P 0 P 40,000 net income was P152,000.
b. P 0 P 64,000
c. P 72,000 P 40,000 The consolidated net income for 2015 will be:
d. P 72,000 P 64,000 a. P302,000 c. P310,720
b. P338,400 d. P274,500

The Carl Company will issue P10 par value common stock
for the net assets of PBA Company. The fair market value 56. P Company acquired a 90% interest in S Company in
per share of Carl’s common stock is P40. The following is 2013 at a time when S Company's book values and fair
the list of accounts of PBA Company on the date of the values were equal to one another. On January 1, 2015,
acquisition. S sold a machine with a P24,000 book value to P
Book Value Fair Market Value Company for P48,000. P depreciates the machine over
Current assets P280,000 P 320,000 10 years using the straight line method. Separate
Plant assets (net) 680,000 incomes for P and S for 2015 are as follows:
1,280,000 P Co. S. Co.
Liabilities 320,000 Sales P960,000 P560,000
Common stock 64,000 Gain on sale of 24,000
Additional paid-in capital machinery
256,000 Cost of goods sold (400,000) (152,000)
Retained earnings 320,000 Depreciation expense (240, 000) (72,000)
52. To have an income from acquisition of P120,000, the Other expenses (96,000) (240,000)
number of shares to be issued by Carl Company should Separate incomes P224,000 P120,000
be”
a. 30,000 shares c. 29,000 shares The consolidated net income for 2015 is:
b. 30,400 shares d. 35,000 shares a. P344,000 c. P310,400
b. P322,400 d. P312,560
The Carl Company will issue P10 par value common stock
for the net assets of PBA Company. The fair market value
per share of Carl’s common stock is P40. The following is RICH Corporation paid P1,125,000 for an 80% interest in
the list of accounts of PBA Company on the date of the HARD Corporation on January 1, 2015 at a price P37,500
acquisition. in excess of underlying book value. The excess was
Book Value Fair Market Value allocated P15,000 to undervalued equipment with a ten-

Page 6 of 8 www.prtc.com.ph AFAR

Downloaded by ruzz ruzz (ruzzst@jruzz.ml)


lOMoARcPSD|12648093

TeamPRTC

year remaining useful life and P22,500 to goodwill which


was not impaired during the year. During 2015, HARD 60. In Aleck’s income statement, the amount that should
Corporation paid dividend of P60,000 to RICH Corporation. be included as a foreign exchange loss
The income statements of RICH and HARD for 2015 are a. P 0 c. P 6,000
given below: b. P21,000 d. P27,000
RICH HARD
Sales P2,500,000 P1,000,000 On April 8, 2013, CALAMBA CORPORATION purchased
Cost of sales (1,250,000) (500,000) merchandise from an unaffiliated foreign company for
Depreciation 10,000 units of the foreign company’s local currency.
expense (250,000) (150,000) CALAMBA paid the bill in full on March 1, 2015 when the
Other expense (500,000) (225,000) spot rate was P0.45. The spot rate was P0.60 on April 8,
Net income P500,000 P125,000 2013 and was P0.55 on December 31, 2014.
61. For the year ended December 31, 2014, CALAMBA
should report a transaction gain of
57. Consolidated net income for 2015 is a. P1,500 c. P1,000
a. P632,125 c. P623,125 b. P 500 d. P 0
b. P263,125 d. P632,215
On December 1, 2014, a Philippine firm purchased a
P Corporation acquired 70% of the voting common speculative hedge to buy 30,000 foreign currency when
stock of S Company at a time when S Company’s book the spot rate was P1.10 and a 60 day forward rate was
values and fair values were equal. Separate incomes P1.12. The spot rate at December 31 (the company’s year-
of P Corporation and S Company for 2015 are as end was P1.25 and a 30-day forward rate was P1.13.
follows: When the speculative hedge was exercised on January 31,
P Corporation S Company 2015 the spot rate was P1.11 and a 30 day forward rate,
Sales 633,600 350,400 P1.12.
Cost of Goods Sold 384,000 192,000 62. The journal entry to record this hedge would include a
Operating expenses 115,200 96,000 debit to Contract Receivable in the amount of
Separate income from a. P33,600 c. P33,000
own operations 134,400 62,400 b. P 600 d. P 0

Intercompany sales from P to S for 2014 and 2015 are 63. The amount of foreign exchange gain/loss that would
summarized as follows: appear on the income statements of the Philippine
Cost Selling Unsold company resulting from this speculative hedge for the
Price at year- years ended 2014 and 2015 are
end a. 2014 = P300 loss; 2015 = 600 loss
Intercompany sales b. 2014 = P300 gain; 2015 = 600 gain
– 2014 240,000 374,400 30% c. 2014 = P300 loss; 2015 = 600 gain
Intercompany sales d. 2014 = P300 gain; 2015= 600 loss
– 2015 168,000 264,000 40%

58. The 2015 consolidated income statement will show From the following data, question 16 to 18 should be
cost of goods sold of answered.
a. P 310,080 c. P 384,000 Opening inventory 4,000 units
b. P 576,000 d. P 192,000 Percentage of Value
Completion
Materials 100% P1,992
On September 1, 2015 Junjun Company received an order Labor 50% 1,074
for equipment form a foreign customer for FC 300,000 Overhead 50% 846
when the Philippine peso equivalent was P96,000. Junjun Put in process 20,000 units
shipped the equipment on October 15, 2015, and billed the Materials value P12,000
customer for FC 300,000 when the Phil. Peso equivalent Labor 9,984
was P100,000. Junjun received the customer’s remittance Overhead is 100% of labor cost
in full on November 16, 2015, and sold the FC 300,000 for Units completed and transferred 21,000 units
P105,000. Units in process at the end 3,000 units
59. In its income statement for the year ended December Materials 100%
31, 2015, Junjun should report a foreign exchange Labor and overhead 60%
gain of 64. The equivalent production for material is
a. P5,000 c. P9,000 Under Average Under FIFO
b. P4,000 d. No gain no loss a. 24,000 20,000

Alecks Corporation had the following foreign currency


b. 20,000 21,000
transactions during 2015 c. 20,000 24,000
a. Merchandise was purchased from foreign supplier on d. 21,000 20,000
January 20, 2015 for the Peso equivalent of P90,000.
The invoice was paid on March 20, 2015 at the Phil.
Peso equivalent of P96,000. Sangley, Inc. manufactures a product which goes
b. On July 1, 2015, Alecks borrowed the Philippine peso
equivalent of P500,000 evidenced by a note that was through three consecutive processes, Process 1, Process
payable in the lender’s local currency on July 1, 2015. 2, and Process 3. Data for the month of September,
On December 31, 2015, the Phil. Peso equivalent of 2006 are as follows:
the principal amount and accrued interest were PROCESS1 PROCESS2 PROCESS3
P520,000 and P26,000 respectively. Interest on the Work in
note is 10% per annum.

Page 7 of 8 www.prtc.com.ph AFAR

Downloaded by ruzz ruzz (ruzzst@jruzz.ml)


lOMoARcPSD|12648093

TeamPRTC

Process, beg. P8,000 P13,000 P2,000 a. P1,245,000 c. P1,335,000


Materials added 20,000 4,000 5,000 b. P1,290,000 d. P1,380,000
Conversion 10,000 10,000 16,000
costs The following information relates to Job No. 2468, which is
Closing work in being carried out by Flexy Co. to meet a customer’s order.
process 6,000 9,000 4,000 Dept. A Dept. B
Direct materials used P5,000 P3,000
65. What was the value of the output transferred from Direct labor hours 400 200
Process 3 to the finished goods warehouse for the employed
month of September? Direct labor rate per hour P4.00 P5.00
a. P63,000 c. P67,000 Overhead rate per DL hour P4.00 P4.00
b. P65,000 d. P69,000 Adm. and other overhead 20% of full production
cost
Lego Plastics, Inc. has two joint products, ABBA and ADDA, Profit markup 25% of selling price
and uses the net realizable value method of allocating joint
costs. The total joint costs for the year 2000 amounted to 69. The selling price to the customer of Job 2468 is:
P300,000. During the year, additional processing costs a. P16,250 c. P17,333
after split-off were P160,000 for ABBA and P240,000 for b. P20,800 d. P19,500
ADDA. Lego produced 16,000 units of ABBA and 8,000
units of ADDA during the year. The selling price for ABBA is Rumors Company applied factory overhead as follows:
P20.00 and for ADDA is P50.00. Department Factory Overhead Rate
66. The portion of joint costs allocated to ADDA during the Fabricating P7.75 per Machine hour
year is Spreading 15.10 per Machine hour
a. P175,000 c. P180,000 Gossiping 2.125 per Machine hour
b. P225,000 d. P150,000
Actual machine hours are: 19,000 hours for fabricating;
Lee Company produces two products in a single operation, 27,500 hours for spreading and 5,500 hours for gossiping.
Bex and Rom. Joint production cost for June, 2014 were
P30,000. During the month, further processing costs 70. If the actual factory overhead cost for the period is
beyond the split-off point needed to convert the products P574,375, how much is over (under) applied factory
into salable form were P25,000 and P35,000 for 1,600 overhead?
units of Bex and 800 units of Rom, respectively. Bex sells a. (P11,875.00) c. (P 187.50)
for P50 per unit and Rom sells for P100 per unit. Lee uses b. (P23,562.50) d. (P76,125.00)
the net realizable method for allocating joint product costs.
67. For June, 2014, the joint cost allocated to product Bex
were
a. P20,000 c. P13,500
b. P16,500 d. P10,000

The accounting records for 2014 of Wagner Music Co.


showed the following:
Decrease in raw materials inventory P 45,000
Decrease in finished goods inventory 150,000
Raw materials purchased 1,290,000
Direct labor payroll 600,000
Factory overhead 900,000
Freight-out 135,000  - end - 

68. The cost of raw materials used for the period


amounted to

Page 8 of 8 www.prtc.com.ph AFAR

Downloaded by ruzz ruzz (ruzzst@jruzz.ml)


No. 125 Brgy. San Sebastian
Lipa City, Batangas, Philippines
Mobile : 0927 283 8234
Telephone : (043) 723 8412
Gmail : icarecpareview@gmail.com

ADVANCED FINANCIAL ACCOUNTING & REPORTING


iCARE Accountancy Review
st
1 Open Preboard Examination, Batch 3

INSTRUCTIONS: Choose the best answer among the given choices. Policies on academic
honesty are strictly enforced.

1. A summary of balance sheet for ABC Co. appears below. Partners A, B and C share
profits and losses in the ratio of 2:3:5, respectively.

Assets Liabilities and Capital


Cash 100,000 A, capital 425,000
Inventory 125,000 B, capital 400,000
marketable Securities 200,000 C, capital 200,000
Land 100,000
Building-net 500,000
Total Assets 1,025,000 Total 1,025,000

The partners agree to admit D for a 1/5 interest. The fair value of partnership land is
appraised at P200,000 and the fair value of inventory is P175,000. The assets are to
be revalued prior to the admission of D and there is a decrease of P30,000 in the
capital interest of the new partner after his admission.

The increase (decrease) in the capital interest of C after admission of D.


a. P60,000 c. P75,000
b. P90,000 d. P36,000

2. The amount of cash investment of D to acquire 1/5 interest in the partnership?


a. P301,250 c. P226,250
b. P256,250 d. P331,250

3. Which of the following statement is correct?


Statement 1: In partnership, each partner has his own owner’s capital account.

Statement 2: A debit to partner’s capital account is made whenever the partner


borrowed substantial amount from the partnership.
a. I only c. Both statements are correct
b. II only d. Both statements are incorrect

4. On January 1, 20x1, A and B formed a partnership the AB partnership. A and B


share profits and losses in a 40:60 ratio, respectively. The following are information
available on the date of formation:
A B
Cash 100,000 25,000
Inventory 300,000
Land 500,000
Building 300,000
Accounts payable 50,000

1|P a g e RFERRER/RLACO/ ATANG/PDEJESUS


No. 125 Brgy. San Sebastian
Lipa City, Batangas, Philippines
Mobile : 0927 283 8234
Telephone : (043) 723 8412
Gmail : icarecpareview@gmail.com

The accounts payable of A will be paid by partner A using his personal assets. The
land contributed by B has a mortgage liability of P150,000 which was assumed by the
partnership.

How much is the capital interest of A and B after formation?


a. P450,000 and 675,000 c. P430,000 and 645,000
b. P400,000 and 675,000 d. P615,000 and P410,000

5. Assuming the partners agree to revalue the assets, so that their capital interest will
agree to their profit and loss sharing ratio, which of the following statements is
incorrect?
a. The capital interest of A after formation is P450,000
b. The capital interest of B after formation is P675,000
c. The amount of revaluation is P50,000
d. The total capital contribution of the partners are P1,125,000

6. Which of the following statement is correct?


Statement 1: The balance of partner’s capital account represents his share in the net
of the partnership.

Statement 2: Temporary withdrawals of cash from the partnership are charged to


the partner’s drawings account, when these are made in anticipation of profit to be
earned by the partnership.
a. I only c. Both statements are correct
b. II only d. Both statements are incorrect

7. Which of the following statement is correct?


Statement 1: As a rule, the industrial partner should share in the partnership’s net
loss.
Statement 2: In the absence of a specific agreement regarding division of losses,
the existing division of profit agreement is to be followed.
a. I only c. Both statements are correct
b. II only d. Both statements are incorrect

8. M, N and O are partners in the MNO Co. Their capital contributed were: M
P100,000; N P220,000 and O P100,000.
 Partner O is to receive a bonus of 10% of net income after bonus
 Interest of 10% shall be paid on that portion of capital beginning in excess of
P200,000.
 Salaries of P20,000 and P24,000 shall be paid to partners M and O,
respectively.

Assuming the net income of the partnership is P88,000, the total profit share of O is:
a. P38,800 c. P40,095
b. P32,000 d. P43,333

9. Which of the following statement is correct?


Statement 1: Funds extended by the partners to the partnership as a loan shall be
accounted for by crediting the partner’s capital account.
Statement 2: When industry is contributed into the partnership, a memorandum
entry is prepared.
a. I only c. Both statements are correct
b. II only d. Both statements are incorrect

2|P a g e RFERRER/RLACO/ ATANG/PDEJESUS


No. 125 Brgy. San Sebastian
Lipa City, Batangas, Philippines
Mobile : 0927 283 8234
Telephone : (043) 723 8412
Gmail : icarecpareview@gmail.com

10. Which of the following statement is correct?


Statement 1: Drawing accounts are included in the computation of the ending
capital balances regardless whether the drawing account is a temporary or a
permanent withdrawals.
Statement 2: The admission of a new partner either by purchase of an existing
partner’s interest or by the investment in the partnership will result in the increase
of the total partnership assets.
a. I only c. Both statements are correct
b. II only d. Both statements are incorrect

11. A and B formed a partnership on March 1 , 20x1. A invested P250,000 and B


150,000. It was agreed that A, will be appointed as the managing partner and he
will receive a salary of P60,000 per year and also 10% bonus on the net profit after
the adjustment for the salary; the balance of the profit was to be shared in a
62.5:37.5 ratio to A and B, respectively. On December 31, 20x1, the following
accounts are available:

Accounts
Cash 140,000 Payable 120,000
Accounts Receivable 134,000 Sales 466,000
Furniture and Fixtures 90,000 A, capital 250,000
Purchases 392,000 B, capital 150,000
Sales Return and
Allowances 10,000 A, drawings (40,000)
Operating expenses 120,000 B, drawings (60,000)

On December 31, 20x1 the inventories on hand were P146,000; unused supplies
P5,000; prepaid insurance were P1,900 (the entity uses the expense method). The
accrued liabilities totaled P3,100 and the depreciation of furniture’s is 20% per year
which are not yet included in the operating expenses.

The share of A in the net income is:


a. P70,580 c. P69,080
b. P64,768 d. P67,205

12. Which of the following statement is correct?


Statement 1: The personal creditors of the general partner have priority of claim
over the creditors of the partnership pertaining to the partner’s personal assets.

Statement 2: Admission of new partner by investment in the partnership will


increase the partnership capital even under the bonus method.
a. I only c. Both statements are correct
b. II only d. Both statements are incorrect

13. A and B are partners with a profit and loss of 75:25 and capital balances of
P200,000 and P100,000, respectively. C is to be admitted into the partnership by
purchasing a 20% interest in the capital, profits and losses for P120,000.

The partners agree to revalue the assets before admitting C as a new partner.
Which of the following statements is incorrect?
a. The capital interest sold by A is P85,000
b. The total increase in capital of B after admission of C is P75,000

3|P a g e RFERRER/RLACO/ ATANG/PDEJESUS


No. 125 Brgy. San Sebastian
Lipa City, Batangas, Philippines
Mobile : 0927 283 8234
Telephone : (043) 723 8412
Gmail : icarecpareview@gmail.com

c. The share of A in the revaluation of asset is P225,000


d. The total amount of revaluation is P300,000

14. A and B are partners having capital balances of P300,000 and P360,000,
respectively. They agree to share profits and losses in the ratio of 50:50. They admit
C as a new partner to a 1/3 interest in capital and profits with an investment of
P390,000. If the partners agree to revalue the assets before admitting C into the
partnership.
a. Revaluation amount is P90,000
b. C capital will be P350,000
c. Total capital will be P1,050,000
d. B capital will be P420,000

15. A, B and C agree to liquidate the partnership. On December 31, 20x2, the following
data are available:
A B C
Capital 200,000 160,000 600,000
Drawing 120,000 80,000 40,000

The partnership was unable to collect on trade receivables and was forced to liquidate.
Operating profit in the year 20x2 amounted to P144,000 which was all exhausted
including the partnership assets. Unsettled creditor’s claim at December 31, 20x2
totaled P168,000. B and C have substantial private resources by A has no personal
assets. The partners share profits and losses equally.

How much is the loss on liquidation?


a. P1,032,000 c. P720,000
b. P960,000 d. P868,000

16. The final distribution to C was:


a. P324,000 c. P156,000
b. P168,000 d. P216,000

17. Which of the following statement is correct?


Statement 1: There is always loss on realization when noncash assets are converted
into cash during liquidation process.

Statement 2: In the liquidation process, the noncash assets are sold only to outside
parties but never to any of the partners.
a. I only c. Both statements are correct
b. II only d. Both statements are incorrect

18. A, B and C decided to liquidate the partnership. Their capital balance and profit and
loss ratio are as follows:
Capital P%L
A 100,000 40%
B 120,000 30%
C 40,000 30%

The net income on that year amounted to P88,000. Also, on this date, cash and
liabilities are P80,000 and P180,000, respectively.

4|P a g e RFERRER/RLACO/ ATANG/PDEJESUS


No. 125 Brgy. San Sebastian
Lipa City, Batangas, Philippines
Mobile : 0927 283 8234
Telephone : (043) 723 8412
Gmail : icarecpareview@gmail.com

How much is the cash realized from the sale of noncash assets in order for A to
receive P110,400.
a. P374,000 c. P354,000
b. P392,000 d. P386,000

19. The joint operation account reflects the transactions of the joint arrangement as
recorded by B:
Year 20x1 JONT OPERATION
Oct. 1 Merchandise - C 85,000 204,000 Nov. 10 Cash sales - A
10 Merchandise - B 70,000 42,000 Dec. 1 Cash Sales - A
15 Freight paid - A 2,000 12,100 Dec. 5 Merchandise - C
Nov. 20 Advertising - A 1,500
30 Purchases - A 35,000
Dec. 10 Selling expenses - A 4,000

The operators agree to share profits and losses to A , B and C in the ratio of 5:3:2,
respectively.
The total profit from joint operation amounted to:
a. P121,200 c. P246,000
b. P181,800 d. P60,600

20. The final settlement to C will be:


a. P97,120 c. P76,080
b. P85,020 d. P60,600

21. Which of the following scenarios indicates that the arrangement is a consignment
arrangement?
A. The entity is able to request for the return of the product subject to approval
of the other party.
B. The other party has an unconditional obligation to pay for the product.
C. The product is controlled by the entity until a specified event occurs.
D. All of the above indicates a consignment arrangement.

22. If there are significant changes in the entity’s expected timing of transfer to the
customer of goods or services, the amortization of costs to fulfill a contract shall
be:
A. accounted for as a correction of a prior period error
B. accounted for as a change in accounting policy
C. accounted for as a change in accounting estimate
D. ignored

23. In case the partnership assets are insufficient to settle outstanding partnership
claims,
A. the partnership must continue operating until such a time that the
partnership assets will be enough to cover all outstanding partnership claims
B. all unsecured creditors will receive a pro-rata settlement

5|P a g e RFERRER/RLACO/ ATANG/PDEJESUS


No. 125 Brgy. San Sebastian
Lipa City, Batangas, Philippines
Mobile : 0927 283 8234
Telephone : (043) 723 8412
Gmail : icarecpareview@gmail.com

C. the individual partners shall contribute additional assets to cover their


respective capital deficiencies, notwithstanding their individual financial
condition
D. the individual partners shall contribute additional assets to cover their
respective capital deficiencies, to the extent that their personal assets exceed
their personal liabilities

24. Which of the following is not a liability that has priority in corporate liquidation?
A. salaries payable to employees
B. taxes due to the government
C. preference dividends in arrears
D. liquidation and administrative related costs

25. A license such as a franchise is considered not distinct from other promised
goods or services in the contract if:
A. it is a license that forms a component of tangible good and that is integral to
the functionality of the good
B. it is a license that the customer can benefit on its own
C. both A and B
D. neither A nor B

Amounts related to the statement of affairs of Resigning Company as of April 30, 2021
follow:
Assets pledged for fully secured liabilities P 80,000
Assets pledged for partially secured liabilities 50,000
Free assets 272,000
Fully secured liabilities 60,000
Partially secured liabilities 80,000
Unsecured liabilities with priority 40,000
Unsecured liabilities without priority 330,000

26. What is the expected amount recoverable by partially secured creditors in the
event of liquidation?
A. P56,000
B. P70,400
C. P80,000
D. P71,000

27. How much is the estimated estate deficit?


A. P98,000
B. P58,000
C. P108,000
D. P88,000

United Co. recognizes construction revenue and expenses using the percentage-of-
completion method. During 2020, a single long-term project was begun, which
continued through 2021. Information on the project follows:
2020 2021
Accounts receivable from construction contract P200,000 P600,000
Construction expenses 210,000 384,000
Construction in progress 244,000 728,000

6|P a g e RFERRER/RLACO/ ATANG/PDEJESUS


No. 125 Brgy. San Sebastian
Lipa City, Batangas, Philippines
Mobile : 0927 283 8234
Telephone : (043) 723 8412
Gmail : icarecpareview@gmail.com

Partial billings on contract to date 200,000 840,000

28. How much is the profit from the long-term construction contract in 2021?
A. P314,000
B. P100,000
C. P30,000
D. P484,000

29. How much is the contract asset or contract liability to be reported as of


December 31, 2021?
A. P68,000 contract asset
B. P68,000 contract liability
C. P112,000 contract asset
D. P112,000 contract liability

30. How much is the cash collected during 2021 from the project?
A. P0
B. P240,000
C. P40,000
D. P128,000

ROMINA and MAUREEN are partners engaged in a manufacturing business.


Transactions affecting the partners’ capital accounts in 2021 are as follows:
ROMINA MAUREEN
Debit Credit Debit Credit
Beg. Balance P250,000 P350,000
April 1 150,000 100,000
June 30 125,000 250,000
September 1 225,000 300,000
October 1 350,000 200,000

The income summary has a debit balance of P225,000.

Agreement between ROMINA and MAUREEN are as follows:


 Interest on average capital at 8%.
 Salaries of P125,000 and P175,000 are given to ROMINA and MAUREEN,
respectively.
 Bonus to MAUREEN at 25% of net income after deducting interest and salaries
but before deducting bonus.
 Balance is to be divided equally.

31. How much is the share of Romina in the partnership profit or loss?
A. P143,000 loss
B. P143,000 gain
C. P99,000 loss
D. P99,000 gain

32. How much is the net increase (decrease) in Maureen’s capital account during
2021?
A. P143,000 decrease

7|P a g e RFERRER/RLACO/ ATANG/PDEJESUS


No. 125 Brgy. San Sebastian
Lipa City, Batangas, Philippines
Mobile : 0927 283 8234
Telephone : (043) 723 8412
Gmail : icarecpareview@gmail.com

B. P168,000 increase
C. P150,000 increase
D. P80,500 decrease

Mermaid Inc. consigned 50 units of its brand-new inventory costing P40,000 per unit to
Merman Company. Merman paid freight of P1,000 per unit in connection with the
consignment agreement, to be deducted from the amount to be remitted to Mermaid.
Merman will sell the inventory for P60,000 per unit and will be entitled to a 10%
commission on such sale. Merman incurs expenses in relation to selling twenty units to
customers amounting to P8,000 which was reimbursable under the consignment
agreement.

33. How much is the revenue to be recognized by Mermaid?


A. P1,080,000
B. P3,000,000
C. P2,700,000
D. P1,200,000

34. How much is the revenue to be recognized by Merman?


A. P70,000
B. P120,000
C. P300,000
D. P250,000

35. How much is the net profit to be reported by Mermaid from the consignment
arrangement?
A. P230,000
B. P400,000
C. P260,000
D. P280,000

On January 1, 2021, Entities ACT and JTC formed a joint operation. They agreed to
share equally on all matters relating to the operation. The following (at their respective
book values) were contributed by the operators:
ACT JTC
Cash P 300,000 P 100,000
Inventory 800,000 200,000
Property, plant and equipment, net 2,500,000 1,200,000

Additional information:
a. The fair values of the inventories are P1,000,000 and P300,000, respectively for
ACT and JTC.
b. The fair value of the property, plant and equipment invested by ACT was
P100,000 higher than the book value while the property, plant and equipment
invested by JTC had a fair value of P1,500,000. Both have a remaining useful life
of ten years.

8|P a g e RFERRER/RLACO/ ATANG/PDEJESUS


No. 125 Brgy. San Sebastian
Lipa City, Batangas, Philippines
Mobile : 0927 283 8234
Telephone : (043) 723 8412
Gmail : icarecpareview@gmail.com

36. How much is the share of ACT in the assets of the joint operation to be
presented in its separate statement of financial position?
A. P2,700,000
B. P2,900,000
C. P2,750,000
D. P2,550,000

37. How much is the property, plant and equipment, net - joint operation to be
presented in the separate statement of financial position of ACT as of December
31, 2021?
A. P1,665,000
B. P1,710,000
C. P1,845,000
D. P1,800,000

38. When the initial franchise fee is not paid in full and the collectability of the note
for the balance is reasonably assured, the method to be used by the franchisor
to recognize revenue from the initial franchise fee is:
a. Installment method
b. Gross Profit Method
c. Accrual Method
d. Cash Basis

39. According to IFRS 15, when should franchise revenue be recognized?


a. When operation commences
b. When the contract was signed
c. When the performance obligations are met
d. When the franchise fee is paid to the franchisor

40. Consignor Co. paid the in-transit insurance premium for consignment goods
shipped to Consignee Co. In addition, Consignor advanced part of the
commission that will be due when Consignee sells the goods. Should Consignor
include the in-transit insurance premium and the advanced commissions in
inventory costs?
Insurance Advanced
premium commission
a. Yes Yes
b. No No
c. Yes No
d. No Yes

41. On January 5, 2030, BenBen, Inc. signed an agreement granting Mr. Bennet a
franchise license for an initial franchise fee of P1,000,000 which was fully paid
when the contract was signed. Mr. Bennet commenced operations on July 1,2016
which cost BenBen Inc. P20,000 to perform its initial services. Mr. Bennet must
also pay BenBen Inc. a continuing franchise fee of 5% of its gross monthly sales.
Mr. Bennet reported gross sales from July 1 to December 31 2030 of P800,000.
On December 31, 2030, what is the net income from the franchise fees to be
reported by BenBen Inc.
a. 1,000,000

9|P a g e RFERRER/RLACO/ ATANG/PDEJESUS


No. 125 Brgy. San Sebastian
Lipa City, Batangas, Philippines
Mobile : 0927 283 8234
Telephone : (043) 723 8412
Gmail : icarecpareview@gmail.com

b. 980,000
c. 1,040,000
d. 1,020,000

AAA Inc. granted BBB a franchise on January 2, 2030. The agreement provided
an initial franchise fee of P2,000,000 payable as follows: P400,000 down
payment and the balance payable in four annual installments starting December
31, 2030. The prevailing interest rate for a similar note is 20% and the present
value of an annuity of 1 for 4 periods is P2.5887. The agreement also provides
for a continuing franchise fee of 5% of gross sales of the franchise payable 10
days the following month. The collectability of the note is reasonably assured.
The franchisee commenced operation on July 1,2030 and reported gross sales of
P4,000,000 from July to December 2030.

42. What is the total revenue from franchise fees to be reported by BBB for the year
ended December 31, 2030?
a. 2,200,000
b. 2,000,000
c. 1,635,480
d. 600,000

43. What is the other income related to the franchise agreement (not from franchise
fees) to be reported by BBB for the year ended December 31, 2030?
a. 0
b. 103,548
c. 320,000
d. 207,096

On January 1, 2030, an entity granted a franchise to a franchisee. The franchise


agreement required the franchisee to pay a nonrefundable upfront fee in the amount
of P400,000 and on-going payment of royalties equivalent to 5% of the sales of the
franchisee. The franchisee paid the nonrefundable upfront fee on January 1, 2030.

In relation to the nonrefundable upfront fee, the franchise agreement required the
entity to render the following performance obligations:
 To construct the franchisee’s stall with stand-alone selling price of P200,000.

 To deliver 10,000 units of raw materials to the franchisee with stand-alone


selling price of P250,000.

 To allow the franchisee to use the entity tradename for a period of 10 years
starting January 1, 2030 with stand-alone selling price of P50,000.

On June 30, 2030, the entity completed the construction of the franchisee’s stall. On
December 31, 2030, the entity was able to deliver 3,000 units of raw materials to the
franchisee. For the year ended December 31, 2030, the franchisee reported sales
revenue amounting to P100,000.

10 | P a g e RFERRER/RLACO/ ATANG/PDEJESUS
No. 125 Brgy. San Sebastian
Lipa City, Batangas, Philippines
Mobile : 0927 283 8234
Telephone : (043) 723 8412
Gmail : icarecpareview@gmail.com

The entity had determined that the performance obligations are separate and distinct
from one another.

44. What is the amount of nonrefundable upfront fee to be allocated to the


construction of the franchisee’s stall?

a. 200,000
b. 160,000
c. 250,000
d. 120,000

45. What is the amount of revenue to be recognized in relation to the use of delivery
of raw materials for the year ended December 31, 2030?

a. 100,000
b. 200,000
c. 60,000
d. 75,000

46. What is the amount of revenue to be recognized in relation to the use of entity’s
tradename for the year ended December 31, 2030?

a. 5,000
b. 4,000
c. 50,000
d. 10,000

Philacor, Inc. consigned twelve refrigerators to Ocampo’s Emporium. The refrigerators


cost P6,000 each and the consignor paid P720 for freight out. The consignee
subsequently rendered an account sales for five units sold at P7,700 each, and
deducted the following items from the selling price:
Commission (based on sales net of commission) 10%
Marketing expense (based on commission) 10%
Delivery and installation (on each unit sold) P 30

47. How much was the net profit of the consignor on the five refrigerators sold?
a. 3,815
b. 37,780
c. 4,200
d. 3,395

48. How much was the net remittance of the consignee on the five refrigerators sold?
a. 34,500
b. 33,780
c. 4,500
d. 4,200

11 | P a g e RFERRER/RLACO/ ATANG/PDEJESUS
No. 125 Brgy. San Sebastian
Lipa City, Batangas, Philippines
Mobile : 0927 283 8234
Telephone : (043) 723 8412
Gmail : icarecpareview@gmail.com

CR Manufacturing Co. consigned to CE Trading Corp. twelve (12) Sony colored TV


sets which cost P9,000 each. Freight out was paid by the consignor in the amount of
P600. CE Trading sold eight (8) sets, rendered an account sales, and remitted the
amount of P82,600 after deducting the following from the selling price of the sets
sold:
Commission on selling price 12%
Selling expenses P1,200
Cost of antennae given free 1,400
Delivery and installation 2,800

49. The total selling price of the eight (8) sets sold by CE Trading Corp. is:
a. 100,000.0
b. 88,000.00
c. 98,560.00
d. 78,571.43

50. The net profit of CR Manufacturing Co. on the eight (8) sets sold by CE Trading
Corp. is:
a. 10,200.00
b. 18,000.00
c. 18,200.00
d. 18,000.00

12 | P a g e RFERRER/RLACO/ ATANG/PDEJESUS
No. 125 Brgy. San Sebastian
Lipa City, Batangas, Philippines
Mobile : 0927 283 8234
Telephone : (043) 723 8412
Gmail : icarecpareview@gmail.com

AFAR First Preboard Examinations


iCARE Accountancy Review
st
1 Open Preboard Examination, Batch 3

1. B 41. D
2. D 42. C
3. A 43. D
4. B 44. B
5. D 45. C
6. B 46. B
7. C 47. C
8. C 48. A
9. B 49. A
10. A 50. A
11. C
12. C
13. B
14. D
15. A
16. C
17. D
18. D
19. D
20. B
21. C
22. C
23. D
24. C
25. A
26. D
27. C
28. B
29. D
30. B
31. A
32. B
33. D
34. B
35. C
36. C
37. D
38. C
39. C
40. C

13 | P a g e RFERRER/RLACO/ ATANG/PDEJESUS
lOMoARcPSD|12648093

AFAR May2021 1st Preboard with answer

Financial Accounting (Ateneo de Manila University)

StuDocu is not sponsored or endorsed by any college or university


Downloaded by ruzz ruzz (ruzzst@jruzz.ml)
lOMoARcPSD|12648093

Accountancy Review Center (ARC)


of the Philippines Inc.
One Dream, One Team

FIRST PREBOARD EXAMINATION

ADVANCED FINANCIAL ACCOUNITNG AND REPORTING BULADACO/SAGOT


BATCH 01 MAY 2021 CPALE REVIEW

INSTRUCTIONS: Select the best answer for each of the following questions. Mark only one answer for each item
on the answer sheet provided. STRICTLY NO ERASURERS ALLOWED.

1. An entity enters into a contract with a customer to provide a weekly service for one year. The contract is
signed on January 1, 2020 and work begins immediately. The entity concludes that the service is a single
performance obligation because the entity is providing a series of distinct services that are substantially
the same and have the same pattern of transfer. In exchange for the service, the customer promises 100
shares of its common stock per week of service (a total of 5,200 shares for the contract). The terms in the
contract require that the shares must be paid upon the successful completion of each week of service.
Under PFRS 15, revenue will be recognized:
A. At the fair value of the shares that are received upon completion of each weekly service
B. At the fair value of the services received upon completion of each weekly service
C. At the fair value of the shares that are received upon completion of each weekly service, and adjusted
subsequently for the changes in the fair value of the shares received
D. At the fair value of the shares or services, whichever is more observable, at the inception of the contract

2. The adjusted balance of the loading in branch inventory represents the overvaluation of the branch’s
A. Total cost of goods available for sale
B. Cost of goods sold
C. Ending inventory
D. Beginning inventory

3. Which partner is considered the most vulnerable as a result of a computation of vulnerability rankings?
A. The partner with the lowest vulnerability ranking, who also has the lowest loss absorption potential.
B. The partner with the lowest vulnerability ranking, who also has the highest loss absorption potential.
C. The partner with the highest vulnerability ratio, who also has the lowest loss absorption potential.
D. The partner with the highest vulnerability ranking, who also has the highest loss absorption potential.

4. Statement 1: A contract with a customer would not have a significant financing component if the customer
has paid for the goods or services in advance and the timing of the transfer of those goods or services is at
the discretion of the customer.
Statement 2: An entity may recognize revenue over time if the entity’s performance does not create an
asset with an alternative use and the entity has an enforceable right to payment for a certain portion of the
contract.

A. Only statement 1 is incorrect.


B. Only statement 2 is incorrect.
C. Both statements are correct.
D. Neither statements is correct.

5. Under Article 2250 of the Civil Code, the excess after the payment of credits which enjoy preference with
respect to specific property shall be:
A. Considered an unsecured credit without preference
B. Attached as security for satisfaction of the related credit
C. Added to the property distributable to the debtor’s shareholders
D. Added to the free property which the debtor may have for the payment of other credits

Downloaded by ruzz ruzz (ruzzst@jruzz.ml)


lOMoARcPSD|12648093

AFAR | FIRST PREBOARD EXAMINATION ARC – ACCOUNTANCY REVIEW CENTER

6. A well-known consignor consigned goods costing P =1,560,000 to be sold at a total advertised selling price
of P
=3,120,000. The consignee remitted = P1,447,600 to its consignor after deducting the following charges:
=52,500 cartage, P
P =22,500 installation and a commission equal to 20% of the sales proceeds after deducting
the commission. Determine the net income of the consignor.
A. 507,600
B. 516,600
C. 546,300
D. 555,840

Solution:

BONUS. Remittance in the problem should be P


=1,422,600.

Determine the percentage of items sold over items consigned during the period.

Remittance 1,422,600
Add back
Cartage 52,500
Installation 22,500
Net remittance after commission 1,497,600
Add commission (20%) 299,520
Proceeds from sale of goods 1,797,120
Divided by total advertised price 3,120,000
Percentage of units sold 57.60%

Revenue from sale 1,797,120


Cost of sales
Purchase price (1,560,000 x 57.6%) (898,560)
Cartage (52,500 x 57.6%) (30,240)
Gross profit 868,320
Installation (22,500)
Commission (299,520)
Net income 546,300

7. A manufacturing firm has found itself in financial difficulty and may file for bankruptcy. Its statement of
affairs reflects the following summarized information:

Book value of assets P700,000


=
Net realizable value of assets 370,000
Total liabilities 400,000
Secured claims 250,000
Unsecured claims with priority 30,000

If the corporation owes a creditor =


P9,000 secured by inventory that is expected to realize P
=7,000, how
much can the creditor expect to receive on this claim?
A. 8,200
B. 8,500
C. 8,600
D. 9,000

Solution:

Determine the:
 Net free assets = 370,000 – 250,000 – 30,000 = 90,000
 Unsecured claims without priority = 400,000 – 250,000 – 30,000 = 120,000

ERP = 90,000/120,000 = 75%

Creditor can expect to receive 8,500 from the claims


(7,000 inventory + 2,000 x 75%)

0961-718-5293; 0936-407-4780; (02)-8376-0405 www.arccpalereview.com Page 2 of 28

Downloaded by ruzz ruzz (ruzzst@jruzz.ml)


lOMoARcPSD|12648093

AFAR | FIRST PREBOARD EXAMINATION ARC – ACCOUNTANCY REVIEW CENTER

8. The Investment in Branch A per home office books is higher than the home office account per Branch A’s
books. What can a possible reason for the difference?
A. Branch A forgot to record advertising expenses allocated by the home office.
B. Home office was not yet notified of the collection made by Branch A on the receivables of Branch B.
C. Branch recorded shipments returned to the home office twice in its books.
D. Home office erroneously recorded the shipments received by Branch A as an additional investment in
Branch B.

9. The partnership of French and Fries, who shares profits and losses on a 3:2 ratio, is in the process of
liquidation. On January 1, 2020, the ledger shows capital balances of French and Fries amounting to
=80,000 and P
P =40,000 respectively and accounts payable to third parties of P
=30,000. Inventories w57.2ere
sold at a loss of P
=30,000, while accounts receivable of P =42,000 were collected and P
=8,000 were deemed
fully impaired and uncollectible. Of the total equity of French, what amount appears to be unrecoverable?
A. 57,200
B. 30,800
C. 22,800
D. 4,800

Solution:

French Fries
Capital balance 80,000 40,000
Loss on sale of inventory (18,000) (12,000)
Loss on write-off (4,800) (3,200)
57,200 24,800

10. An asset recognized for an entity’s right to recover products from a customer on settling a refund liability
shall be measured considering which of the following factors:
I. Former carrying amount of the product
II. Expected costs to recover the product
III. Potential decreases in the value of the returned products

A. I only
B. I and II only
C. I and III only
D. I, II and III

11. The main office pays the following standard freight charges:

From Branch A to B, or vice versa P6,000


=
From Branch A to C, or vice versa 4,500
From Branch B to C, or vice versa 8,000
From main office to Branches A and B 10,000
From main office to Branch C 7,000

The main office ships merchandise to Branch B and later on orders Branch B to distribute half of the same
merchandise to Branch A and half to Branch C, then the difference in freight charge would be disposed as:
A. 1,500 freight savings and 3,500 charged to head office
B. 2,000 charged to head office
C. 3,000 freight savings
D. 4,000 freight savings and 1,000 charged to head office

BONUS.

12. On September 1, 2020, a consignor consigned 10,000 units of inventory which costs
=12 per unit and sells for P
P =15 per unit. Freight charges of P
=15,000 were paid by the consignee. The
consignee was able to sell 6,000 units from September 1 to November 30, 2020 and incurred. On December
1, 2020, the products were marked to sell for P =13 per unit to accommodate the Christmas rush. During
December, the consignee was able to sell 3,000 more units. The discounted price lasted until February 28,
2021. Consignee is entitled to a commission of 10% of the selling price of the products. For the period
ending December 31, 2020, the consignor will report net loss from this consignment arrangement
amounting to:
A. 7,200
B. 6,000
C. 5,900
D. 5,400

0961-718-5293; 0936-407-4780; (02)-8376-0405 www.arccpalereview.com Page 3 of 28

Downloaded by ruzz ruzz (ruzzst@jruzz.ml)


lOMoARcPSD|12648093

AFAR | FIRST PREBOARD EXAMINATION ARC – ACCOUNTANCY REVIEW CENTER

Solution:

Revenue
First sale (6,000 x P
=15) 90,000
Second sale (3,000 x P =13) 39,000
Cost of sale (9,000 x P
=13.5)* (121,500)
Commission (10%) (12,900)
Inventory write-down** (1,800)
(7,200)

*Purchase price + Freight


*NRV is selling price less cost to sell (P
=13 less P
=1.3 commission = P
=11.7), since inventories are carried at
LCNRV. Cost of the remaining 1,000 units amounting to P =13,500 will be brought down to NRV and the
consignor will record an inventory write-down of P =7,200 (P
=13,500 cost versus P=11,700 NRV).

Situational
Problems 13 to 15.

The main office is in the process of combining the separate books of its branches Y and Z and has prepared the
following condensed data from their trial balances.

Main Branch Y Branch Z


Cash 3,200,000 350,000 680,000
Receivables 4,870,000 580,000 950,000
Inventories–January 1 1,120,000 680,000 760,000
Branch Y 1,525,000 – –
Branch Z 1,465,000 – –
Loading in branch inventories (1,412,200) – –
Fixed assets 5,400,000 – –
Accounts payables (2,560,000) (1,250,000) (950,000)
Long-term debt (6,500,000) – –
Capital stock (5,000,000) – –
Retained earnings–January 1 (3,200,000) – –
Dividends paid 800,000 – –
Home office – (1,245,000) (1,675,000)
Sales (6,900,000) (3,500,000) (2,750,000)
Purchases from outsiders 9,500,000 450,000 145,000
Net shipments to branches (4,724,000) – –
Net shipments from main office – 3,150,000 2,475,000
Expenses 2,416,200 785,000 365,000

Additional information:
a. Home office consistently bills its two branches at a 25% markup on cost.
b. Branch Y returned merchandise on December 29, 2020. This was still in transit as of year-end, and the
main office has not yet recorded this in its books.
c. Branch Z failed to record the following:
• Debit memo of the home office representing expenses allocated by the main office for its
promotional activities during the year
• Credit memo from Branch Y representing cash collected by Branch Y on its accounts receivable
amounting to P=250,000; the collection was subject to a two percent prompt payment discount
and the home office was already notified of this transaction.
d. Per physical count on December 28, 2020, inventories amounted to P =1,560,000 for main office, =P
1,785,000 for Branch Y (at billed price) and P=840,000 for Branch Z (at billed price). The count provides
that inventories of Branch Y and Z still have P
=125,000 and P=40,000 from outside vendors, respectively.

In the main office’s external financial statements, determine the:

13. Cost of sales.


A. 8,506,800
B. 8,674,800
C. 8,730,800
D. 8,954,800

0961-718-5293; 0936-407-4780; (02)-8376-0405 www.arccpalereview.com Page 4 of 28

Downloaded by ruzz ruzz (ruzzst@jruzz.ml)


lOMoARcPSD|12648093

AFAR | FIRST PREBOARD EXAMINATION ARC – ACCOUNTANCY REVIEW CENTER

14. Net income.


A. 589,000
B. 813,000
C. 869,000
D. 1,037,000

15. Total assets.


A. 19,529,000
B. 19,697,000
C. 19,249,000
D. 19,473,000

Solution:

13 – Cost of sales

a. Determine the cost of sales per separate books.

Main Branch Y Branch Z


Inventories–January 1 1,120,000 680,000 760,000
Purchases from outsiders 9,500,000 450,000 145,000
Net shipments to branches (4,724,000) – –
Net shipments from main office – 3,150,000 2,475,000
Unrecorded shipment returns at cost* 224,000 – –
Inventories–December 31** (1,784,000) (1,505,000) (840,000)
Cost of sales 4,336,000 2,775,000 2,540,000

*This represents the cost equivalent of the difference in branch Y per main office books and
home office account per branch books, and the difference between net shipments to
branches per main office books and the net shipments from main office per the two
branches’ books. Inventories are carried in the main office books at cost.

**Remember that the inventories of the main office should include the shipment in transit.
Also, the shipment in transit should be removed in the amount of the inventories of Branch Y
(as provided in item d) at the billed price of P
=280,000, since the count happened on
December 28 and the shipment was made the day after.

b. Determine the overvaluation in branches’ cost of sales

Loading in branch per books 1,412,200


Overvaluation on unrecorded shipment in transit (56,000)
Overvaluation in branch ending inventories
Branch Y [(1,505,000 – 125,000) x 25/125] (276,000)
Branch Z [(840,000 – 40,000) x 25/125] (160,000)
920,200

c. Determine the cost of sales to be reported.

Combined COS per books 9,651,000


Overvaluation (920,200)
Combined COS per external financial statements 8,730,800

14 – Net income

Combined
Sales 13,150,000
Sales discount (item C) (5,000)
Cost of sales (8,730,800)
Expenses
Per trial balance (3,566,200)
Unrecorded debit memo by Branch Z* (35,000)
Combined net income 813,000

0961-718-5293; 0936-407-4780; (02)-8376-0405 www.arccpalereview.com Page 5 of 28

Downloaded by ruzz ruzz (ruzzst@jruzz.ml)


lOMoARcPSD|12648093

AFAR | FIRST PREBOARD EXAMINATION ARC – ACCOUNTANCY REVIEW CENTER

*A reconciliation of reciprocal accounts will reveal:

Main office books Branch Z books


Reciprocal account 1,465,000 1,675,000
Unrecorded transactions
Debit memo (squeezed) 35,000
Collections by Branch Y (245,000)
Adjusted balance 1,465,000 1,465,000

15 – Total assets

Solution A:

Combined
Cash (3,200,000 + 350,000 + 680,000) 4,230,000
Receivables (4,870,000 + 580,000 + 950,000) 6,400,000
Unrecorded credit memo from Branch Y (250,000)
Inventories
Main office 1,784,000
Branch Y (1,505,000 – 276,000) 1,229,000
Branch Z (840,000 – 160,000) 680,000
Fixed assets 5,400,000
Total assets 19,473,000

Solution B:

Combined
Accounts payable 4,760,000
Long-term det 6,500,000
Capital stock 5,000,000
Retained earnings 3,200,000
Combined net income 813,000
Dividends paid (800,000)
Total assets 19,473,000

Problem 16 to 18.
After completing the purpose for which the partnership was established, Blow, Plow, and Glow voted to
dissolve their partnership and liquidate by selling its other assets. A balance sheet and the residual profit and
loss sharing percentages are as follows:

Cash =
P 2,200,000 Accounts payable =
P 2,000,000
Blow, loan 300,000 Plow, loan 1,000,000
Other assets 2,200,000 Glow, loan 100,000
Blow, capital (30%) 1,060,000
Plow, capital (50%) (50,000)
Glow, capital (20%) 590,000
Total assets =
P 4,700,000 Total liabilities and equity =
P 4,700,000

Situation 1 – Assume that other assets were sold as follows: (1) half at a markup of 20% above its carrying
amount, and (2) one-fourth at a discount of 75% of its carrying amount. Further, proceeds were used to settle
ninety per cent of the accounts payable for =
P1,700,000.

Situation 2 – Assume that Partner Plow invested P


=30,000 from his excess personal assets to cure a portion of
his deficiency. Partner Blow received P
=148,000.

Situation 3 - Assume that Plow’s capital deficit and Glow’s capital credit after the first distribution amounted
to P
=160,000 and P=198,000, respectively after the partnership incurred liquidation expenses of P =20,000 and net
losses on sale. The partnership settled 75% of the accounts payable at face value.

16. Under the third situation, how much is the total unpaid liabilities of the partnership after available cash
was distributed to all partners?
A. 500,000
B. 1,155,000
C. 1,405,000
D. 1,505,000

0961-718-5293; 0936-407-4780; (02)-8376-0405 www.arccpalereview.com Page 6 of 28

Downloaded by ruzz ruzz (ruzzst@jruzz.ml)


lOMoARcPSD|12648093

AFAR | FIRST PREBOARD EXAMINATION ARC – ACCOUNTANCY REVIEW CENTER

17. Under the first situation, at the first distribution, Glow received payment for his capital balance at an
amount of:
A. 461,500
B. 501,500
C. 516,500
D. 561,500

18. Under the second situation, how much is the combined losses from sale and write-off?
A. 1,970,000
B. 2,000,000
C. 2,040,000
D. 2,070,000

Solution:

16 – Situation 3

Since Plow’s capital balance is still a deficit even after the distribution, it means that no distributions
were made out of his capital, and he only suffered losses from liquidation.

Plow
Capital prior to liquidation (50,000)
Share in liquidation expense (10,000)
Share in net losses on sale (squeezed) (100,000)
Deficit after first distribution (160,000)

Hence, the losses on sale incurred by the entire partnership was P


=200,000 (100,000/50%).

Next, let’s determine the amount distributed to Partner Glow.

Glow
Capital prior to liquidation 590,000
Share in liquidation expense (4,000)
Share in net losses on sale (40,000)
Capital distribution (squeezed) (348,000)
Capital after first distribution 198,000
If Partner Glow received cash out from his capital, that means all his loans were also paid out. Hence,
total cash distributed to Partner Glow would be P
=448,000.

Next, we determine the amount distributed to Partners Blow and Plow. We first compare the net
interest of Glow versus the cash distributed to get his share in the net loss and absorption of deficit.
We distribute the difference of =
P242,000 using the profit or loss ratio to determine the cash
payments to other partners.

Blow Plow Glow


Net interest 760,000 950,000 690,000
Net loss (363,000) (605,000) (242,000)
Cash payments 397,000 345,000 448,000

Then, we determine the total liabilities.

Glow
Unpaid AP to third parties (25%) 500,000
Unpaid loan from Plow (1,000,000 – 345,000) 655,000
Unpaid liabilities 1,155,000

0961-718-5293; 0936-407-4780; (02)-8376-0405 www.arccpalereview.com Page 7 of 28

Downloaded by ruzz ruzz (ruzzst@jruzz.ml)


lOMoARcPSD|12648093

AFAR | FIRST PREBOARD EXAMINATION ARC – ACCOUNTANCY REVIEW CENTER

17 – Situation 1

a. Determine available cash for distribution.

Cash, beginning 2,200,000


Proceeds from sale of other assets
First half (1,100,000*1.20) 1,320,000
Second half (550,000 x 25%) 137,500
Actual payment of liabilities (1,700,000)
Cash withheld for remaining liabilities (200,000)
1,757,500

b. Compare with net interest of partners in the partnership. Net interest of partners amount to
=2,400,000 (capital and loans). Total loss is P
P =642,500.

c. Determine cash payments to partners.

Blow Plow Glow


Net interest 760,000 950,000 690,000
Total loss (642,500) (192,750) (321,250) (128,500)
Cash payments 567,250 628,750 561,500

If Glow received P
=561,500, this shall be first applied to his P
=100,000 loans. The remainder of
P461,500 will be charged for his capital balance.
=

18 – Situation 2

First compare the cash received by Blow versus its net interest. The difference will be grossed up to
determine the total loss and will be allocated to other partners using the P/L ratio.

Blow Plow Glow


Net interest (plus investment) 760,000 980,000 690,000
Net loss (612,000) (1,020,000) (408,000)
Cash payments 148,000 (40,000) 282,000

The total cash distributed was P


=430,000, representing the positive amounts in the table. The total
loss was P
=2,040,000, comprising the P=40,000 negative balance of Plow absorbed by the other
partners and the =P2,000,000 losses on write-off and sale.

Problem 19 and 20.

An entity enters into a contract with a customer, a distributor, on December 1, 2020. The entity transfers 1,000
products at contract inception for a price stated in the contract of P=100 per product (total consideration is P
=
100,000). Payment from the customer is due when the customer sells the products to the end customers. The
entity's customer generally sells the products within 90 days of obtaining them. Control of the products
transfers to the customer on December 1, 2020.

On the basis of its past practices and to maintain its relationship with the customer, the entity anticipates
granting a price concession to its customer because this will enable the customer to discount the product and
thereby move the product through the distribution chain. Consequently, the consideration in the contract is
variable.

19. The entity has significant experience selling this and similar products. The observable data indicate that
historically the entity grants a price concession of approximately 20 per cent of the sales price for these
products. Current market information suggests that a 20 per cent reduction in price will be sufficient to
move the products through the distribution chain. The entity has not granted a price concession
significantly greater than 20 per cent in many years. How much will be the transaction price under PFRS
15 recognized as revenue on December 1, 2020?
A. 100,000
B. 80,000
C. 20,000
D. Nil.

0961-718-5293; 0936-407-4780; (02)-8376-0405 www.arccpalereview.com Page 8 of 28

Downloaded by ruzz ruzz (ruzzst@jruzz.ml)


lOMoARcPSD|12648093

AFAR | FIRST PREBOARD EXAMINATION ARC – ACCOUNTANCY REVIEW CENTER

20. The entity has experience selling similar products. However, the entity's products have a high risk of
obsolescence and the entity is experiencing high volatility in the pricing of its products. The observable
data indicate that historically the entity grants a broad range of price concessions ranging from 20–60 per
cent of the sales price for similar products. Current market information also suggests that a 15–50 per cent
reduction in price may be necessary to move the products through the distribution chain. The entity's
actual results have been consistent with then-current market information in previous, similar transactions.
How much will be the transaction price under PFRS 15 recognized as revenue on December 1, 2020?
A. 30,000
B. 40,000
C. 50,000
D. 60,000

Solution:

19 – P
=100,000 less 20 percent price concession. Lifted from PFRS 15 illustrative examples.

20 – Using the expected value method, the entity estimates that a discount of 40 per cent will be
provided and, therefore, the estimate of the variable consideration is P =60,000 (P =60 × 1,000 products).
The entity considers the factors in paragraph 57 of IFRS 15 and observes that the amount of
consideration is highly susceptible to factors outside the entity’s influence (i.e. risk of obsolescence)
and it is likely that the entity may be required to provide a broad range of price concessions to move
the products through the distribution chain. Consequently, the entity cannot include its estimate of
=60,000 (i.e. a discount of 40 per cent) in the transaction price because it cannot conclude that it is
P
highly probable that a significant reversal in the amount of cumulative revenue recognized will not
occur. Although the entity’s historical price concessions have ranged from 20–60 per cent, market
information currently suggests that a price concession of 15–50 per cent will be necessary. The
entity’s actual results have been consistent with then-current market information in previous, similar
transactions. Consequently, the entity concludes that it is highly probable that a significant reversal in
the cumulative amount of revenue recognized will not occur if the entity includes P =50,000 in the
transaction price (P =100 sales price and a 50 per cent price concession) and therefore, recognizes
revenue at that amount. Therefore, the entity recognizes revenue of = P50,000 when the products are
transferred and reassesses the estimates of the transaction price at each reporting date until the
uncertainty is resolved

Problem 21 to 23.

An entity is experiencing financial problems which resulted to its ultimate bankruptcy. The statement of
financial position of the said entity before its liquidation follows:

Cash =
P 900,000 Trade payables =
P 300,000
Inventories 825,000 Accrued salaries 600,000
Equipment 3,600,000 Income tax payable 900,000
Long-term debt 1,200,000
Mortgage debt 1,500,000
Equity 825,000
Total assets =
P 5,325,000 Total liabilities and equity =
P 5,325,000

Additional information:
• Liquidation expenses amounting to P =1,800,000 were paid.
• Long-term debt is secured by the inventories. P =450,000 of which is currently marked to sell at a
markup of twenty percent of sales price, while the remainder can be sold at its scrap value of eighty
percent of the carrying amount.
• Equipment serves as a collateral to the mortgage debt.

In each of the following independent situation, determine the amount expected to be received by the holder of
the trade payables. Round ERP to two decimal places (e.g. 69%)

21. Holder of the long-term debt received =


P1,020,000 at the end of the liquidation.
A. 100,000
B. 140,000
C. 150,000
D. 207,000

0961-718-5293; 0936-407-4780; (02)-8376-0405 www.arccpalereview.com Page 9 of 28

Downloaded by ruzz ruzz (ruzzst@jruzz.ml)


lOMoARcPSD|12648093

AFAR | FIRST PREBOARD EXAMINATION ARC – ACCOUNTANCY REVIEW CENTER

22. Fair value of the equipment is P


=4,282,500.
A. 180,000
B. 174,000
C. 132,000
D. 129,000

23. Equipment is marked to sell at a markup of 10% on cost.


A. 21,000
B. 27,000
C. 45,000
D. 48,000

Solution:

21

First, let’s determine the realizable value of inventories.

MV of 450,000 (450,000/80%) 562,500


MV of remainder (375,000*80%) 300,000
Net realizable value 862,500

Then, let’s analyze the long-term debts’ carrying and settlement value.

Carrying amount ERP Settlement amount


Secured portion 862,500 100% 862,500
Unsecured portion 337,500 46.67%* 157,500
1,200,000 1,020,000
*worked back

Last, determine the amount expected to be received by the holder of trade payable.

Trade payables–carrying amount 300,000


ERP 46 2/3%
Trade payables–settlement amount 140,000

22

NRV Secured Free assets Unsecured


Cash 900,000 900,000 –
Inventories 862,500 1,200,000 – 337,500
Equipment 4,282,500 1,500,000 2,782,500 –
6,045,000 2,700,000 3,682,500 337,500
Unsecured credits
With priority
Accrued salaries (600,000)
Liquidation expenses (1,800,000)
Income tax payable (900,000)
Without priority–Trade 300,000
payables
382,500 637,500

Trade payables–carrying amount 300,000


ERP (382,500/637,500) 60%
Trade payables–settlement amount 180,000

0961-718-5293; 0936-407-4780; (02)-8376-0405 www.arccpalereview.com Page 10 of 28

Downloaded by ruzz ruzz (ruzzst@jruzz.ml)


lOMoARcPSD|12648093

AFAR | FIRST PREBOARD EXAMINATION ARC – ACCOUNTANCY REVIEW CENTER

23

NRV Secured Free assets Unsecured


Cash 900,000 900,000 –
Inventories 862,500 1,200,000 – 337,500
Equipment 3,960,000 1,500,000 2,460,000 –
5,722,500 2,700,000 3,360,000 337,500
Unsecured credits
With priority
Accrued salaries (600,000)
Liquidation expenses (1,800,000)
Income tax payable (900,000)
Without priority–Trade 300,000
payables
60,000 637,500

Trade payables–carrying amount 300,000


ERP (60,000/637,500) 9%
Trade payables–settlement amount 27,000

Problem 24 to 25.

The working paper for the liquidation of a partnership shows the following:

Assets Debts PAR PART PAT


Beginning balance ? ? 40,000 25,000 5,000
Proceeds from sale 37,000 – – – –
Book value of assets sold ? – – – –
Loss on sale – – ? ? ?
Payments (12,000) (12,000) – – –
Elimination of deficiency – – ? ? ?
Distribution (28,000) – ? ? ?
Ending balance – – – – –

PAT, PAR and PART share profits and losses in a 1:3:2 ratio, respectively.

24. Determine the loss on sale.


A. 40,000
B. 42,000
C. 44,000
D. 45,000

25. Determine the share of PAR in the distribution.


A. 19,000
B. 18,000
C. 17,800
D. 40,000

Solution

PAR PART PAT


Assets Debts (3/6) (2/6) (1/6)
Beginning balance 82,000 12,000 40,000 25,000 5,000
Proceeds from sale 37,000 – – – –
Book value of assets sold (79,000) – – – –
Loss on sale – – (21,000) (14,000) (7,000)
Payments (12,000) (12,000) – – –
Elimination of deficiency – – (1,200) (800) 2,000
Distribution (28,000) – (17,800) (13,200) ?
Ending balance – – – – –

0961-718-5293; 0936-407-4780; (02)-8376-0405 www.arccpalereview.com Page 11 of 28

Downloaded by ruzz ruzz (ruzzst@jruzz.ml)


lOMoARcPSD|12648093

AFAR | FIRST PREBOARD EXAMINATION ARC – ACCOUNTANCY REVIEW CENTER

Problem 26 to 28.

During 2020, a franchisor granted four franchisees the right to access the intellectual property of the franchisor
allowing the latter to use the former’s trade name, proprietary know-how and market area.

The contractual consideration of the contracts follow:

Franchisee Consideration Commencement date Term


Franchisee Gray 6,000,000 May 30 5 years
Franchisee Pray 7,000,000 December 1 5 years
Franchisee Tray 8,000,000 September 30 5 years

Review of the contract details provides the following:

Situation 1 – The consideration of Gray is payable as follows: P =1,000,000 at the start of the contract and the
remainder in two annual installment starting one year from the contract date. This is evidenced by a note
bearing 6% annual interest collectible every year. For this franchise agreement, equipment, with cost of P =
500,000 will also be delivered. The franchisor estimates the stand‑alone selling price of the equipment to be
equal to the cost plus ten per cent margin, which the franchisor concludes is consistent with the rate that a
market participant would require as compensation for providing the equipment and for assuming the risk
associated with it. The performance obligation to deliver the equipment was assessed by the franchisor to be
distinct from the provision of access to the franchise right and was satisfied during the year.

Situation 2 – The consideration of Tray is payable in four equal annual instalments starting on commencement
date. Such is evidenced by a note bearing 6% annual interest collectible every year. However, the 6%
contractual rate of interest is significantly lower than the 12% interest rate that would be used in a separate
financing transaction between the entity and its customer at contract inception.

Situation 3 – The consideration of Pray was payable upfront and was charged in consideration of the following
distinct performance obligations: training services, construction services and franchise right, with stand-alone
selling prices of P
=1,200,000, P
=3,375,000 and P =2,925,000, respectively. Training services were completed in
2021, while construction of the establishment was 40% complete as of December 31, 2020 and was completed
by 2021.

In all its franchise arrangements, the franchisor assesses that it grants the franchisees the right to access the
franchise right. Hence, revenue from franchise rights shall be recognized by the entity over time consistent
with the requirements of PFRS 15.

Round both present and future value factors to two decimal places only.

26. Determine the interest income in 2021 earned by the franchisor from its franchise arrangement with
Franchisee Tray.
A. 796,041
B. 596,343
C. 211,272
D. 161,928

27. How much of the transaction price from Franchisee Pray will be presented as a liability as of December 31,
2020?
A. 3,807,500
B. 5,694,500
C. 4,647,000
D. 6,404,000

28. Determine the contract liability of the franchisor with its franchise arrangement with Franchisee Gray as
of December 31, 2021.
A. 3,724,167
B. 4,858,333
C. 3,758,333
D. 4,814,167

0961-718-5293; 0936-407-4780; (02)-8376-0405 www.arccpalereview.com Page 12 of 28

Downloaded by ruzz ruzz (ruzzst@jruzz.ml)


lOMoARcPSD|12648093

AFAR | FIRST PREBOARD EXAMINATION ARC – ACCOUNTANCY REVIEW CENTER

Solution:

26 – Franchisee Tray

Principal Interest Cash flows PV factor PV


9/30/2020 2,000,000 – 2,000,000 1.00 2,000,000
9/31/2021 2,000,000 360,000 2,360,000 0.89 2,100,400
9/31/2022 2,000,000 240,000 2,240,000 0.80 1,792,000
9/31/2023 2,000,000 120,000 2,120,000 0.71 1,505,200
8,000,000 720,000 8,720,000 7,397,600

Interest Interest Carrying


Principal received income Amortization amount
9/30/2020 – – – – 7,397,600
9/30/2020 2,000,000 – – – 5,397,600
9/31/2021 2,000,000 360,000 647,712 287,712 3,685,312
9/31/2022 2,000,000 240,000 442,237 202,237 1,887,549
9/31/2023 2,000,000 120,000 232,451 112,451 –
8,000,000 720,000 1,322,400

Interest income in 2021 would be 596,343 (647,712 x 9/12 + 442,237 x 3/12).

27 – Franchisee Pray

Performance obligation Stand-alone Allocation Transaction price


Training services 1,200,000 16% 1,120,000
Construction services 3,375,000 45% 3,150,000
Franchise right 2,925,000 39% 2,730,000
7,500,000 7,000,000

Performance obligation Satisfied in 2020 Liability,


Transaction price 12/31/20
Training services 1,120,000 – 1,120,000
Construction services 3,150,000 1,260,000 1,890,000
Franchise right 2,730,000 45,500 2,684,500
7,000,000 1,305,500 5,694,500

28 – Franchisee Gray

Transaction price 6,000,000


Stand-alone selling price of equipment (550,000)
Transaction price allocated to the franchise right using the residual approach 5,450,000
Less amortization for 19 months (1,725,833)
Contract liability, as of December 31, 2021 3,724,167

29. On June 30, 2021, an entity grants a franchisee the right to operate a restaurant in a specific market using
the entity’s brand name, concept and menu for a period of ten years. The entity has granted others similar
rights to operate this restaurant concept in other markets. The entity commonly conducts national
advertising campaigns, promoting the brand name, and restaurant concept generally. The franchisee will
also purchase kitchen equipment from the entity. The consideration of the franchise agreement is P =
9,500,000 for the initial franchise fee plus a royalty, paid quarterly, based on 4% of the franchisee’s sales
over the life of the contract. The entity assesses the performance obligations to transfer the equipment
and to provide access to the franchise right as distinct. Their stand-alone selling prices are P
=1,000,000 and
=9,00,000, respectively. The equipment was transferred on July 15, 2021 and the franchisee reported sales
P
of P
=34,000,000 in 2021. Assume that the consideration is paid upfront. Determine the contract liability as
of December 31, 2023.
A. 7,367,500
B. 7,362,500
C. 7,267,500
D. 6,412,500

0961-718-5293; 0936-407-4780; (02)-8376-0405 www.arccpalereview.com Page 13 of 28

Downloaded by ruzz ruzz (ruzzst@jruzz.ml)


lOMoARcPSD|12648093

AFAR | FIRST PREBOARD EXAMINATION ARC – ACCOUNTANCY REVIEW CENTER

Solution:

Transaction price 9,500,000


Allocation ratio for franchise right (90%) (550,000)
Transaction price allocated to the franchise right 8,550,000
Less amortization for 30 months (2,137,500)
Contract liability, as of December 31, 2023 6,412,500

Problem 30 to 32.

The income statement submitted by the branch to the main office for the year 2020 is shown below:

Sales 600,000
Cost of sales:
Inventory, January 1 80,000
Shipments from H.O. 350,000
Local purchases 30,000
Total available for sale 460,000
Inventory, December 31 100,000 360,000
Gross profit 240,000
Operating expenses 180,000
Net income 60,000

The branch inventories include those purchased from outsiders amounting to P =16,000 and P =10,000 as of
December 31 and January 1, respectively. After effecting the necessary adjustments, the true net income of the
branch was ascertained to be P
=156,000.

30. The ending inventory of the branch as far as the home office is concerned is:
A. 73,000
B. 76,000
C. 77,600
D. 83,200

31. The unadjusted balance of the allowance for overvaluation of the branch inventories as of December 31,
2020 amounted to:
A. 70,000
B. 112,000
C. 120,000
D. 135,000

32. How much is the understatement of the home office account prior to closing entries?
A. 60,000
B. 96,000
C. 156,000
D. 264,000

Solution:

30 – Ending inventory

Segregate cost of sales from local suppliers and from main office:

Local suppliers
Inventories, January 10,000
Purchases 30,000
Inventories, December (16,000)
24,000

The overvaluation in cost of sales represents the ‘markup’ realized from the cost of sales from the
main office.

0961-718-5293; 0936-407-4780; (02)-8376-0405 www.arccpalereview.com Page 14 of 28

Downloaded by ruzz ruzz (ruzzst@jruzz.ml)


lOMoARcPSD|12648093

AFAR | FIRST PREBOARD EXAMINATION ARC – ACCOUNTANCY REVIEW CENTER

True branch income 156,000


Branch income per branch books 60,000
Overvaluation in cost of sales 96,000
Compared with cost of sales from main office
(360,000 – 24,000) 336,000
Cost of sales without markup 240,000

Hence, the markup is 40% over cost (96,000/240,000).

Next, we determine the ending inventory:

From local suppliers 16,000


From main office (100,000 – 16,000)/1.4 60,000
Ending inventory at cost 76,000

31

Beginning inventory (80,000 – 10,000) x 40/140 20,000


Shipments from main office (350,000 x 40/140) 100,000
Unadjusted balance 120,000

32

The problem is simply asking for the net income in the books of the branch. Net income is closed
to home office account after the closing entries; hence, this represents the amount of
understatement of the home office account.

Problem 33 to 35.

On June 30, 2020, an entity enters into three contracts (Contract A, B and C) with separate customers to provide
services. Each contract has a two-year noncancelable term.

Contract A
Cleaning services are to be provided over the next two years typically at least once per month. For services
provided, the customer pays an hourly rate of P =25. Because the entity bills a fixed amount for each hour of
service provided, the entity has a right to invoice the customer in the amount that corresponds directly with
the value of the entity's performance completed to date.

Contract B
Cleaning services and lawn maintenance services are to be provided as and when needed with a maximum of
four visits per month over the next two years. The customer pays a fixed price of P =400 per month for both
services. The entity measures its progress towards complete satisfaction of the performance obligation using a
time-based measure.

Contract C
Cleaning services are to be provided as and when needed over the next two years. The customer pays fixed
consideration of P=100 per month plus a one-time variable consideration payment ranging from P =0-P=1,000
corresponding to a one-time regulatory review and certification of the customer's facility (i.e. a performance
bonus). The entity estimates that it will be entitled to P
=600 of the variable consideration. On the basis of the
entity's assessment of the factors in paragraph 57 of PFRS 15, the entity includes its estimate of =
P600 of variable
consideration in the transaction price because it is highly probable that a significant reversal in the amount of
cumulative revenue recognized will not occur. The entity measures its progress towards complete satisfaction
of the performance obligation using a time-based measure.

Under PFRS 15, the entity discloses the amount of the transaction price that has not yet been recognized as
revenue in a table with quantitative time bands that illustrates when the entity expects to recognize the amount
as revenue.

0961-718-5293; 0936-407-4780; (02)-8376-0405 www.arccpalereview.com Page 15 of 28

Downloaded by ruzz ruzz (ruzzst@jruzz.ml)


lOMoARcPSD|12648093

AFAR | FIRST PREBOARD EXAMINATION ARC – ACCOUNTANCY REVIEW CENTER

33. In relation to Contract A, to comply with the requirements of PFRS 15, how will the entity disclose the
transaction price that has not yet been recognized as revenue in the financial statements as of December
31, 2020?
A. No disclosure may be warranted.
B. Disclose the expected revenue (or services to be rendered) for the next year only.
C. Disclose the expected revenue (or services to be rendered) for the remaining term of the service
contract.
D. Disclose the expected revenue (or services to be rendered) for a reasonable period of time.

34. In relation to Contract B, how much is the unrecognized revenue on this contract as of December 31, 2020?
A. 9,600
B. 7,200
C. 4,800
D. 2,400

35. In relation to Contract C, the entity discloses qualitatively that part of the performance bonus has been
excluded from the disclosure because it was not included in the transaction price. That part of the
performance bonus was excluded from the transaction price in accordance with the requirements for
constraining estimates of variable consideration. Quantitatively, however, the information for Contract C
related to the transaction price that has not yet been recognized as revenue as of December 31, 2020, will
include an overall disclosure of:
2021 2022
A. 1,600 800
B. 1,500 750
C. 1,200 600
D. 1,200 1,200

Solution:

34

Total revenue for entire term (400 x 24 months) 9,600


Revenue recognized in 2020 (400 x 6 months) 2,400
Unrecognized revenue 7,200

35

Determine the transaction price.

Fixed consideration (100 x 24 months) 2,400


Variable consideration 600
Transaction price recognized over time 3,000

Recognition of revenue will be as follows:

2020 – 3,000 x 6/24 months 750


2021 – 3,000 x 12/24 months 1,500
2022 – 3,000 x 6/24 months 750
3,000

These problems were lifted from PFRS 15 illustrative examples.

36. Which of the following statements about partnership accounts is true?


A. . The drawing account is credited with the partner’s withdrawals of cash or noncash assets during the
period.
B. Three accounts are generally maintained for each partner, the drawing account, the capital account
and the loans to/from account.
C. B is true, A is false
D. A and B are both false

0961-718-5293; 0936-407-4780; (02)-8376-0405 www.arccpalereview.com Page 16 of 28

Downloaded by ruzz ruzz (ruzzst@jruzz.ml)


lOMoARcPSD|12648093

AFAR | FIRST PREBOARD EXAMINATION ARC – ACCOUNTANCY REVIEW CENTER

37. Lorie and Sally formed a partnership and agreed to divide initial capital in the ratio of 55:45, even though
Lorie contributed P100,000 and Sally contributed P90,000 in identifiable assets. Under the bonus approach
to adjust the capital accounts, Lorie’s unidentified assets should be debited (credited) for how much?
A. 4,500
B. 16,000
C. 8,000
D. 0

38. On January 1, 2020, A and B agreed to form a partnership out of their separate businesses. The balance
sheet presented below shows their financial position as of December 31, 2019:

A B
Cash ₱ 24,000 ₱ 30,000
Accounts Receivable, net 144,000 48,000
Inventories 198,000 265,000
Prepaid Expenses - 30,000
Store Equipment 240,000 200,000
Accumulated Depreciation (95,000) (80,000)
Land 400,000 -
Building 800,000 -
Accumulated Depreciation (200,000) -
Total ₱ 1,511,000 ₱ 493,000

Accounts Payable 60,000 58,000


Loans Payable 550,000 -
Owner's, Capital 901,000 435,000
Total ₱ 1,511,000 ₱ 493,000

A’s assets has the following fair values: Inventories, P150,000; Store Equipment, P150,000; Land, P450,000
and Building, P550,000. Receivables were initially provided with 4% allowance. Both A and B decided to
double. B’s assets are stated at their fair values except for prepaid expenses which the parties agreed to
mark 0. They agreed to divide the capital in the ratio of 70:30. How much shall be debited to A’s capital as
an adjustment in effecting the bonus method?
a. P0
b. P26,500
c. P46,500
d. P55,000
Reconstruct: Alternative Solution:
A B A B
Cash 24,000 30,000 Owner's, Capital 901,000 435,000
Accounts Receivable, net 138,000 46,000 Adjustments:
Inventories 150,000 265,000 AR (6,000) (2,000)
Prepaid Expenses - - Inventories (48,000)
Store Equipment 150,000 200,000 Store Equipment 5,000
Accumulated Depreciation (80,000) Land 50,000
Land 450,000 - Building (50,000)
Building 550,000 - Prepaid Expenses (30,000)
Accumulated Depreciation - Adjusted 852,000 403,000
Total 1,462,000 461,000
A + B capital 1,255,000
Accounts Payable 60,000 58,000 A, Capital 878,500 70%
Loans Payable 550,000 - B, Capital 376,500 30%
Owner's, Capital 852,000 403,000
Total 1,462,000 461,000 Adjustment due to bonus method
B, Capital 26,500
A + B capital 1,255,000 A, Capital 26,500
A, Capital 878,500 70%
B, Capital 376,500 30%

Adjustment due to bonus method


B, Capital 26,500
A, Capital 26,500 >> credited, not debited

0961-718-5293; 0936-407-4780; (02)-8376-0405 www.arccpalereview.com Page 17 of 28

Downloaded by ruzz ruzz (ruzzst@jruzz.ml)


lOMoARcPSD|12648093

AFAR | FIRST PREBOARD EXAMINATION ARC – ACCOUNTANCY REVIEW CENTER

39. Peter admits Theo as a partner. Peter’s accounts on the partnership commencement show the following:
Cash ₱ 30,000
Accounts receivable 150,000
Merchandise Inventory 180,000
Short term payables 52,000
Peter, Capital 308,000
For the purpose of finalizing Peter’s balances, the following were considered:
1. An allowance for doubtful accounts of 2% shall be established.
2. Merchandise inventory has a fair value of P175,000. Peter and Theo agreed to value it by P10,000
lower than the book value.
3. Prepaid rent of P6,300 and accrued interest of P3,000.

Theo shall invest cash for a 2/5 interest in the partnership. How much shall Theo contribute then?
a. P298,300
b. P198,867
c. P202,200
d. P204, 200

Peter, Capital 308,000


Adjustments:
Allowance for D/A (3,000)
Inventory (10,000)
Prepaid Rent 6,300
Accrued Interest (3,000)
Peter, Capital (adjusted) 298,300
Theo's Contribution 198,867 >> Peter, Capital * 2/3

40. Which of the following statements below is true?


a. A newly established partnership shall be accounted as a joint operation in accordance with PFRS 11.
b. A partnership has a juridical personality that’s dependent on each partner’s financial capacity as general
partners have unlimited liability.
c. General Professional Partnerships are not subject to corporate taxes.
d. A contract of partnership is consensual, nominate and remuneratory.

41. Which among the partnerships established below is considered valid?


a. A and B formed a partnership with a capitalization of P50,000 in cash. The agreement was executed
orally but was not registered to SEC.
b. A and B formed a partnership for the sale of merchandise items as forefront. However at night, their
office building secretly holds gambling activities.
c. A and B formed a partnership in writing. A contributed ownership rights to a commercial space.
d. A and B formed a partnership in public instrument. B contributed ownership rights to a parcel of land. An
inventory of said property is attached and signed by B.

42. In accordance with the Civil Code, if only the sharing of losses has been agreed upon by the parties to a
partnership, how shall the sharing profits be made?
a. equally
b. the same as the share in losses, however, industrial partners should not share in losses
c. the same as the share in losses and industrial partners shall be prioritized with bonuses
d. in the ratio of initial capital balances

43. Which of the following is not a component formula for the distribution of partnership profit and losses?
a. Salary allocation of managing partners even in case of loss.
b. Payment of bonus to identified partners. Payment eventually results to negative residual profit.
c. Interest on loans from partners.
d. Payment of bonus after salaries, interest and bonus.

44. ABC partnership which started operations on the second quarter of 2020 specifies that partnership net
income be allocated as follows:
A B C
Salary 40,000 25,000 35,000
Interest on average capital balances 10% 12% 10%
Residual 1/2 1/3 1/6

0961-718-5293; 0936-407-4780; (02)-8376-0405 www.arccpalereview.com Page 18 of 28

Downloaded by ruzz ruzz (ruzzst@jruzz.ml)


lOMoARcPSD|12648093

AFAR | FIRST PREBOARD EXAMINATION ARC – ACCOUNTANCY REVIEW CENTER

Average capital balances for the current year were P30,000 for A, P45,000 for B, and P40,000 for C.
Partners A and B are given annual bonus equal to 30% of net income after salaries, interest and bonus.
Assuming a current year net income of P200,000, how much shall C receive as share in profit?
a. P90, 100
b. P70,267
c. P39,633
d. P50,900

A B C
Salary 40,000 25,000 35,000
Interest on average capital ba 10% 12% 10%
Residual 1/2 1/3 1/6
Average Capital Balances 30,000 45,000 40,000

Net Income 200,000

A B C Total
Salary 30,000 18,750 26,250 75,000 >> for 9 months
Interest on ave. capital bal. 2,250 4,050 3,000 9,300 >> for 9 months
Bonus 26,700 26,700 - 53,400 >> no need to prorate for 9 months as
Residual 31,150 20,767 10,383 62,300 income covers 9 months only
Total 90,100 70,267 39,633 200,000
Check -
B = 30% (NI - Salary - Interest - Bonus)

45. ABC partnership which started operations on the last quarter of 2020 specifies that partnership net income
be allocated as follows:
A B C
Salary 35,000 30,000 40,000
Interest on average capital balances 8% 10% 12%
Residual 1/6 1/3 1/2

Average capital balances for the current year were P10,000 for A, P15,000 for B, and P20,000 for C.
Partners A shall be given annual bonus equal to 40% of net income after salaries, interest and bonus.
Assuming a current year net income of P25,000, how much shall B receive as share in profit?
a. P8,546
b. P9,388
c. P8,067
d. P7,067

A B C
Salary 35,000 30,000 40,000
Interest on average capital ba 8% 10% 12%
Residual 1/6 1/3 1/2
Average Capital Balances 10,000 15,000 20,000

Net Income 25,000

A B C Total
Salary 8,750 7,500 10,000 26,250 >> for 3 months
Interest on average capital ba 200 375 600 1,175 >> for 3 months
Bonus - >> no bonus
Residual (404) (808) (1,213) (2,425)
Total 8,546 7,067 9,388 25,000
Check -
B = 30% (NI - Salary - Interest - Bonus)

0961-718-5293; 0936-407-4780; (02)-8376-0405 www.arccpalereview.com Page 19 of 28

Downloaded by ruzz ruzz (ruzzst@jruzz.ml)


lOMoARcPSD|12648093

AFAR | FIRST PREBOARD EXAMINATION ARC – ACCOUNTANCY REVIEW CENTER

46. R, M and T have capital balances of P30,000, P20,000 and P40,000, respectively. Profit/Losses shall be
divided as follows:
a. 10% interest on capital balances
b. M is entitled to a salary of P12,000
c. T is guaranteed a minimum share of P24,000.
d. Remaining profit shall be divided as 30:30:40

Minimum profit to have M’s share reach at least P20,000 is:


a. P56,000
b. P53,000
c. P48,000
d. P60,000

R M T Total
Interest 3,000 2,000 4,000 9,000
Capital Balances 30,000 20,000 40,000

Salary - 12,000 - 12,000


Residual 6,000 6,000 20,000 32,000
% 30% 30% 40%
Total 9,000 20,000 24,000 53,000

Note: Squeeze for M's residual income first. Don't forget that T's guaranteed share is P24,000.

47. Isabel is trying to decide on her income sharing options:


Option 1: P50,000 salary + interest
Option 2: P25,000 salary + interest+ 10% bonus after salaries, interest and bonus

Salaries traceable to other partners amount to P75,000. Interests shall be given as follows:
Aliyah Leandro Isabel
P10,000 P5,000 P5,000

At what level of income shall her option be indifferent?


a. P390,000
b. P395,000
c. P260,000
d. P365,000

Option 1 =salary + interest


55,000

Option 2 =salary + interest + bonus


= 25,000 + 5,000 + 25,000 bonus (squeeze)

B = 10% (NI - S - I - B)
25,000 = 10% (NI - S - I - 25,000)
25,000 = 10% (NI - 100,000 - 20,000 - 25,000)
25,000 = 10%NI - 10,000 - 2,000 - 2,500)
25,000 = 10%NI - 10,000 - 2,000 - 2,500)
39,500 = 10% NI
395,000 = NI

48. It is the change in the relation of the partners caused by any partner ceasing to be associated in the carrying
out of the business.
a. Liquidation
b. Tampuhan
c. Dissolution
d. Cool off

0961-718-5293; 0936-407-4780; (02)-8376-0405 www.arccpalereview.com Page 20 of 28

Downloaded by ruzz ruzz (ruzzst@jruzz.ml)


lOMoARcPSD|12648093

AFAR | FIRST PREBOARD EXAMINATION ARC – ACCOUNTANCY REVIEW CENTER

49. In cases of dissolution, the partnership assets and liabilities at the date of dissolution may need to be
revalued to their fair values. Any revaluation increase or decrease is
a. allocated to all of the existing partners as at the date of dissolution
b. allocated to remaining partners after the dissolution
c. allocated only to the partner ceasing to be associated with the partnership
d. revaluation shall not be made

50. Joy owns 70% interest in the partnership with a capital balance of P280,000. Dory on the other hand owns
30% interest and has a capital balance of P120,000. Ivy invests P100,000 for a 25% interest. Which of the
bonus or goodwill method will give Ivy an advantage in capital credit and by how much?
a. Bonus method, P25,000
b. Goodwill method, P33,333
c. Bonus, Method, P8,333
d. Goodwill Method, P8,333
TCC Bonus TAC
Joy 280,000 (17,500) 262,500 70% old ownership ratio
Dory 120,000 (7,500) 112,500 30% old ownership ratio
Ivy 100,000 25,000 125,000 25% acquired interest
500,000 - 500,000
-
TCC Goodwill TAC
Joy 280,000 280,000 70% old ownership ratio
Dory 120,000 120,000 30% old ownership ratio
Ivy 100,000 33,333 133,333 25% acquired interest
500,000 533,333
-
8,333 advantage using goodwill method
TCC - Total Contributed Capital
TAC - Total Agreed Capital
Under Goodwill Method - TAC > TCC

51. On August 30, 2020, C of CPA Partnership decided to retire from the group as he plans to migrate outside
the country and can no longer dedicate enough time to help manage the business’ affairs. As of the date,
the partnership’s balance sheet follows:
Total Assets 200,000

C, Loan 19,000
C, Capital 42,000 30%
P, Capital 39,000 25%
A, Capital 100,000 45%
Total Assets 200,000

By agreement, totals assets shall be adjusted to their fair value of P220,000. C shall be paid P55,200 for
his capital. Loans due to him shall be paid in full as well. After the retirement, how much is A’s capital
balance?
a. P74,200
b. P41,430
c. P104,370
d. P145,800

Total Assets 200,000 220,000 220,000


(adj) (adj after retirement)

C, Loan 19,000 19,000 - 19,000


C, Capital 42,000 30% 48,000 7,200 55,200
P, Capital 39,000 25% 44,000 (2,571) 41,429
A, Capital 100,000 45% 109,000 (4,629) 104,371
Total Assets 200,000 220,000 - 220,000

Adjusted FV of assets 220,000


Increment 20,000

0961-718-5293; 0936-407-4780; (02)-8376-0405 www.arccpalereview.com Page 21 of 28

Downloaded by ruzz ruzz (ruzzst@jruzz.ml)


lOMoARcPSD|12648093

AFAR | FIRST PREBOARD EXAMINATION ARC – ACCOUNTANCY REVIEW CENTER

52. Anna and Christian are partners with capital account balances of P160,000 and P190,000 respectively.
They agree to admit Patricia in the partnership with a 2/5 interest in capital and profits for an investment
of P200,000 after the appropriate revaluation of partnership assets. Applying the guidelines under the
Philippine Financial Reporting Standards, how much goodwill shall be recognized in this transaction?
a. P0
b. P83,333
c. P100,000
d. P116,667

53. Carol got admitted to the partnership of Angeline and Beauty. Just before her admission, the financial
position of Angeline and Beauty shows the following:
Cash 50,000
Receivables 150,000
Inventories 160,000
Fixed Assets (50% EUL) 500,000
Total Assets 860,000

Short-term payables 80,000


Angeline, Capital (60%) 505,000 60%
Beauty, Capital (40%) 275,000 40%
Total Liabilities and Equity 860,000

The following adjustments are required:


a. The recoverability of receivables is estimated at 90%.
b. Inventories have NRV of P150,000.
c. Fixed Assets have current replacement cost of P900,000.
d. Unrecorded liabilities amounted to P25,000.

Carol acquires half of Beauty's capital for P120,000. How much is Beauty’s capital after admission?
a. P137,500
b. P117,500
c. P127,500
d. P115,000

Cash 50,000 Adjustments:


Receivables 150,000 Doubtful Accounts (15,000) >> 10% allowance
Inventories 160,000 Inventory Writedown (10,000) >> 150k - 160k
Fixed Assets (50% EUL) 500,000 FA Writedown (50,000) >> 900k *50% - 500k
Total Assets 860,000 Unrecorded Liabilities (25,000)
(100,000)
Short-term payables 80,000
Angeline, Capital (60%) 505,000 60% (60,000) 445,000
Beauty, Capital (40%) 275,000 40% (40,000) 235,000
Total Liabilities and Equity 860,000
Carol acquires half of Beauty's capital for P120,000.
The acquisition is a personal transaction.
Beauty's capital after admission:
117,500 >> 235k *50%

0961-718-5293; 0936-407-4780; (02)-8376-0405 www.arccpalereview.com Page 22 of 28

Downloaded by ruzz ruzz (ruzzst@jruzz.ml)


lOMoARcPSD|12648093

AFAR | FIRST PREBOARD EXAMINATION ARC – ACCOUNTANCY REVIEW CENTER

54. Carol got admitted to the partnership of Angeline and Beauty. Just before her admission, the financial
position of Angeline and Beauty shows the following:
Cash 50,000
Receivables 150,000
Inventories 160,000
Fixed Assets (50% EUL) 500,000
Total Assets 860,000

Short-term payables 80,000


Angeline, Capital (60%) 505,000 60%
Beauty, Capital (40%) 275,000 40%
Total Liabilities and Equity 860,000

The following adjustments are required:


a. The recoverability of receivables is estimated at 90%.
b. Inventories have NRV of P150,000.
c. Fixed Assets have current replacement cost of P900,000.
d. Unrecorded liabilities amounted to P25,000.

Carol invested P150,000 for 20% ownership. The entry to record the transaction would include?
a. Debit to Angeline, Capital by P6,400
b. Debit to Cash by 164,000
c. Debit to Beauty, Capital by P6,400
d. Credit to Angeline, Capital by P9,600

Cash 50,000 Adjustments:


Receivables 150,000 Doubtful Accounts (15,000) >> 10% allowance
Inventories 160,000 Inventory Writedown (10,000) >> 150k - 160k
Fixed Assets (50% EUL) 500,000 FA Writedown (50,000) >> 900k *50% - 500k
Total Assets 860,000 Unrecorded Liabilities (25,000)
(100,000)
Short-term payables 80,000
Angeline, Capital (60%) 505,000 60% (60,000) 445,000
Beauty, Capital (40%) 275,000 40% (40,000) 235,000
Total Liabilities and Equity 860,000

Carol invested P150,000 for 20% ownership.


Angeline 445,000 (9,600) 435,400 60% old p/l ratio
Baeuty 235,000 (6,400) 228,600 40% old p/l ratio
Carol 150,000 16,000 166,000 20% ownership acquired
830,000 - 830,000
-
Cash 150,000
Carol, Capital 150,000

Angeline, Capital 9,600


Beauty, Capital 6,400
Carol, Capital 16,000

0961-718-5293; 0936-407-4780; (02)-8376-0405 www.arccpalereview.com Page 23 of 28

Downloaded by ruzz ruzz (ruzzst@jruzz.ml)


lOMoARcPSD|12648093

AFAR | FIRST PREBOARD EXAMINATION ARC – ACCOUNTANCY REVIEW CENTER

55. Abby, Bianca and Camille have capital balances of P224,000, P260,000 and P116,000, respectively. P/L
ratio is 3:2:1. Dina invest cash for a 1/4 interest. Assuming Dina is credited with P40,000 bonus upon
admission, how much did she invest?
a. P146,667
b. P200,000
c. P186,667
d. P160,000

TCC Bonus TAC


Abby 224,000 (20,000) 204,000 3.00 old p/l ratio
Bianca 260,000 (13,333) 246,667 2.00 old p/l ratio
Camille 116,000 (6,667) 109,333 1.00 old p/l ratio
Dina 146,667 40,000 186,667 1/4 ownership acquired
746,667 - 746,667
186,667
40,000 checking

(600,000+x)*.25 = x+ 40,000
150,000 + .25x = x + 40,000
.75x = 110,000
x= 146,667

56. In a joint arrangement, which of the following establishes joint control?


a. mutual sharing of control
b. ownership interest of more than 20%
c. contractual arrangement
d. majority stock ownership

57. TC, Inc. and RC, Inc. are joint operators in the development of new android phone. Each of them retains
control over the assets contributed to the arrangement and share profits and losses equally. During the
year ending December 31, 2020, TC earns revenue amounting P500,000 from its own operation. Sales of
android phone amounted to P400,000. How much total revenue shall be reported by TC in his statement
of profit/loss for the year?
a. P700,000
b. P500,000
c. P450,000
d. P400,000

58. Which of the following is considered a joint venture?


a. The parties sharing joint control obtain rights and obligations over the joint arrangement’s assets and
liabilities.
b. The parties sharing joint control obtain rights over the joint arrangement’s net assets.
c. The joint arrangement is structured in a separate vehicle as a corporation.
d. b or c

59. X, Y and Z formed a joint operation for the sale of stuffed toys especially now that sales is at its peak with
the celebration of Valentine’s Day. Their transactions during the month are summarized below:
Joint Operation
Merchandise – X 12,500 Cash Sales – Z 10,000
Merchandise – Y 3,000 Credit Sales – Z 8,000
Freight-in – Z 100
Purchases – Z 1,500
Selling Expenses – Z 200

The joint arrangement is to provide allowance for doubtful accounts at 10% of credit sales. With the close
of books for the month, how much is the joint operation’s profit (loss)?
a. P700
b. P(100)
c. P300
d. P(500)

0961-718-5293; 0936-407-4780; (02)-8376-0405 www.arccpalereview.com Page 24 of 28

Downloaded by ruzz ruzz (ruzzst@jruzz.ml)


lOMoARcPSD|12648093

AFAR | FIRST PREBOARD EXAMINATION ARC – ACCOUNTANCY REVIEW CENTER

Joint Operation
Merchandise – X 12,500 Cash Sales – Z 10,000
Merchandise – Y 3,000 Credit Sales – Z 8,000
Freight-in – Z 100
Purchases – Z 1,500
Selling Expenses – Z 200
Allowance for D/A 800
18,100 18,000
(100) Profit (Loss)

60. Drei owns 40% interest in Raph, Inc. and uses the equity method to account for its interest in the joint
venture. Drei has joint control over Raph, Inc. On April 1, 2020, Drei sold an equipment with remaining
book value of P235,000 to Raph, Inc. for P315,000. The said equipment has a remaining life of 5 years. Both
Drei and Raph uses the straight line method of depreciation.

Raph, Inc. reports profit of P500,000 and P650,000 in 2020 and 2021, respectively. How much shall be
Drei’s share in the profit of the joint venture in 2020?
a. P200,000
b. P120,000
c. P132,000
d. P136,000

Year 2020
Raph, Inc. 2020 profit 500,000
Drei's owership interest 40%
Drei's share in profit - unadjusted 200,000
Elimination of unrealized gain
from downstream transaction (80,000)
Realized gain by the passage of ti 12,000 (80,000 / 5 years) * 9/12
Drei's share in profit - adjusted 132,000

While the sale is downstream and the unrealized gain should have been eliminated in
total, note that the asset sold is depreciable. The economic benefits of a depreciable
asset is realized by the passage of time.

61. Drei owns 30% interest in Raph, Inc. and uses the equity method to account for its interest in the joint
venture. Drei has joint control over Raph, Inc. On April 1, 2020, Raph, Inc. sold an equipment to Drei at a
gain of P90,000. The said equipment has a remaining life of 8 years. Both Drei and Raph uses the straight
line method of depreciation.

Raph, Inc. reports profit of P300,000 and P250,000 in 2020 and 2021, respectively. How much shall be
Drei’s share in the profit of the joint venture in 2021?
a. P90,000
b. P78,375
c. P75,000
d. P86,250

Year 2021
Raph, Inc. 2021 profit 250,000
Drei's owership interest 30%
Drei's share in profit - unadjusted 75,000
Realized gain by the passage of time 3,375 (90,000 / 8 years) * 30%
Drei's share in profit - adjusted 78,375

The transaction is upstream, so consider Drei's share only. Since the question is on
the second year (also full year), the adjustment is just on the realization of income.

0961-718-5293; 0936-407-4780; (02)-8376-0405 www.arccpalereview.com Page 25 of 28

Downloaded by ruzz ruzz (ruzzst@jruzz.ml)


lOMoARcPSD|12648093

AFAR | FIRST PREBOARD EXAMINATION ARC – ACCOUNTANCY REVIEW CENTER

62. NMB has its business of selling cars through installment. Pertinent data is shown for Yr. 1-3.
Year 1 Year 2 Year 3
Installment Sales 2,000,000 2,400,000 1,850,000

GP Rate ? ? 25%
Collections:
From Y1 sales 1,000,000 600,000 400,000
From Y2 sales 1,200,000 720,000
From Y3 sales 1,830,000

Upon the satisfaction of performance obligation, collection of these installment sales were not probable.
However, all payments received are nonrefundable. Applying the revenue recognition principles under
PFRS 15, how much revenue shall be recognized in Year 3?
a. P1,850,000
b. P2,000,000
c. P2,950,000
d. P3,830,000

Year 1 2,000,000
Year 3 1,830,000
Total 3,830,000
Theory:
No revenue shall be recognized without meeting the Step 1 criteria (CARPP). However, any collection may be
recognized as revenue only when either of these two circumstances occurs:
1. The entity has no remaining obligation to transfer goods or services to the customer and all, or
substantially all, of the consideration has been received and is non-refundable; or
2. The contact has been terminated and the consideration received is non-refundable.

63. Under PFRS 15, gross profit from an installment sale


a. is recognized in full at the point of sale
b. is initially deferred and amortized over the settlement using the effective interest method
c. is initially deferred and periodically recognized as the installment payments are received
d. is recognized upon the satisfaction of performance obligation at a point in time or over time.

64. Pol Corporation sells completed apartments on installment basis. At the inception of the year, it sold one
of its units costing P1,050,000 to a customer for P1.8 million under the following terms:
1. 15% shall be paid upon the execution of sale.
2. Balance is collectible in 10 annual installments of P153,000 starting December 31 this year.
3. 12% is the currently prevailing interest rate.
4. At the inception of sale, the collection was determined not probable.

At the end of Year 2 where collection remained improbable, how much shall be reported as deposit
liability?
a. P576,000
b. P528,578
c. P1,134,484
d. P1,800,000

Collection Schedule: PV factor PV Alternatively


Downpayment 270,000 1.00 270,000 270,000
Year 1 153,000 0.89 136,607
Year 2 153,000 0.80 121,971 153,000
Year 3 153,000 0.71 108,902 5.65 >> PV of ordinary annuity
Year 4 153,000 0.64 97,234 864,484
Year 5 153,000 0.57 86,816
Year 6 153,000 0.51 77,515 1,134,484
Year 7 153,000 0.45 69,209 -
Year 8 153,000 0.40 61,794
Year 9 153,000 0.36 55,173
Year 10 153,000 0.32 49,262
1,134,484

Year 1 to 2 collection 576,000

0961-718-5293; 0936-407-4780; (02)-8376-0405 www.arccpalereview.com Page 26 of 28

Downloaded by ruzz ruzz (ruzzst@jruzz.ml)


lOMoARcPSD|12648093

AFAR | FIRST PREBOARD EXAMINATION ARC – ACCOUNTANCY REVIEW CENTER

65. Drei Corporation sells completed apartments on installment basis. At the inception of the year, it sold one
of its units costing P950,000 to a customer for P1.5 million under the following terms:
1. 15% shall be paid upon the execution of sale.
2. Balance is collectible in 10 annual installments of P127,500 starting December 31 this year.
3. 8% is the currently prevailing interest rate.

Collection was determined probable for this sale. How much total revenue shall be recognized in year 1?
a. P1,148,978
b. P452,366
c. P1,080,535
d. P1,500,000

Collection Schedule: PV factor PV Alternatively


Downpayment 225,000 1.00 225,000 225,000
Year 1 127,500 0.93 118,056
Year 2 127,500 0.86 109,311 127,500
Year 3 127,500 0.79 101,214 PV of ordinary annuity 6.71
Year 4 127,500 0.74 93,716 855,535
Year 5 127,500 0.68 86,774
Year 6 127,500 0.63 80,347 1,080,535
Year 7 127,500 0.58 74,395 -
Year 8 127,500 0.54 68,884
Year 9 127,500 0.50 63,782 Installment Sales 1,080,535
Year 10 127,500 0.46 59,057 Interest Rev. 68,443
1,080,535 Total 1,148,978

66. A performance obligation is satisfied over time if


a. The customer receives and consumes the benefits provided by the entity’s performance as the entity
performs.
b. The entity’s performance creates or enhances an asset (e.g. work in progress) that the customer
controls as the asset is created or enhanced.
c. The entity’s performance does not create an asset with an alternative use to the entity and the entity
has an enforceable right to payment for performance completed to date.
d. All of the above

67. Which of the following describes the revenue recognition issue met in the accounting for construction
contracts?
a. Performance obligation regarding construction contracts are generally satisfied at a point in time.
Thus, the primary issue is the collectability of the consideration.
b. Performance obligation regarding construction contracts are generally satisfied over time. Thus, the
primary issue is the collectability of the consideration.
c. Construction contracts are generally for long term. The start and completion of the project may fall in
different accounting periods. Thus, the primary issue is the timing of recognition of revenue and costs.
d. Construction contracts general involve a high cost of financing. Thus, the primary concern is whether
the contractor can sustain the project cost until completion.

68. An entity uses the input method based on costs in measuring the progress of performance obligation
satisfied over time. Which of the following does not affect the computation of revenue to be recognized
each year?
a. Revenue previously recognized
b. Progress billings
c. Estimated cost to complete
d. Transaction Price

69. On January 1, 2020, Helen, Inc. sealed a contract with Leopold, Inc. for the construction of a building at a
contract price of P4.2 million. The entity uses the input method in determining the POC. Relevant data
follows:

1. Initial estimate of total construction cost is P3.8 million.


3. Cost data follow:
Year 1 Year 2 Year 3
Cost incurred to date P2,500,000 P3,800,000 P4,000,000
Est. cost to complete 1,300,000 700,000 -

0961-718-5293; 0936-407-4780; (02)-8376-0405 www.arccpalereview.com Page 27 of 28

Downloaded by ruzz ruzz (ruzzst@jruzz.ml)


lOMoARcPSD|12648093

AFAR | FIRST PREBOARD EXAMINATION ARC – ACCOUNTANCY REVIEW CENTER

How much gain on reversal of provision for onerous contract shall be recognized in Year 3?
a. P19,247
b. P18,333
c. P46, 667
d. P33,333

Year 1 Year 2 Year 3


Cost incurred to date 2,500,000 3,800,000 4,000,000
Estimated cost to complete 1,300,000 700,000 -
Estimated total cost 3,800,000 4,500,000 4,000,000

Contract Price 4,200,000 4,200,000 4,200,000

Estimated Total GP 400,000 (300,000) 200,000

% of completion 66% 84% 100%


Revenue to date 2,763,158 3,546,667 4,200,000
Less: Revenue - prior year 2,763,158 3,546,667
Revenue for the year 2,763,158 783,509 653,333
Cost per year (2,500,000) (1,300,000) (200,000)
GP/ GL for the year 263,158 (516,491) 453,333
Provision for onerous contract (46,667) 46,667 >> workback
P/L for the year 263,158 (563,158) 500,000
Target (300,000) 200,000

70. Viki, Inc. started a project with a contract price of P55 million. The cost incurred to date is P12.5 million
and the estimated cost to complete is still P38.2 million. The company is unable to reliably estimate its
percentage of completion despite having data on actual cost and estimated cost to complete. During the
year, how much is the balance of construction in progress?
a. P13,560,000
b. P50,700,000
c. P12,500,000
d. none

0961-718-5293; 0936-407-4780; (02)-8376-0405 www.arccpalereview.com Page 28 of 28

Downloaded by ruzz ruzz (ruzzst@jruzz.ml)

You might also like